Você está na página 1de 249

General Surgery MCQ

1. Skeletal muscle breakdown produces predominantly liberation of which


two amino acids?
A. Lysine.
B. Tyrosine.
C. Alanine.
D. Glutamine.
E. Arginine.
Answer: CD
DISCUSSION: Alanine is released from skeletal muscle and extracted by the
liver, where it is converted to new glucose. Glutamine is also released from
muscle and participates in renal acid-base homeostasis and serves as fuel for
rapidly growing cells such as enterocytes, stimulated macrophages, and
fibroblasts. Together, these two amino acids account for approximately two
thirds of the nitrogen released from skeletal muscle.

2. In catabolic surgical patients, which of the following changes in body


composition do not occur?
A. Lean body mass increases.
B. Total body water increases.
C. Adipose tissue decreases.
D. Body weight decreases.
Answer: A
DISCUSSION: Lean body mass represents the body compartment that
contains protein. Because critical illness stimulates proteolysis and increased
excretion of body nitrogen, this compartment is consistently reduced, not
increased. The change in body composition is associated with a loss of body

weight, an increase in total body water, and a decrease in body fat.

3. The hormonal alterations that follow operation and injury favor accelerated
gluconeogenesis. This new glucose is consumed by which of the following
tissues?
A. Central nervous system.
B. Skeletal muscle.
C. Bone.
D. Kidney.
E. Tissue in the healing wound.
Answer: ADE
DISCUSSION: Glucose is produced in increased amounts to satisfy the fuel
requirements of the healing wound. In addition, nerve tissue and the renal
medulla also utilize this substrate. Skeletal muscle primarily utilizes fatty
acids, and bone utilizes mineral substrate.

4. Cytokines are endogenous signals that stimulate:


A. Local cell proliferation within the wound.
B. The central nervous system to initiate fever.
C. The production of acute-phase proteins.
D. Hypoferremia.
E. Septic shock.
Answer:ABCD
DISCUSSION: Although cytokines exert primarily autocrine and paracrine
effects, they may also cause systemic effects.

5. The characteristic changes that follow a major operation or moderate to

severe injury do not include the following:


A. Hypermetabolism.
B. Fever.
C. Tachypnea.
D. Hyperphagia.
E. Negative nitrogen balance.
Answer: D
DISCUSSION: The characteristic metabolic response to injury includes
hypermetabolism, fever, accelerated gluconeogenesis, and increased
proteolysis (creating a negative nitrogen balance). Food intake is generally
impossible because of abdominal injury or ileus. With time, food intake
increases, but the patient generally experiences anorexia, not hyperphagia.

6. Shock can best be defined as:


A. Hypotension.
B. Hypoperfusion of tissues.
C. Hypoxemia.
D. All of the above.
Answer: B
DISCUSSION: Shock, no matter what the cause, is a syndrome associated
with tissue hypoperfusion. Tissue hypoperfusion leads to tissue hypoxia,
which may or may not be due to hypoxemia. Hypotension is a late sign of
shock and, therefore, is not a good clinical indicator of the presence of tissue
hypoperfusion.

7. Which of the following statements about continuous cardiac output


monitoring are true?
A. Continuous cardiac output monitoring may unmask events not detected by

intermittent cardiac output measurements.


B. Continuous cardiac output monitoring by the thermodilution method
requires continuous infusion of fluid injectate at a constant rate and
temperature.
C. The major advantage of the Fick method over the thermodilution method
of calculating cardiac output is that it is noninvasive, requiring only the
determination of oxygen consumption by respiratory gas analysis.
D. The technique of thoracic electrical bioimpedance utilizes sensors to
determine stroke volume by detecting changes in resistance to a small,
applied alternating current.
Answer: AD
DISCUSSION: Various techniques are available to measure cardiac output
continuously. The advantages of continuous cardiac output monitoring, as
compared with intermittent methods, are (1) previously undetected events
may be unmasked; (2) more prompt recognition of adverse events may be
achieved; and (3) earlier therapeutic intervention may be possible.
Continuous cardiac output monitoring using the thermodilution method
appears to be as accurate as the standard intermittent bolus method, but it
does not require fluid injectates. In this method, a modified pulmonary artery
catheter incorporating a thermal filament heats blood in the right ventricle at
pulsed intervals, and a distal thermistor detects the temperature change,
which can be related mathematically to cardiac output. The Fick method
combines respiratory gas analysis with oximetery to determine oxygen
consumption (V(overdot)O 2) and to estimate mixed venous and arterial
oxygen content differences, respectively. Cardiac output (CO) is then
determined from the formula: CO = V(overdot)O 2/ {C(a-v)O 2 10} @
V(overdot)O 2/ {SaO 2 - SvO 2) (Hb) (1.39) 10}. Thoracic electrical
bioimpedance is a technique by which the resistance to a small-amplitude
alternating current (i.e., the impedance) is measured using various electrodes.
The impedance change induced by each cardiac ejection is a function of the
stroke volume, which then can be used to calculate the cardiac output.

8. Which of the following statements regarding cytokines is incorrect?


A. Cytokines act directly on target cells and may potentiate the actions of one
another.
B. Interleukin 1 (IL-1) is a major proinflammatory mediator with multiple
effects, including regulation of skeletal muscle proteolysis in patients with
sepsis or significant injury.
C. Platelet-activating factor (PAF) is a major cytokine that results in platelet
aggregation, bronchoconstriction, and increased vascular permeability.
D. Tumor necrosis factor alpha (TNF-a), despite its short plasma half-life,
appears to be a principal mediator in the evolution of sepsis and the multiple
organ dysfunction syndrome because of its multiple actions and the
secondary cascades that it stimulates.
Answer: C
DISCUSSION: Cytokines are soluble peptide molecules that are synthesized
and secreted by a number of cell types in response to injury, inflammation,
and infection. Cytokines, which include the interleukins, tumor necrosis
factor, colony-stimulating factors, and the interferons, comprise only one
category of inflammatory mediators involved in the host response.
Endotoxin, complement fragments, eicosanoids, kinins, nitric oxide, oxidants,
and PAF are noncytokine mediators that also have important roles in the
systemic inflammatory response. IL-1 and TNF-a, like other cytokines, have
multiple effects on target cells and potentiate the actions of other mediators to
produce an amplified inflammatory response. TNF-a is thought to play a
central role in the stress response, particularly in response to endotoxemia.

9. True statements concerning hypoadrenal shock include which of the


following?
A. Adrenocortical insufficiency may manifest itself as severe shock refractory

to volume and pressor therapy.


B. The presence of hyperglycemia and hypotension may suggest the
diagnosis of shock due to adrenocortical insufficiency.
C. Hydrocortisone does not interfere with the serum cortisol assay and should
be given to hemodynamically unstable patients suspected of having
hypoadrenal shock.
D. The rapid adrenocorticotropic hormone (ACTH) stimulation test should be
performed to help establish the diagnosis of acute adrenocortical
insufficiency.
Answer: AD
DISCUSSION: Shock due to acute adrenocortical insufficiency is relatively
uncommon but must be considered when shock refractory to volume
replacement and pressor therapy is present. Hypoglycemia may be present.
Patients with high metabolic stress may exhibit adrenal insufficiency only
under conditions of severe stress; thus, a history of adrenal insufficiency or
steroid dependency need not be elicited. When adrenocortical insufficiency is
suspected, the rapid ACTH (cosyntropin) stimulation test should be
performed. Serum cortisol levels should be drawn before intravenous
administration of 250 mg. of cosyntropin, and 30 and 60 minutes afterward.
A peak cortisol level of less than 20 mg./100 ml. suggests abnormal adrenal
function. In a hemodynamically unstable patient therapy should be instituted
before the test results become available. Dexamethasone does not interfere
with the cortisol assay, and it is the corticosteroid of choice while the ACTH
stimulation test is being performed.

10. All of the following are true about neurogenic shock except:
A. There is a decrease in systemic vascular resistance and an increase in
venous capacitance.
B. Tachycardia or bradycardia may be observed, along with hypotension.
C. The use of an alpha agonist such as phenylephrine is the mainstay of

treatment.
D. Severe head injury, spinal cord injury, and high spinal anesthesia may all
cause neurogenic shock.
Answer: C
DISCUSSION: Neurogenic shock occurs when severe head injury, spinal
cord injury, or pharmacologic sympathetic blockade leads to sympathetic
denervation and loss of vasomotor tone. Both arteriolar and venous vessels
dilate, causing reduced systemic vascular resistance and a great increase in
venous capacitance. The patient's extremities appear warm and dry, in
contrast to those of a patient in cardiogenic or hypovolemic shock.
Tachycardia is frequently observed, though the classic description of
neurogenic shock includes bradycardia and hypotension. Volume
administration to fill the expanded intravascular compartment is the mainstay
of treatment. The use of alpha-adrenergic agonist is infrequently necessary to
treat neurogenic shock.

11. True statements regarding eicosanoids include which of the following?


A. Prostaglandins and thromboxanes are synthesized via the cyclo-oxygenase
pathway.
B. The vasoconstricting, platelet-aggregating, and bronchoconstricting effects
of thromboxane A 2 are balanced by the actions of prostacyclin, which
produces the opposite effects.
C. Leukotriene synthesis is inhibited by the action of nonsteroidal antiinflammatory drugs (NSAIDs).
D. The principal prostaglandins have a short circulation half-life and exert
most of their effects locally.
Answer: ABD
DISCUSSION: The eicosanoids are a group of compounds arising from the
metabolism of arachidonic acid. The prostaglandins and thromboxanes are

synthesized via the cyclo-oxygenase pathway; thus, their synthesis is blocked


by NSAIDs. Leukotrienes, on the other hand, are synthesized via the
lipoxygenase pathway. Prostacyclin, produced largely by vascular
endothelium, inhibits platelet aggregation and causes vasodilatation as well
as bronchodilatation. Its effects are balanced by those of thromboxane A 2,
which is produced by platelets and also local actions, including platelet
aggregation, vasoconstriction, and bronchoconstriction. The leukotrienes also
have pulmonary and hemodynamic effects and may be involved in the
physiologic responses associated with anaphylactic and septic shock.

12. Which of the following statements about delivery-dependent oxygen


consumption are true?
A. Below the critical oxygen delivery (D(overdot)O 2crit), one would expect
to see a decrease in the lactate-pyruvate ratio.
B. D(overdot)O 2crit may be increased in patients with sepsis.
C. A desirable goal in the treatment of shock is to achieve deliveryindependent oxygen consumption.
D. The oxygen extraction ratio remains constant as long as oxygen delivery
remains above D(overdot)O 2crit.
Answer: BC
DISCUSSION: Oxygen consumption is said to be delivery dependent below
a critical point, D(overdot)O 2crit, at which anaerobic metabolism
supervenes. Above this point, oxygen consumption is relatively independent
of oxygen delivery because the body's cells can compensate for falls in
oxygen delivery by extracting more oxygen. In the delivery-dependent
region, if cellular hypoxia is present, the lactate-pyruvate ratio rises, owing to
the switch to anaerobic metabolism. Generally, it is desirable to achieve
delivery-independent oxygen consumption, to avoid ongoing tissue hypoxia.
There is considerable debate, however, about the nature of the oxygen
consumptionoxygen delivery relationship in cases of established sepsis or

multiple organ dysfunction syndrome. In such cases, D(overdot)O 2crit may


be increased, although the therapeutic benefit of trying to achieve
supranormal oxygen delivery has not been firmly established.

13. All of the following may be useful in the treatment of cardiogenic shock
except:
A. Dobutamine.
B. Sodium nitroprusside.
C. Pneumatic antishock garment.
D. Intra-aortic balloon pump.
Answer: C
DISCUSSION: Cardiogenic shock occurs when the heart fails to generate
adequate cardiac output to maintain tissue perfusion. Intrinsic causes such as
myocardial dysfunction secondary to coronary artery disease, or extrinsic
causes such as pulmonary embolism, tension pneumothorax, and pericardial
tamponade, may produce cardiogenic shock. Principles of treatment of
cardiogenic shock are aimed at optimizing preload, cardiac contractility, and
afterload. Preload is usually adequate or high in cardiogenic shock.
Dobutamine is a useful inotropic agent, particularly when filling pressures are
high, because of its mild vasodilatory effect, as well as its effect to enhance
cardiac contractility. Afterload-reducing agents, such as sodium nitroprusside,
may be beneficial in cardiogenic shock in the setting of elevated filling
pressures, low cardiac output, and elevated systemic vascular resistance.
Cardiac output may improve with use of afterload-reducing agents by
decreasing myocardial wall tension and optimizing the myocardial oxygen
supply-demand ratio. The intra-aortic balloon pump (IABP), by providing
diastolic augmentation, reducing left ventricular afterload, and reducing
myocardial oxygen consumption, is sometimes useful in the treatment of
cardiogenic shock. The IABP is especially useful in lowcardiac output
postcardiotomy patients, in patients awaiting revascularization, and in

patients with acute myocardial infarction complicated by mitral insufficiency


or ventricular septal defect. The pneumatic antishock garment (PASG), which
causes an increase in systemic vascular resistance, is contraindicated in
cardiogenic shock.

14. Which of the following statements concerning monitoring techniques in


the intensive care unit are true?
A. Pulmonary artery and pulmonary capillary wedge pressure readings
should be made at end inspiration, to minimize ventilatory artifacts.
B. Continuous SvO 2 monitoring based on the technique of reflectance
spectrophotometry has been shown to be accurate and reliable.
C. Direct measurement of gastric intramucosal pH can be provided by
gastrointestinal tonometry.
D. Hyperlactatemia may be seen in a number of clinical conditions not
associated with tissue hypoxia, including liver disease and hypermetabolic
states.
Answer: BD
DISCUSSION: Many different monitoring techniques may be used to assess
the adequacy of therapy for shock. The pulmonary artery catheter can provide
important hemodynamic and oxygen transport data that are very useful in
directing therapy aimed at optimizing cardiac function and oxygen delivery.
Pulmonary artery and pulmonary capillary wedge pressure readings should be
made at end-expiration to minimize ventilatory artifacts. Continuous SvO 2
monitoring, an accurate, reliable method that combines pulmonary artery
catheterization with the technique of reflectance spectrophotometry, may
provide early warning signs of hemodynamic compromise or inadequate
oxygen delivery. Gastrointestinal tonometry provides information that allows
one to infer the adequacy of splanchnic tissue perfusion. In this technique,
intramucosal pH is calculated using the Henderson-Hasselbalch equation and
measurements of gut intraluminal PCO 2 and arterial bicarbonate

concentration. Serum lactate concentration may be monitored in shock to


detect metabolic acidosis associated with anaerobic metabolism; however,
mild to moderate hyperlactatemia may also be seen with liver disease, toxin
ingestion, and hypermetabolic states not associated with shock.

15. An 18-year-old man shot once in the left chest has a blood pressure of
80/50 mm. Hg, a heart rate of 130 beats per minute, and distended neck
veins. Immediate treatment might include:
A. Administration of one liter of Ringer's lactate solution.
B. Subxiphoid pericardiotomy.
C. Needle decompression of the left chest in the second intercostal space.
D. Emergency thoracotomy to cross-clamp the aorta.
Answer: AC
DISCUSSION: The finding of distended neck veins in conjunction with
hypotension should suggest tension pneumothorax or pericardial tamponade.
Absent ipsilateral breath sounds and a trachea deviated to the contralateral
side may provide additional evidence for a tension pneumothorax, the
immediate treatment of which is needle decompression of the chest in the
second or third intercostal space in the midclavicular line. Pericardial
tamponade may initially respond to volume administration by enhancing
preload. Pericardiocentesis may need to be performed emergently if
hemodynamic instability persists after an initial fluid bolus when signs of
compressive cardiogenic shock are present. Subxiphoid pericardiotomy
should be performed only in the operating room by experienced persons who
are trained to deal with penetrating cardiac injuries. There is no role for aortic
cross-clamping in this scenario of cardiogenic shock.

16. Which of the following statements are true of the multiple organ
dysfunction syndrome (MODS)?

A. The two-hit model proposes that secondary MODS may be produced


when even a relatively minor second insult reactivates, in a more amplified
form, the systemic inflammatory response that was primed by an initial insult
to the host.
B. The systemic inflammatory response syndrome (SIRS), shock due to
sepsis or SIRS, and MODS may be regarded as a continuum of illness
severity.
C. Prolonged stimulation or activation of Kupffer cells in the liver is thought
to be a critical factor in the sustained, uncontrolled release of inflammatory
mediators.
D. The incidence of MODS in intensive care units has decreased owing to
increased awareness, prevention, and treatment of the syndrome.
Answer: ABC
DISCUSSION: MODS is part of a clinical continuum that begins with the
systemic inflammatory response syndrome, which is the host's stress response
to any major insult such as injury or infection. MODS may develop as a
result of the initial insult, but more commonly, it develops following a second
or subsequent insult to the host. The two-hit theory holds that the systemic
inflammatory response is amplified following the second hit, such as
nosocomial pneumonia, leading to exaggerated, persistent release of
inflammatory mediators that contribute to the pathogenesis of MODS. The
liver appears to be a pivotal organ in the progression and outcome of MODS,
partly because of the activation and prolonged stimulation of the Kupffer
cells, which comprise the majority of the body's macrophage population.
Macrophages are known to play a critical role in the elaboration of numerous
inflammatory mediators. Despite advances in critical care and in the
understanding of the pathogenesis of MODS, the incidence of MODS
continues to increase without a significant improvement in outcome.

17. All of the following statements about hemorrhagic shock are true except:

A. Following hemorrhagic shock, there is an initial interstitial fluid volume


contraction.
B. Dopamine, or a similar inotropic agent, should be given immediately for
resuscitation from hemorrhagic shock, to increase cardiac output and improve
oxygen delivery to hypoperfused tissues.
C. The use of colloid solutions or hypertonic saline solutions is
contraindicated for treatment of hemorrhagic shock.
D. In hemorrhagic shock, a narrowed pulse pressure is commonly seen before
a fall in systolic blood pressure.
Answer: BC
DISCUSSION: Hemorrhagic shock is associated with a contraction of the
interstitial fluid compartment because of precapillary vasoconstriction and
reabsorption of interstitial fluid into the vascular compartment along
hydrostatic pressure gradients. Systolic hypotension may not be evident in
hemorrhagic shock until at least 30% or more of blood volume is
exsanguinated. A decrease in the pulse pressure (the difference between
systolic and diastolic pressures) may be observed with losses of 15% to 30%
of blood volume. Treatment of hemorrhagic shock includes intravascular
fluid administration and definitive control of the source of the hemorrhage.
Inotropic agents should not be started before volume resuscitation but may be
added to improve oxygen delivery to hypoxic tissues if volume
administration alone does not produce resuscitative goals. Colloid or
hypertonic saline solutions are not contraindicated in the treatment of
hemorrhagic shock; however, definitive evidence that such solutions are
better than standard crystalloid solutions is lacking.

18. Which of the following statements about septic shock are true?
A. A circulating myocardial depressant factor may account for the cardiac
dysfunction sometimes seen with shock due to sepsis or SIRS.
B. A cardiac index (CI) of 6 liters per minute per square meter of body

surface, a pulmonary capillary wedge pressure of 15 mm. Hg, and a systemic


vascular resistance index (SVRI) of 800 dynes-sec/(cm 5-m 2) is a
hemodynamic profile consistent with septic shock.
C. An increase in SvO 2 in septic patients may be explained by the finding of
anatomic arteriovenous shunts.
D. Results of human trials employing antimediator therapy, such as
antiendotoxin antibodies, IL-1 receptor antagonist, and tumor necrosis factor
(TNF) antibodies, have confirmed animal studies that demonstrate a
significant improvement in survival with the use of such agents.
Answer: AB
DISCUSSION: Shock due to sepsis or SIRS frequently manifests as a
hyperdynamic cardiovascular response, consisting of an elevated CI and a
decreased SVR or SVRI. Occasionally, myocardial depression may be seen,
characterized by increased ventricular volumes and decreased ejection
fractions. A circulating myocardial depressant factor, possibly TNF, may be
responsible for the cardiac dysfunction in such instances. The cause of the
increased SvO 2 frequently observed in septic patients is unclear, but it may
be secondary to bioenergetic failure, metabolic downregulation, or
microcirculatory maldistribution leading to physiologic shunting. True
anatomic arteriovenous shunting has not been demonstrated in humans in
septic shock. Treatment of septic shock consists of appropriate antibiotic use
and supportive therapy. Experimental antimediator therapies have not been
encouraging thus far in human clinical trials, despite the promising results
from many animal studies.

19. Which of the following statements are true of oxidants?


A. In addition to their pathophysiologic roles in inflammation, injury, and
infection, oxidants also have physiologic roles.
B. Oxidants may be generated from activated neutrophils and during
reperfusion following a period of ischemia.

C. The deleterious effects of oxidants include lipid peroxidation and cell


membrane damage, oxidative damage to DNA, and inhibition of adenosine
triphosphate (ATP) synthesis.
D. The mechanism of ischemia-reperfusion injury involved the catalytic
production of superoxide anion (O 2) by the enzyme xanthine oxidase.
Answer: ABCD
DISCUSSION: Oxidants are reactive oxygen metabolites that have both
physiologic and pathophysiologic roles. As potent oxidizing agents, oxidants
are involved in cytochrome P 450mediated oxidations, for example. In
pathophysiologic processes associated with inflammation, injury, and
infection, oxidants may be generated by activated neutrophils and in
ischemia-reperfusion injury. During ischemia, the enzyme xanthine oxidase
accumulates. When oxygen availability increases during reperfusion, O 2 is
formed in a reaction catalyzed by xanthine oxidase. Further oxidant
formation ensues, causing the production of H 2O 2 and the extremely
reactive hydroxyl ion (OH). Oxidants may cause direct cell damage by the
mechanisms of lipid peroxidation and cell membrane disruption, inhibition of
ATP synthesis, reduction of cellular nicotinamide adenine dinucleotide
(NAD), and oxidative damage to DNA and amino acids. In addition, oxidants
may have a chemotactic role, leading to leukocyte infiltration and activation,
causing further tissue damage by the release of cytotoxic proteases.

20. Which of the following statements about the role of the gut in shock and
sepsis are true?
A. Selective decontamination of the digestive tract with the use of oral
antibiotics has been shown to reduce nosocomial pneumonias and to improve
mortality rates.
B. Enteral nutrition, as compared with parenteral nutrition, preserves the
villus architecture of the gut.
C. Gut dysfunction may be an effect of shock, but it may also contribute to

the development of MODS by the mechanism of bacterial translocation.


D. As compared with parenteral nutrition, enteral nutrition is associated with
a reduction in septic morbidity.
Answer: BCD
DISCUSSION: The gut has a vital role in the pathophysiology of shock. The
splanchnic circulation is very vulnerable to the circulatory redistribution that
occurs in shock, thus, gut ischemia may occur early in the various shock
syndromes. Gut injury, as a result of ischemia or reperfusion injury, leads to
disruption in the intestinal mucosal barrier and increased gut permeability.
Translocation of enteric flora or bacterial toxins across the gut wall may then
occur, resulting in amplification of the systemic inflammatory response and
the development of multiple organ dysfunction. Gut dysfunction, therefore,
may perpetuate the inflammatory process. Various methods have been tried to
modulate the deleterious effects of gut dysfunction. Selective
decontamination of the digestive tract by oral antibiotics has been shown to
reduce the incidence of nosocomial pneumonias, but no improvement in
mortality has been demonstrated thus far with this controversial technique.
Early enteral nutrition probably has the biggest impact on the preservation of
gut architecture and function. When compared to parenteral nutrition, enteral
feeding is more cost effective and is associated with a lower rate of septic
morbidity.

21. Which of the following statements about head injury and concomitant
hyponatremia are true?
A. There are no primary alterations in cardiovascular signs.
B. Signs of increased intracranial pressure may be masked by the
hyponatremia.
C. Oliguric renal failure is an unlikely complication.
D. Rapid correction of the hyponatremia may prevent central pontine injury.
E. This patient is best treated by restriction of water intake.

Answer: A
DISCUSSION: Acute symptomatic hyponatremia is characterized by central
nervous system signs of increased intracranial pressure. Changes in blood
pressure and pulse are secondary to increased intracranial pressure. In the
absence of hypovolemia, asymptomatic patients may be treated by restriction
of water intake; however, in such patients, hyponatremia should be partially
corrected by parenteral sodium administration. Rapid correction, particularly
to hypernatremia, may lead to central pontine myelinolysis. Oliguric renal
failure may rapidly develop in severe hyponatremia.

22. Which of the following statements about total body water composition are
correct?
A. Females and obese persons have an increased percentage of body water.
B. Increased muscle mass is associated with decreased total body water.
C. Newborn infants have the greatest proportion of total body water.
D. Total body water decreases steadily with age.
E. Any person's percentage of body water is subject to wide physiologic
variation.
Answer: CD
DISCUSSION: Since fat contains little water, lean persons with a
proportionately greater muscle mass have a greater than expected volume of
total body water. Likewise, the female body habitus and obesity contribute to
decreased total body water percentage. The highest proportion of total body
water is found in newborn infants, and total body water decreases steadily
and significantly with age. The actual figure for a healthy person is
remarkably constant.

23. Which of the following statements about extracellular fluid are true?

A. The total extracellular fluid volume represents 40% of the body weight.
B. The plasma volume constitutes one fourth of the total extracellular fluid
volume.
C. Potassium is the principal cation in extracellular fluid.
D. The protein content of the plasma produces a lower concentration of
cations than in the interstitial fluid.
E. The interstitial fluid equilibrates slowly with the other body compartments.
Answer: B
DISCUSSION: The total extracellular fluid volume represents 20% of body
weight. The plasma volume is approximately 5% of body weight. Sodium is
the principal cation. The Gibbs-Donan equilibrium equation explains the
higher total concentration of cations in plasma. Except for joint fluid and
cerebrospinal fluid, the majority of the interstitial fluid exists as a rapidly
equilibrating component.

24. Which of the following statements are true of a patient with


hyperglycemia and hyponatremia?
A. The sodium concentration must be corrected by 5 mEq. per 100 mg. per
100 ml. elevation in blood glucose.
B. With normal renal function, this patient is likely to be volume overloaded.
C. Proper fluid therapy would be unlikely to include potassium
administration.
D. Insulin administration will increase the potassium content of cells.
E. Early in treatment adequate urine output is a reliable measure of adequate
volume resuscitation.
Answer: D
DISCUSSION: Each 100-mg. per 100 ml. elevation in blood glucose causes a
fall in serum sodium concentration of approximately 2 mEq. per liter. Excess
serum glucose acts as an osmotic diuretic, producing increased urine flow,

which can lead to volume depletion. Insulin therapy and the correction of the
patient's associated acidosis produce movement of potassium ions into the
intracellular compartment.

25. Which of the following statements about respiratory acidosis are true?
A. Compensation occurs by a shift of chloride out of the red blood cells.
B. Renal compensation occurs rapidly.
C. Retention of bicarbonate and increased ammonia formation are normal
compensatory mechanisms.
D. Narcotic administration is an unusual cause of respiratory acidosis.
E. The ratio of bicarbonate to carbonic acid is less than 20:1.
Answer: CE
DISCUSSION: Renal compensation for acute hypoventilation is relatively
slow. Depression of the respiratory center by morphine can lead to respiratory
acidosis. Renal retention of bicarbonate, ammonia formation, and shift of
chloride into red cells combine to increase the ratio of bicarbonate to
carbonic acid to 20:1.

26. Which of the following statements are true of elevatedanion gap


metabolic acidosis?
A. Hypoperfusion from the shock state rarely produces an elevated anion gap.
B. Retention of sulfuric and phosphoric acids may lead to this form of
acidosis.
C. Copious diarrhea does not produce this condition.
D. Rapid volume expansion may produce this form of acidosis.
E. Use of lactated Ringer's solution is inappropriate in the treatment of lactic
acidosis.
Answer: BC

DISCUSSION: An elevated anion gap may be produced by lactic acidosis


from shock or by retention of inorganic acids from uremia. Lactated Ringer's
solution rapidly corrects the lactic acidosis from hypovolemia, as lactate is
converted to bicarbonate with hepatic reperfusion. Bicarbonate loss from
diarrhea and dilutional acidosis are nonanion gap types of metabolic
acidosis.

27. Which of the following is true of loss of gastrointestinal secretions?


A. Gastric losses are best replaced with a balanced salt solution.
B. Potassium supplementation is unnecessary in replacement of gastric
secretions.
C. Bicarbonate wasting is an unusual complication of a high-volume
pancreatic fistula.
D. Balanced salt solution is a reasonable replacement fluid for a small bowel
fistula.
E. A patient with persistent vomiting usually requires hyperchloremic
replacement fluids.
Answer: DE
DISCUSSION: Gastric secretions are relatively high in chloride and
potassium. Other than an isolated pancreatic fistula, gastrointestinal tract
losses below the pylorus are best replaced by a balanced salt solution.
Although potassium concentrations are low, copious losses require potassium
supplementation to prevent hypokalemia.

28. Which of the following statements regarding hypercalcemia are true?


A. The symptoms of hypercalcemia may mimic some symptoms of
hyperglycemia.
B. Metastatic breast cancer is an unusual cause of hypercalcemia.
C. Calcitonin is a satisfactory long-term therapy for hypercalcemia.

D. Severely hypercalcemic patients exhibit the signs of extracellular fluid


volume deficit.
E. Urinary calcium excretion may be increased by vigorous volume repletion.
Answer: ADE
DISCUSSION: Markedly elevated serum calcium levels produce polydipsia,
polyuria, and thirst. Vigorous volume repletion and saline diuresis correct the
extracellular fluid volume deficit and promote the urinary excretion of
calcium. Metastatic breast cancer is the most common cause of
hypercalcemia, from bony metastasis. The calcitonin effect on calcium is
diminished with repeat administrations.

29. Which of the following statements about normal salt and water balance
are true?
A. The products of catabolism may be excreted by as little as 300 ml. of urine
per day.
B. The lungs represent the primary source of insensible water loss.
C. The normal daily insensible water loss is 600 to 900 ml.
D. Excessive cell catabolism causes significant loss of total body water.
E. In normal humans, urine represents the greatest source of daily water loss.
Answer: CE
DISCUSSION: The skin is the primary source of insensible water loss.
Including losses from the lungs, this averages 600 to 900 ml. per day.
Catabolism liberates water of solution. In normal humans, urine represents
the greatest source of water loss. The patient deprived of external access to
water must still excrete a minimum of 500 to 800 ml. of urine per day to
expel the products of catabolism.

30. Which of the following is/are not associated with increased likelihood of

infection after major elective surgery?


A. Age over 70 years.
B. Chronic malnutrition.
C. Controlled diabetes mellitus.
D. Long-term steroid use.
E. Infection at a remote body site.
Answer: C
DISCUSSION: Controlled diabetes mellitus has been shown repeatedly not
to be associated with increased likelihood of incisional infection provided
one avoids operations on body parts that may be ischemic or neuropathic.
Uncontrolled diabetes mellitus, such as ketoacidosis, is associated with a
dramatic increase in surgical infection. The other parameters notedage over
70, chronic malnutrition, regular steroid use, and an infection at a remote
body siteare well-recognized adverse predictive factors and are identified
in tables within the chapter.

31. Which of the following are not determinants of a postoperative cardiac


complication?
A. Myocardial infarct 4 months previously.
B. Clinical evidence of congestive heart failure in a patient with 8.5 gm. per
dl. hemoglobin.
C. Premature atrial or ventricular contractions on electrocardiogram.
D. A harsh aortic systolic murmur.
E. Age over 70 years.
Answer: B
DISCUSSION: Clinical evidence of congestive heart failure in a patient with
8.5 gm. per dl. hemoglobin concentration is a misleading sign. Evidence of
congestive failure is ordinarily a major risk factor, but in this particular
patient the anemia lends itself to correction by preoperative transfusion with

packed red blood cells, and often it is found that congestive failure and the
associated increased risks disappear when the hemoglobin concentration is
returned to the 12 gm. per dl. or higher ratio. All other factors are overt signs
of increased likelihood of a postoperative cardiac event, the most ominous
being a myocardial infarction 4 months preoperatively or the presence of a
harsh aortic systolic murmur suggesting the presence of aortic stenosis. Age
over 70 years and the presence of premature atrial or ventricular contractions
on the electrocardiogram are less strong determinants of a postoperative
cardiac complication.

32. Rank the clinical scenarios in order of greatest likelihood of serious


postoperative pulmonary complications.
A. Transabdominal hysterectomy in an obese woman that requires 3 hours of
anesthesia time.
B. Right middle lobectomy for bronchogenic cancer in a 65-year-old smoker.
C. Vagotomy and pyloroplasty for chronic duodenal ulcer disease in a 50year-old who had chest film findings of old, healed tuberculosis.
D. Right hemicolectomy in an obese 60-year-old smoker.
E. Modified radical mastectomy in a 58-year-old woman who is obese.
Answer: BDCAE
DISCUSSION: If one considers the constellation of risk factors for
pulmonary complications that is provided in tabular form in the
accompanying chapter, one should readily recognize B, right middle
lobectomy for bronchogenic cancer in a 65-year-old smoker, as the highest
risk of a clinical situation for the likelihood of serious pulmonary
complications. The next in rank may be properly debated between answer D
and answer C. D, right hemicolectomy, is judged to have somewhat greater
likelihood of complications since the patient is older, smokes, and is obese,
although the procedure may be done through a transverse or lower abdominal
incision. C, vagotomy and pyloroplasty, is viewed as being somewhat less

serious since it is an upper abdominal operation on an elective basis in a 50year-old whose only abnormalities include old, healed tuberculosis on a chest
film. A very low risk of pulmonary complication should follow a
transabdominal hysterectomy done through a lower abdominal incision in a
woman whose only risk factors are obesity and a 3-hour anesthesia time. The
lowest risk probably resides with the younger patient undergoing modified
radical mastectomy, whose only risk factor is obesity. This is particularly true
since this operation is conducted on the surface of the body, is associated
with relatively little postoperative pain, and provides free and unrestricted
respiratory function.

33. Rank the following laboratory tests and procedures in terms of their
relative value to a 65-year-old woman who is to undergo elective resection of
a sigmoid cancer.
A. Carcinoembryonic antigen (CEA).
B. Blood urea nitrogen (BUN).
C. Electrocardiogram (ECG).
D. Hemoglobin concentration (Hgb).
E. Serum creatinine (Cr).
F. Arterial blood oxygen tension (PaO 2) on room air.
G. Serum sodium concentration (Na+).
Answer: CDFEBAG
DISCUSSION: The most important test by far is the electrocardiogram, with
its capacity to indicate signs of occult heart disease. The second most
important evaluation is the hemoglobin concentration, which in this patient
may show an anemia related to chronic alimentary tract blood loss that would
require correction prior to safe induction of a general anesthetic. Arterial
blood gases vary from individual to individual depending primarily on
smoking habits and age. Accordingly, each older person should have a resting
baseline determination prior to operation. Serum creatinine may show

evidence of occult renal disease and is substantially more useful than blood
urea nitrogen, which is more vulnerable to transient volume changes.
Carcinoembryonic antigen is important to know in many patients with cancer
with respect to postoperative follow-up since in some cases it may be an early
herald of recurrent disease. However, it has little to do with the patient's
preoperative assessment in terms of risk and preparation for an elective
operation. The presence of liver metastases, for example, can be discovered
with significant accuracy by palpation at the time of operation, and an
elevated carcinoembryonic antigen in no set of circumstances would lead one
to withhold colon resection with its relief of potential obstruction and
bleeding. Finally, serum sodium concentration in a 65-year-old woman who
is admitted electively for resection of the colon is always normal and would
be of least value among these tests.

34. Which of the following statements regarding whole blood transfusion


is/are correct?
A. Whole blood is the most commonly used red cell preparation for
transfusion in the United States.
B. Whole blood is effective in the replacement of acute blood loss.
C. Most blood banks in the United States have large supplies of whole blood
available.
D. The use of whole blood produces higher rates of disease transmission than
the use of individual component therapies.
Answer: B
DISCUSSION: Whole blood is effective as a replacement fluid for acute
blood loss because it provides both volume and oxygen-carrying capacity
(red blood cells). It is rarely used in the United States nowadays, and most
blood banks do not provide whole blood transfusions. It is significantly more
efficient to separate donated blood into its components. In this manner, the
red blood cell mass can be used to provide oxygen-carrying capacity, the

plasma can be used for factor replacement, and the platelets and white cells
can be used for patients deficient in these components. The use of whole
blood to replace acute blood loss is associated with lower disease
transmission rates than the use of packed red blood cells, fresh frozen plasma,
and platelets, each from a different donor.

35. Which of the following statements about the preparation and storage of
blood components is/are true?
A. Solutions containing citrate prevent coagulation by binding calcium.
B. The shelf life of packed red blood cells preserved with CPDA-1 is
approximately 35 days at 1 to 6 C.
C. There are normal numbers of platelets in packed red blood cells stored at 1
to 6 C for more than 2 days.
D. The storage lesion affecting refrigerated packed red blood cells includes
development of acidosis, hyperkalemia, and decreased intracellular 2,3DPG
(diphosphoglycerate).
Answer: ABD
DISCUSSION: After blood has been collected from a donor, it is
anticoagulated with a solution containing citrate, which acts by binding
calcium. Blood is then separated into its components. Packed red blood cells
stored at 1 to 6 C using CPDA-1 preservative have a shelf life of 35 days.
There are essentially no functional platelets in refrigerated blood stored at 1
to 6 C after approximately 48 hours in storage. Refrigerated packed red
blood cells undergo progressive changes termed a storage lesion. Such
changes include acidosis, hyperkalemia, and decreased levels of 2,3-DPG,
which are reversed after transfusion or produce effects other than those
predicted based on the content of the unit of blood.

36. Which of the following is/are acceptable reasons for the transfusion of red

blood cells based on currently available data?


A. Rapid, acute blood loss with unstable vital signs but no available
hematocrit or hemoglobin determination.
B. Symptomatic anemia: orthostatic hypotension, severe tachycardia (greater
than 120 beats per minute), evidence of myocardial ischemia, including
angina.
C. To increase wound healing.
D. A hematocrit of 26% in an otherwise stable, asymptomatic patient.
Answer: AB
DISCUSSION: Currently accepted guidelines for the transfusion of packed
red blood cells include acute ongoing blood loss, as might occur in an injured
patient, and the development of symptomatic anemia with manifestations of
decreased tissue perfusion related to decreased oxygen-carrying capacity of
the blood. This includes situations in which the patient is unable to
compensate for a decreased oxygen-carrying capacity by the usual
mechanisms, such as increased cardiac output. Such patients develop
myocardial dysfunction if an excessive demand is placed on the heart. The
patient should be transfused with packed red blood cells, which afford added
oxygen-carrying capacity. This decreases the workload on the myocardium
while providing the necessary oxygen-delivery capability. The use of packed
red blood cells to improve wound healing or to improve the patient's sense of
well-being is highly questionable. No data support such a practice. In general,
the use of a transfusion trigger such as a hematocrit of 30% or hemoglobin of
10 gm. per dl. constitutes a questionable indication for transfusion. If a
patient is asymptomatic and stable and has no risk of myocardial ischemia,
packed red blood cell transfusion should not be given based solely or
predominantly on a numerical value such as a hematocrit of 28%.

37. The transfusion of fresh frozen plasma (FFP) is indicated for which of the
following reasons?

A. Volume replacement.
B. As a nutritional supplement.
C. Specific coagulation factor deficiency with an abnormal prothrombin time
(PT) and/or an abnormal activated partial thromboplastin time (APTT).
D. For the correction of abnormal PT secondary to warfarin therapy, vitamin
K deficiency, or liver disease.
Answer: CD
DISCUSSION: The use of FFP as a volume expander is not indicated. There
are currently several preparations (both crystalloid and colloid) that are
equally effective and do not carry the infectious and other risks associated
with the use of FFP. The use of FFP as a nutritional supplement is to be
condemned. Patients with specific deficiencies of coagulation factors
generally benefit greatly from the infusion of FFP. In cases of specific factor
deficiency, other preparations may be more appropriate, but FFP is generally
immediately available and is effective in most patients. Patients receiving
warfarin therapy, those who have vitamin K deficiency, and those with liver
dysfunction have abnormalities of the vitamin Kdependent factors II, VII,
IX, and X, as well as protein C and protein S.

38. In patients receiving massive blood transfusion for acute blood loss,
which of the following is/are correct?
A. Packed red blood cells and crystalloid solution should be infused to restore
oxygen-carrying capacity and intravascular volume.
B. Two units of FFP should be given with every 5 units of packed red blood
cells in most cases.
C. A six pack of platelets should be administered with every 10 units of
packed red blood cells in most cases.
D. One to two ampules of sodium bicarbonate should be administered with
every 5 units of packed red blood cells to avoid acidosis.
E. One ampule of calcium chloride should be administered with every 5 units

of packed red blood cells to avoid hypocalcemia.


Answer: A
DISCUSSION: Patients who are suffering from acute blood loss require
crystalloid resuscitation as the initial maneuver to restore intravascular
volume and re-establish vital signs. If 2 to 3 liters of crystalloid solution is
inadequate to restore intravascular volume status, packed red blood cells
should be infused as soon as possible. There is no role for prophylactic
infusion of FFP, platelets, bicarbonate, or calcium to patients receiving
massive blood transfusion. If specific indications exist patients should receive
these supplemental components. In particular, patients who have abnormal
coagulation tests and have ongoing bleeding should receive FFP. Patients
who have depressed platelet counts along with clinical evidence of oozing
(microvascular bleeding) benefit from platelet infusion. Sodium bicarbonate
is not necessary, since most patients who receive blood transfusion ultimately
develop alkalosis from the citrate contained in stored red blood cells. The use
of calcium chloride is usually unnecessary unless the patient has depressed
liver function, ongoing prolonged shock associated with hypothermia, or,
rarely, when the infusion of blood proceeds at a rate exceeding 1 to 2 units
every 5 minutes.

39. Hemostasis and the cessation of bleeding require which of the following
processes?
A. Adherence of platelets to exposed subendothelial glycoproteins and
collagen with subsequent aggregation of platelets and formation of a
hemostatic plug.
B. Interaction of tissue factor with factor VII circulating in the plasma.
C. The production of thrombin via the coagulation cascade with conversion
of fibrinogen to fibrin.
D. Cross-linking of fibrin by factor XIII.
Answer: ABCD

DISCUSSION: Hemostasis requires the interaction of platelets with the


exposed subendothelial structures at the site of injury followed by
aggregation of more platelets in that area. Interactions between endothelial
cell and subendothelial tissue factor with factor VII activate the coagulation
cascade. The end product is large amounts of thrombin that catalyze the
conversion of fibrinogen into fibrin. Fibrin thus formed is cross-linked by
factor XIII to form a stable clot that incorporates the platelet plug and fibrin
thrombus into a stable clot.

40. Which of the statements listed below about bleeding disorders is/are
correct?
A. Acquired bleeding disorders are more common than congenital defects.
B. Deficiencies of vitamin K decrease production of factors II, VII, IX, and
X, protein C, and protein S.
C. Hypothermia below 32C rarely causes a bleeding disorder.
D. Von Willebrand's disease is a very uncommon congenital bleeding
disorder.
Answer: AB
DISCUSSION: Acquired bleeding disorders are significantly more common
than congenital bleeding defects. Vitamin K deficiency may be related to
malnutrition or competitive inhibition of the production of the vitamin K
dependent factors II, VII, IX, X, protein C, and protein S by warfarin
(Coumadin). Hypothermia causes significant platelet dysfunction with a
significant bleeding disorder in many patients. It is among the least
recognized causes of altered coagulation in surgical patients. Von
Willebrand's disease is a relatively common disorder of bleeding and is
generally undetectable by routine screening methods.

41. The evaluation of a patient scheduled for elective surgery should always
include the following as tests of hemostasis and coagulation:
A. History and physical examination.
B. Complete blood count (CBC), including platelet count.
C. Prothrombin time (PT) and activated partial thromboplastin time (APTT).
D. Studies of platelet aggregation with adenosine diphosphate (ADP) and
epinephrine.
Answer: A
DISCUSSION: The evaluation of most patients scheduled for elective
surgery who do not have a history of significant bleeding disorders is
somewhat controversial. An adequate history and physical examination
screen out most patients with bleeding problems. For patients who are
scheduled to undergo a major surgical procedure, it is advisable to obtain a
CBC and platelet count, as well as a PT and APTT level. This detects a large
number of bleeding disorders but does not rule out all possible causes of
perioperative bleeding. Studies of platelet aggregation are indicated only for
patients who are suspected of having qualitative defects of platelet function
(e.g., von Willebrand's disease).

42. Which of the following statements regarding the transmission of


infectious agents through blood transfusions is/are true?
A. The transmission rates for human immunodeficiency virus (HIV) have
been decreasing progressively since the early 1980s.
B. The transmission rates of hepatitis have been decreasing steadily since the
1980s.
C. Cytomegalovirus (CMV) is the infectious agent most commonly
transmitted in blood.
D. Severely immunocompromised patients (such as patients undergoing
transplantation) should receive specially screened blood products.
Answer: ABCD

DISCUSSION: The incidence of both HIV and hepatitis transmitted via


blood transfusions has been steadily decreasing since the 1980s. This is
related to improved methods for detection and increased awareness of
surrogate markers of disease. The currently available techniques for the
detection of HIV are highly effective, provided the donor is not in the
window before the formation of specific antibody. The surrogate markers
for hepatitis C, as well as the specific assays for the organism, are now
sufficiently refined to allow the detection of a large percentage of hepatitis C
infection in donated blood. Screening for hepatitis B surface antigen has
effectively eliminated the transmission of hepatitis B through blood products
in most cases. CMV is the most commonly transmitted infectious agent in
blood. Since a large percentage of the population carry the virus, routine
screening is not performed for this organism; however, severely
compromised patients such as those undergoing transplantation should
receive CMV-negative blood products.

43. The most common cause of fatal transfusion reactions is:


A. An allergic reaction.
B. An anaphylactoid reaction.
C. A clerical error.
D. An acute bacterial infection transmitted in blood.
Answer: C
DISCUSSION: The most common cause of fatalities related to transfusion
reactions result from ABO-incompatible transfusion related to clerical error.
Most such reactions occur if a type O person receives type A red cells owing
to a clerical error that occurs either at the time the blood sample was drawn,
during processing in the laboratory, or at the time a unit is administered. The
importance of extremely careful labeling, transfer, and handling of specimens
and of cross-matched blood products cannot be overemphasized. Allergic and

other reactions are common but rarely fatal. The transmission of bacterial
organisms (e.g., Staphylococcus aureus) has been reported especially with
platelet concentrates maintained at or near room temperature. Fortunately,
such reactions are rare.

44. Which of the following statements about the coagulation cascade is/are
true?
A. The intrinsic pathway of coagulation is the predominant pathway in vivo
for hemostasis and coagulation.
B. The intrinsic pathway beginning with the activation of factor XII is the
predominant in vivo mechanism for activation of the coagulation cascade.
C. Deficiencies of factor VIII and IX cause highly significant coagulation
abnormalities.
D. Deficiencies of factor XII cause severe clinical bleeding syndromes.
Answer: AC
DISCUSSION: Although it was previously held that two somewhat distinct
pathways existed for the activation of the coagulation cascade, it is now
recognized that the predominant mechanism for coagulation in vivo is the
extrinsic pathway. Tissue factor is exposed in the subendothelial tissues
when endothelial cell injury occurs. Tissue factor then tightly binds factor VII
circulating in the plasma and activates the coagulation cascade. Factor VIII
and factor IX deficiency cause the clinical syndromes of hemophilia A and
hemophilia B, respectively. Both of these disorders involve very severe
clinical bleeding disorders, whereas deficiencies of factor XII do not
generally cause clinically significant bleeding. This further emphasizes the
secondary role that the intrinsic pathway plays in coagulation.

45. A major problem in nutritional support is identifying patients at risk.


Recent studies suggest that these patients can be identified. Which of the

following findings identify the patient at risk?


A. Weight loss of greater than 10% over 2 to 4 months.
B. Serum albumin of less than 3 gm. per 100 ml. in the hydrated state.
C. Malnutrition as identified by global assessment.
D. Serum transferrin of less than 220 mg. per 100 ml.
E. Functional impairment by history.
Answer: ABCDE
DISCUSSION: All of these are at least partially correct. It is not clear
whether weight loss of 10% or 15% is the required threshold, but it certainly
is close. Serum albumin of less than 3 gm per 100 ml. remains the most
constant identifier of patients at risk in the literature and has been so for
years. Global assessment in the hands of an experienced investigator is quite
efficacious at identifying persons at risk. Serum transferrin is certainly a
confirmatory identifier of patients with malnutritionand may be even a
primary one. Graham Hill and his co-workers have pioneered the concept of
global assessment using functional parameters, and in the hands of an
experienced observer is quite a reasonable way of approaching and
identifying patients at risk.

46. Essential fatty acid deficiency may complicate total parenteral nutrition
(TPN). Which of the following statements are true?
A. Essential fatty acid deficiency may be prevented by the administration of
1% to 2% of total calories as fat emulsion.
B. Fat-free parenteral nutrition results in the appearance of plasma
abnormalities, indicating essential fatty acid deficiency, within 7 to 10 days
of initiation.
C. An abnormal plasma eicosatrienoic-arachidonic acid ratio is always
associated with essential fatty acid deficiency.
D. Following initiation of fat-free parenteral nutrition, dry, scaly skin
associated with a maculopapular rash indicates essential fatty acid deficiency.

Answer: BD
DISCUSSION: Biochemical evidence of essential fatty acid deficiency may
occur as early as 7 to 10 days following initiation of fat-free parenteral
nutrition. The decrease in arachidonic acid in plasma and the appearance of
the abnormal eicosatrienoic acid may yield the earliest indication of
prostaglandin deficiency; it is not absolute. Decreased intraocular pressure,
another early indication of prostaglandin deficiency, may appear as soon as 7
days following initiation of fat-free parenteral nutrition. While my current
practice is to give at least 500 ml. of 10% lipid emulsion daily to provide
20% to 25% of total calories to support hepatic protein synthesis, as little as
4% to 6% of total daily calories as fat prevents essential fatty acid deficiency.
Practically, this may be undertaken by the administration of 500 ml. of 10%
lipid three times weekly. The appearance of eicosatrienoic acid and a
decrease in arachidonic acid, and a change in ratio, is not essential to the
diagnosis of essential fatty acid deficiency, but this plasma abnormality is
often present.

47. It is stated that enteral nutrition is safer than parenteral nutrition. Which
of the following may be complications of enteral nutrition?
A. Hyperosmolar, nonketotic coma.
B. Vomiting and aspiration.
C. Pneumatosis cystoides intestinalis.
D. Perforation and peritonitis.
Answer: ABCD
DISCUSSION: It is not necessarily true that enteral nutrition is safer than
parenteral nutrition, and it may in fact be associated with a higher risk of
death than parenteral nutrition. Specifically, a well-run parenteral nutrition
service should not be associated with significant mortality, except for the
occasional death due to undetected yeast infection. On the other hand, enteral

nutrition, especially if not carried out safely, can result in significant


mortality. The most common of the severe complications of enteral nutrition
result from the gastrostomy, or tube feedings into the stomach. Sudden
changes in gastric motility, such as those associated with sepsis, may result in
aspiration. Nasoenteric or nasoduodenal tubes help prevent this complication,
as does shutting off enteral feedings between the hours of 11 P.M. and 7 A.M.
It is also essential to keep the patient's head elevated 30 degrees. Also
necessary is the use of extreme care when initiating enteral nutrition. If
hypertonic material is given into the stomach, one can increase osmolality
followed by an increase in volume. If, however, the material is given into the
small bowel, volume must be increased first and then tonicity, with the
expectation that osmolality greater than 400 or 500 mOsm per liter may never
be achieved without provoking severe diarrhea. If care is not taken with the
initiation of enteral nutrition, massive diarrhea may result, including fluid
loss, the absorption of enormous amounts of carbohydrate into the circulation
with inadequate fluid to support it, and the development of hyperosmolar,
nonketotic coma. Alternatively, severe unremitting diarrhea may result in
necrosis of the intestinal wall, the appearance of pneumatosis cystoides
intestinalis, and, finally, perforation and death. All of these complications
may be prevented by judicious use of enteral nutrition with the same care one
uses for parenteral nutrition.

48. It has been suggested that enterocyte-specific fuels be utilized for all
patients receiving parenteral nutrition. Theoretically, the benefits of such
fuels include:
A. Glutamine increases gut mucosal protein content and wall thickness.
B. Butyrate increases jejunal mucosal protein content and wall thickness.
C. The short-chain fatty acidsbutyrate, propionate, and acetateare useful
in supporting ileal mucosal protein content and thickness.
D. The use of glutamine-enriched solutions for parenteral nutrition for
patients with chemotherapy toxicity or radiation enteritis is without hazards.

Answer: NONE IS ENTIRELY TRUE


DISCUSSION: The use of enterocyte-specific fuels is part of a new and
potentially exciting phase of nutritional pharmacology in parenteral
nutrition; however, exciting as the research may be, the use of such fuels is
by no means acceptable for indiscriminate use at present. Though some
studies have shown that the provision of glutamine in amounts up to 2% in
standard parenteral nutrition solutions increases both jejunal and ileal
mucosal protein content, cell wall thickness, and DNA content, this has not
been the case in all studies, and this reported effect seems very dependent on
experimental design. In many of the studies that have shown such an effect,
2% glutamine has been used to replace virtually all nonessential amino acids,
probably initiating a deficiency state. The beneficial effects seen with
glutamine are far less impressive than those seen with epidermal growth
factor, for example, and disappear entirely when a different experimental
design is used in which 2% glutamine is added to an adequate amino acid
formulation in which glutamine does not replace nonessential amino acids
but is added to them. Nonetheless, the use of enterocyte-specific fuels,
specifically glutamine, is potentially exciting and should be carefully
investigated. More striking are the results that follow massive bowel
resection, radiation enteritis, and chemotherapy toxicity. Glutamine may help
the small bowel regenerate more quickly, enabling more rapid use of the
small bowel for nutrition. It should be pointed out, however, that glutamine is
a fuel utilized by many tumors and, thus, one runs the risk of stimulating the
growth of the tumor with excessive glutamine. The short-chain fatty acids,
produced from bacterial fermentation of soluble pectin, may be useful in both
the maintenance of colonocyte-specific nutrition and, in the case of butyrate,
ileal enterocyte nutrition.

49. Essential amino acids have been advocated as standard therapy for renal
failure. Which of the following statements are true?

A. Increased survival from acute renal failure has been reported with both
essential and nonessential amino acid therapy of patients in renal failure.
B. Essential amino acids retard the rise of blood urea nitrogen (BUN)
secondary to decreased urea appearance.
C. Essential amino acids and hypertonic dextrose are a convenient form of
therapy for hyperkalemia.
D. Essential amino acids decrease BUN and creatinine to the same degree as
solutions containing excessive nonessential amino acids.
Answer: BC
DISCUSSION: Essential amino acids and hypertonic dextrose, as opposed to
hypertonic dextrose alone, was reported by Abel and co-workers to be
associated with a decreased mortality rate in mostly surgical patients with
acute tubular necrosis. The most significant improvement in mortality, as
compared with the control group receiving hypertonic dextrose, was among
patients who required dialysis (i.e., the more severely affected patients).
Another group responding favorably to treatment includes patients with
nonoliguric renal failure whose need for dialysis is not clearly established.
The effect of essential amino acids in preventing a rise in BUN, as well as its
beneficial effect in preventing hyperkalemia, may obviate dialysis in such
patients. With increasing amounts of nonessential amino acids, BUN
increases, and thus, dialysis is required. Prospective randomized studies
comparing the use of essential versus nonessential amino acids in patients
with acute renal failure have not been carried out in sufficient numbers to
yield answers to this question.

50. A modified amino acid solution with increased equimolar branched-chain


amino acids and decreased aromatic amino acids has been proposed for
patients with hepatic insufficiency. Which of the following statements is/are
true?
A. This formulation is proposed for the use of patients with fulminant

hepatitis.
B. Nitrogen balance is achieved in such patients with amounts of 40 gm. of
amino acids per 24 hours.
C. The use of 80 to 100 gm. of such solutions is associated with hepatic
encephalopathy.
D. In some studies of surgical patients, improvements in mortality have been
reported.
Answer: D
DISCUSSION: The use of modified amino acid solutions is based on the false
neurotransmitter hypothesis of the cause of hepatic coma. According to this
hypothesis, the imbalance between aromatic and branched-chain amino acids in
plasma results in abnormally high levels of the toxic aromatic amino acids in the

modified amino acid


mixtures, with glucose as the calorie base, has been associated in a number of
studies with improvement in encephalopathy. Meta-analysis has concluded
that the use of such solutions is indicated as therapy for hepatic
encephalopathy but has been proposed only for hepatic encephalopathy
complicating acute exacerbation of chronic liver disease. Although there are a
few anecdotal reports of beneficial effects on hepatic encephalopathy of acute
fulminant hepatitis, the use of such a solution has not been advocated, but
such a modified solution is tolerated better than standard amino acid mixtures
in patients requiring TPN. In some studies, particularly in complicated
surgical cases, the use of a highbranched-chain, lowaromatic amino acid
solution has been associated with lower mortality. These statements are true
only for studies in which the modified solutions are given with hypertonic
glucose as a calorie base. Studies in which lipid was the principal calorie
brain, thus provoking hepatic encephalopathy. The use of

source have not revealed such improvements in mortality. In recent studies, giving
an aromatic amino aciddeficient, branched-chain amino acidenriched solution to
patients about to undergo resection of the liver has proved particularly efficacious in
a group of patients with cirrhosis, decreasing morbidity and showing a trend toward
decreased mortality.

51. In the nutritional support of patients with cancer, which of the following
statements is/are true?
A. Nutritional support benefits the patient's lean body mass but does not
enable the tumor to grow.
B. In experimental animals, the growth of implanted tumors is directly
proportional to the amount of calories and protein supplied.
C. Prospective randomized trials of nutritional support utilizing
chemotherapy and radiation therapy have revealed benefits to patients
receiving total parenteral nutrition.
D. Studies of nutritional support for patients with cancer about to undergo
surgery revealed decreased morbidity and mortality, especially morbidity
from sepsis.
Answer: B
DISCUSSION: The problem with the patient with cancer is a very vexing
one. Clearly, one of the metabolic effects of cancer, cachexia, affects patients
in the last quartile of their disease and makes such patients intolerant of
chemotherapy, radiation therapy, and, in many cases, operative procedures.
Total parenteral nutrition (TPN) has been proposed as a means of reversing
cachexia and enabling patients to better tolerate surgery, chemotherapy, and
radiation therapy. In experimental animals, it is clear that the provision of
calories and protein, especially in excessive amounts, is associated with the
more rapid growth of tumors and decreased survival, especially in the group
that is overfed in the extreme. There is also evidence suggesting that
overfeeding, or at least TPN, may result in increased growth (or at least
change cell kinetics) in patients who are overnourished with TPN. Of the
randomized prospective trials that have been carried out, no trial utilizing
chemotherapy or radiation therapy has revealed a survival advantage for
patients receiving TPN. Indeed, in Shamberger's study, there is a suggestion
that the tumor-free interval following treatment of lymphoma may be shorter
in patients receiving TPN. In patients undergoing surgery, however,
especially those who are severely malnourished (as recently revealed in the

VA study) or in patients with major procedures such as esophagogastrectomy


(as in Muller's study), evidence suggests that TPN is beneficial. In a late
follow-up in Muller's study, there was no apparent increase in recurrence, and
the survival rate was the same, despite much higher mortality in the non-TPN
group. This suggests that any improved survival following operation may
have been offset by an increased late recurrence rate, although it is difficult to
reach this conclusion. In summary, for patients with cancer TPN probably
nourishes the tumor as well as the host. Nonetheless, in severely
malnourished patients provision of TPN from 5 to 10 days preoperatively
may increase survival and decrease morbidity. Overfeeding must be avoided.
Future studies will undoubtedly reveal that there are certain nutrients that
tumors require, which probably should be best avoided.

52. Glucose overload results in increased CO 2 production. Which of the


following statements are true?
A. In patients with respiratory insufficiency, administration of glucose as a
principal calorie source is contraindicated.
B. In patients with pulmonary infection and sepsis, calorie support should
consist of 95% fat and 5% glucose.
C. In Askanazi's study, increased CO 2 production and difficulty in weaning
was associated only with pronounced overfeeding.
D. CO 2 production should be measured in most patients who are supported
by respirators in intensive care units and are receiving nutritional support.
Answer: C
DISCUSSION: Few papers have excited as much interest as that by
Askanazi, Kinney, and co-workers, which demonstrated that glucose calories
given to patients with severe respiratory impairment may result in difficulty
in weaning from a respirator. Subsequent research has suggested, however,
that this occurs only with severe overfeeding, when the respiratory quotient is
greater than 1 and calories are excessive. If one examines the conditions

under which Askanazi's patients were studied, these were a group of septic,
depleted patients who were taken from almost no nutritional support to a
caloric supply of 2.25 times their caloric requirement, most of the calories
consisting of glucose. Suffice it to say that, in patients with impaired
respiratory function, one should measure VCO2 and, when VCO2 is
significantly elevated and appears to interfere with weaning, decrease the
amount of glucose calories and increase the amount of fat. If one measures or
estimates calorie requirements and does not overfeed, lipid can be utilized for
25% of the caloric requirement and glucose for the remainder, without much
fear of excessive CO 2 production.

53. Hepatic abnormalities have been noted in adults since the beginning of
hyperalimentation. Which of the following statements are true?
A. Hepatic steatosis appears to be associated with an overload of glucose.
B. Hepatic steatosis is usually associated with abnormalities in hepatic
enzymes.
C. Hyperbilirubinemia is inevitably associated with hepatic steatosis.
D. Abnormalities in the portal insulin-glucagon ratio are thought to be
causative of hepatic steatosis in experimental animals.
Answer: AD
DISCUSSION: The most common metabolic complication of TPN in adults
is hepatic steatosis. Unlike the hepatic abnormalities in children, which may
progress to cholestasis, liver damage, and in some cases death, hepatic
steatosis, or fatty infiltration of the liver with triglycerides, appears to be a
rather benign complication. It may be, but is not necessarily, associated with
hepatic enzymatic abnormalities, which usually occur in the first week, peak
at the third week, and generally disappear by the sixth week of parenteral
nutrition. Abnormalities in the transaminases are most common, with alkaline
phosphatase also being elevated, but there is no correlation between the
degree of fatty infiltration and enzymatic abnormalities. Fatty infiltration

appears to be largely vacuolization with increased storage of triglycerides.


Hepatic steatosis is almost always associated with an overload of glucose.
Recent studies in experimental animals have suggested that the portal insulinglucagon ratio, which is elevated under these circumstances, may be causally
related to hepatic steatosis. Insulin is the leading storage enzyme and is
responsible for lipogenesis. The presence of insulin inhibits lipolysis.
Glucagon, on the other hand, results in the mobilization of hepatic lipid. The
liver sees the portal vein insulin-glucagon ratio. Excesses of insulin elicited
by hypertonic dextrose increase lipid deposition in the liver, whereas
glucagon, which is elicited by certain amino acids, results in the mobilization
of hepatic lipid.

54. Which of the following statements about the presence of gallstones in


diabetes patients is/are correct?
A. Gallstones occur with the same frequency in diabetes patients as in the
healthy population.
B. The presence of gallstones, regardless of the presence of symptoms, is an
indication for cholecystectomy in a diabetes patient.
C. Diabetes patients with gallstones and chronic biliary pain should be
managed nonoperatively with chemical dissolution and/or lithotripsy because
of severe complicating medical conditions and a high operative risk.
D. The presence of diabetes and gallstones places the patient at high risk for
pancreatic cancer.
E. Diabetes patients with symptomatic gallstones should have prompt
elective cholecystectomy, to avoid the complications of acute cholecystitis
and gallbladder necrosis.
Answer: E
DISCUSSION: Gallstones have been found to be very prevalent in patients
with type II (noninsulin-dependent) diabetes mellitus, perhaps related to the
dyslipoproteinemia in such patients. Although the complications of acute

cholecystitis (infection, sepsis, gangrene of the gallbladder) are more


common in diabetics, a decision-analysis study has shown that prophylactic
cholecystectomy cannot be justified since the risk of morbidity and/or
mortality from the cholecystectomy procedure is as great as that of
complications or death from acute cholecystitis. Patients who become
symptomatic should be promptly prepared and should undergo elective
cholecystectomy, because an emergency operation in these patients with
comorbid conditions, especially coronary artery disease, has substantial
added mortality associated with it. There is no causal relationship between
diabetes and pancreatic cancer.

55. Intensive insulin therapy:


A. Prevents the aggressive development of atherosclerosis in diabetic
patients.
B. Is not associated with unawareness of hypoglycemia.
C. Improves peripheral neuropathy.
D. Improves established retinopathy and nephropathy.
E. Is indicated in all patients with noninsulin-dependent diabetes mellitus
(NIDDM).
Answer: C
DISCUSSION: Intensive insulin therapy is indicated in patients with IDDM
who can actively participate in their own management and the attainment of
the goals set for their blood glucose and glycosylated hemoglobin (HgA1 c)
levels. Because the main complication of intensive therapy is iatrogenic
hypoglycemia, this mode of treatment is not indicated for patients with
NIDDM, who often have coexisting medical conditions such as coronary
artery disease and who tolerate hypoglycemia poorly. There is little or no
evidence that macrovascular disease is affected by intensive insulin therapy,
and the added weight gain and hyperinsulinemia associated with the therapy
may worsen atherosclerosis. Unawareness of hypoglycemia is directly related

to a recent hypoglycemia episode, so patients treated intensively are often


unaware of the problem. Intensive therapy does not improve established
retinopathy or nephropathy but slows or prevents progression of these
complications; however, better glucose control may improve peripheral
neuropathy.

56. Which of the following statements about hypertension in diabetes patients


is/are correct?
A. Hypertension worsens the macrovascular disease of diabetes patients.
B. Hypertension accelerates the progression of diabetic nephropathy.
C. Hypertension is associated with sodium retention in diabetes patients.
D. Angiotensin-converting enzyme (ACE) inhibitors should be used in all
patients with chronic hyperglycemia, regardless of the presence of
hypertension.
E. Diuretics, as single-drug therapy, are not indicated in the treatment of
hypertension in diabetes patients.
Answer: ABCDE
DISCUSSION: All of the answers listed are correct. By damaging endothelial
cells, hypertension worsens macrovascular disease in all patients but
especially in diabetics. Hypertension dramatically accelerates the onset and
progression of diabetic renal disease and proteinuria, and this phenomenon
can be slowed or prevented by a combination of treatment modalities,
including ACE inhibitors, which dilate efferent glomerular vessels and lower
intraglomerular pressure. Despite sodium retention in diabetes patients,
single-drug therapy with a diuretic is not indicated because the chronic state
of dehydration in such patients may become worse.

57. What is the major determinant in an individual patient's risk for


perioperative complications?

A. The surgical procedure.


B. The length of the surgical procedure.
C. The anesthetic technique (e.g., general, regional).
D. The length of anesthesia.
E. All of the above.
Answer: A
DISCUSSION: The planned surgical procedure is the major determining
factor in assessing an individual patient's risk for perioperative complications
and in deciding which anesthetic technique will be most appropriate. Good
communication between the surgeon and the anesthesiologist is vital, as the
surgeon knows better than anyone else the extent of the operation and the
length of time it will require.

58. Which of the following are considered routine intraoperative monitors?


A. Temperature probe.
B. Electrocardiogram.
C. Capnograph.
D. Blood pressure cuff.
E. Foley catheter.
Answer: ABD
DISCUSSION: The American Society of Anesthesiologists requires that the
patient's ventilation, circulation, oxygenation, and temperature be continually
monitored during all anesthetics. Routine monitors are considered to be a
temperature probe, electrocardiogram, pulse oximetry, and blood pressure
cuff.

59. Muscle relaxants can be used for which of the following?


A. To facilitate intubation.

B. To provide optimal surgical conditions.


C. To optimize ventilator support.
D. To provide sedation.
Answer: ABC
DISCUSSION: Muscle relaxants are administered to facilitate endotracheal
intubation, to provide the surgeon with optimal working conditions during
anesthesia, and to optimize mechanical ventilator support in some patients.
They do not produce analgesia, sedation, or amnesia. Therefore, muscle
paralysis should not be performed without sedation or general anesthesia.

60. Local anesthetics:


A. Inhibit transmission of nerve impulses by increasing sodium membrane
permeability and the displacement of ionized calcium.
B. Are classified as amides or esters.
C. Produce peripheral vasodilation.
D. Are weak acids.
Answer: BC
DISCUSSION: Local anesthetics act within the nerve membrane, where they
inhibit transmission of nerve impulses by reducing sodium membrane
permeability and the displacement of ionized calcium. All local anesthetics
consist of a hydrophilic region and a hydrophobic region separated by an
alkyl chain. The bond of the alkyl chain is either an ester or an amide, and
these drugs are classified based on this bond. All local anesthetics except
cocaine produce vasodilatation and are weak bases.

61. Absolute indications for a double-lumen endotracheal tube during


thoracic surgery are:
A. Massive hemorrhage from one lung.

B. Unilateral lung infection.


C. Facilitation of surgical exposure.
D. Unilateral bronchopulmonary lavage.
E. All of the above.
Answer: ABD
DISCUSSION: The absolute indications for a double-lumen tube are for the
purposes of protecting one lung from the other. These indications include
ventilation with a bronchopleural fistula, massive hemorrhage from one lung,
pulmonary air cyst resection, unilateral lung infection, and unilateral
bronchopulmonary lavage. Relative indications include facilitation of surgical
exposure, for pneumonectomy, upper lobectomy, and thoracic aneurysm
repair.

62. Determinants of cerebral blood flow include:


A. Preoperative neurologic dysfunction.
B. Arterial CO 2 tension.
C. Arterial O 2 tension.
D. Systemic arterial pressure.
E. All of the above.
Answer: BCD
DISCUSSION: Determinants of cerebral blood flow include arterial CO 2
and O 2 tensions, systemic arterial pressure, and temperature. Other factors
that may affect cerebral blood flow and intracranial pressure are head
position, jugular venous obstruction, and positive end-expiratory pressure.

63. Discharge criteria following ambulatory surgery include:


A. Ability to eat solid food.
B. Stable vital signs.

C. Ability to ambulate.
D. Ability to have protective airway reflexes.
Answer: BCD
DISCUSSION: Discharge criteria following ambulatory surgery include the
patient's being fully awake and oriented, the ability to have protective airway
reflexes, stable vital signs, adequate hydration with the ability to hold down
oral intake, the ability to ambulate, and adequate pain control. All patients
must have a competent person with them to transport themand ideally to
stay with them on the first postoperative night.

64. Advantages of patient-controlled analgesia (PCA) include:


A. Immediate medication delivery.
B. Less contact with nursing staff.
C. Rapid onset of analgesia.
D. Patient control over pain medication.
E. All of the above.
Answer: ACD
DISCUSSION: Advantages of PCA are immediate medication delivery, rapid
onset of analgesia, and patient control over pain medication. Disadvantages
of PCA are less contact with nursing staff and patients' fears that they could
inadvertently administer an overdose or possibly become addicted to the
opioid.

65. Advantages of epidural analgesia include:


A. Earlier mobilization after surgery.
B. Earlier return of bowel function.
C. Shorter hospitalizations.
D. Decreased stress response to surgery.

E. All of the above.


Answer: E
DISCUSSION: Epidural analgesia include excellent pain relief, decreased
sedation with more rapid recovery to presurgical levels of consciousness,
earlier mobilization after surgery with increased ability to co-operate with
respiratory therapy and physical therapy. Following vascular surgery epidural
analgesia may also improve graft flow through mild sympathetic blockade.
Earlier return of bowel function, decreased stress response, shorter
hospitalizations, and decreased morbidity have all been associated with
epidural analgesia.

66. Ketorolac:
A. Is a nonsteroidal anti-inflammatory drug (NSAID) approved for
intravenous, intramuscular, and oral administration.
B. Can be used indefinitely for postoperative analgesia.
C. Can cause renal dysfunction.
D. May decrease surgical blood loss.
Answer: AC
DISCUSSION: Ketorolac tromethamine, an NSAID, is approved by the FDA
for intravenous, intramuscular, and oral administration. The agent is an
effective analgesic with minimal side effects; however, ketorolac, like all
NSAIDs, can enhance surgical bleeding and cause renal and platelet
dysfunction. Additionally, it is recommended that ketorolac should not be
used for more than 5 consecutive days.

67. Factors that decrease collagen synthesis include all of the following
except:
A. Protein depletion.

B. Infection.
C. Anemia.
D. Advanced age.
E. Hypoxia.
Answer: C
DISCUSSION: Collagen synthesis, an integral part of wound healing, is
affected by many local and systemic factors. Protein depletion impairs
fibroplasia. Hypoproteinemia leads to diminution of fibroblast proliferation,
proteoglycan and collagen synthesis, angiogenesis, and wound remodeling.
Although anemia was once believed to be a significant cause of wound
disruption, studies have shown that, in the absence of malnutrition or
hypovolemia, anemia with a hematocrit greater than 15% does not interfere
with wound healing. In contrast, molecular oxygen is critical for collagen
synthesis because it is one of the factors required for the hydroxylation of
lysine and proline. Also, hypoxia favors wound infection. The role of age in
collagen synthesis is not clear, but the incidence of wound failure and
incisional hernias is greater in patients older than 60. Fibroplasia occurs at a
slower rate in older animals. Perhaps more than any other factor, wound
infection is associated with the risk of wound failure.

68. Wound contraction and ultimate contracture may be controlled by which


of the following drugs?
A. Colchicine.
B. D-Penicillamine.
C. Thiphenamil (Trocinate).
D. Glucocorticoids.
E. Ibuprofen (Motrin).
Answer: AC
DISCUSSION: Wound contraction is carried out by highly specialized cells

called myofibroblasts, which, as their name implies, have histologic


characteristics of fibroblasts and smooth muscle cells. The activity of these
cells, and therefore wound contraction, can be influenced by topical
application of smooth muscle inhibitors such as thiphenamil. Inhibitors of
microtubule formation in myofibroblasts, such as colchicine and vinblastine,
also inhibit wound contraction under experimental conditions.
Glucocorticoids and NSAIDs do not affect the wound contraction process.

69. Which of the following is/are true of the actions of transforming growth
factor beta (TGF-b) during wound repair?
A. Increased matrix and proteoglycan synthesis.
B. Inhibition of proteases.
C. Stimulation of plasminogen inhibitor.
D. Chemotaxis for fibroblasts and macrophages.
E. Autoinduction of TGF-b.
Answer: ABDE
DISCUSSION: Through autocrine and paracrine mechanisms TGF-b
stimulates the deposition of collagen and other matrix components by
fibroblasts, inhibits proteases, blocks plasminogen inhibitor, enhances
angiogenesis, and is chemotactic for fibroblasts, monocytes, and
macrophages. TGF-b modulates the expression of cell-surface integrins in a
manner that enhances cell-matrix interaction and matrix assembly. TGF-b
also induces cell production by cells, thus amplifying its biologic effects. The
sustained production of TGF-b at the wound site leads to tissue fibrosis.

70. In contrast to adult wound healing with scar formation, which of the
following are characteristic of scarless fetal skin repair?
A. Matrix rich in hyaluronic acid.
B. Increased inflammatory response.

C. Increased production of TGF-b.


D. No collagen deposition.
E. Minimal angiogenesis.
Answer: AE
DISCUSSION: The ability of a fetus to heal without scar formation depends
on its gestational age at the time of injury and the size of the wound defect. In
general, linear incisions heal without scar until late in gestation, whereas
excisional wounds heal with scar at an earlier gestational age. The profiles of
fetal proteoglycans, collagens, and growth factors are different from those in
adult wounds. The less differentiated state of fetal skin is probably an
important characteristic responsible for scarless repair. There is minimal
inflammation and angiogenesis in fetal wounds. Fetal wounds are
characterized by high levels of hyaluronic acid and its stimulator(s) with
more rapid, highly organized collagen deposition. The roles of peptide
growth factors such as TGF-b and basic fibroblast growth factor are less
prominent in fetal than in adult wound healing. An understanding of scarless
tissue repair has possible clinical applications in the modulation of adult
fibrotic diseases and abnormal scar-forming conditions.

71. Which of the following cell types are not crucial for healing a clean,
incisional wound?
A. Macrophage.
B. Platelet.
C. Fibroblast.
D. Polymorphonuclear leukocyte.
E. Myofibroblast.
Answer: DE
DISCUSSION: Experimental studies have shown that healing may progress
normally in the absence of polymorphonuclear leukocytes in an uninfected

wound. In contrast, depletion of monocytes and macrophages causes a severe


alteration in wound healing with poor dbridement, delayed fibroblast
proliferation, and inadequate angiogenesis. Platelets carry a cadre of
biologically active substances that are important for wound repair, including
peptide growth factors like platelet-derived growth factor (PDGF) and TGFb. Fibroblasts are the principal cell for matrix synthesis and deposition.
Myofibroblasts are important for wound contraction in open defects but have
little if any role in clean, incisional wounds.

72. Which of the following is/are not a substrate or cofactor for prolyl
hydroxylase?
A. Alpha-ketoglutarate.
B. Ascorbate.
C. Biotin.
D. Oxygen.
E. Copper.
Answer: CE
DISCUSSION: Prolyl hydroxylase is one of the rate-limiting enzymes in
collagen synthesis. Substrates and cofactors such as iron, alpha-ketoglutarate,
ascorbate, and oxygen are important participants in this process. If
insufficient prolines are hydroxylated, then the alpha-peptide collagen chains
cannot assume a stable triple helix, the collagen cannot be exported from the
fibroblasts, and the incomplete, unassociated alpha chains are broken down.
Thus, ascorbate deficiency (scurvy) and hypoxia have similar effects on
collagen synthesis.

73. Which of the following is an adhesion glycoprotein?


A. Fibronectin.
B. Tenascin.

C. Laminin.
D. Hyaluronic acid.
E. Collagen type IV.
Answer: ABC
DISCUSSION: Cell adhesion glycoproteins such as fibronectin, vitronectin,
laminin, and tenascin provide a railroad track to facilitate epithelial and
mesenchymal cell migration over the wound matrix. Hyaluronic acid is a
glycosaminoglycan, and collagen type IV is a protein that is a crucial
component of basement membrane.

74. Which of the following is/are true concerning wound fibroblasts?


A. Fibroblasts synthesize and secrete collagen molecules.
B. Wound fibroblasts are derived from blood-borne precursor cells.
C. Fibroblasts migrate to a wound along fibrin strands, which are used as a
scaffold.
D. Large amounts of fibrin or blood clot can act as a physical barrier to
fibroblast penetration, which delays normal wound healing.
Answer: ACD
DISCUSSION: Fibroblasts appear in the wound on about the third day of
healing and begin to synthesize and secrete collagen molecules. Wound
fibroblasts arrive from cells surrounding the wound (e.g., the adventitia of
blood vessels), change their phenotype(s), and become mobile during the
process of replication. Fibroblasts migrate into a wound using the provisional
fibronectin and fibrin matrix as a scaffold. Fibroblasts do not have
fibrinolytic enzymes, and large amounts of fibrin and blood clot prevent
fibroblasts from entering the wound.

75. Which of the following is/are true?

A. Because of its thickness, the tensile strength of a healing wound on the


eyelid is much less than one on the thick skin of the back.
B. By 2 days, the experimental burst strength of skin is minimal since
collagen has been formed in the wound but has not yet cross-linked.
C. Wound strength reaches a plateau by 3 weeks.
D. Wounds rarely, if ever, regain the strength of intact tissues.
Answer: BD
DISCUSSION: Tensile strength measures load per cross-section area at
rupture, whereas burst strength measures load required to break a wound,
regardless of dimension. Therefore, skin wounds have comparable tensile
strength, regardless of thickness. Collagen appears in the wound by 3 to 4
days. Minimal wound strength on day 2 is due to fibrin polymerization and
adhesion of globular proteins. Wounds rapidly gain strength for about 4
months and then continue to gain strength at a slower rate for more than a
year. Wounds do not regain the strength of normal tissue.

76. Which of the following interfere with normal collagen formation or crosslinking?
A. Beta-aminopropionitrile.
B. Iron chelators.
C. Vitamin C depletion.
D. Proline analogs (e.g., cis-hydroxyproline).
E. D-Penicillamine.
Answer: ABCDE
DISCUSSION: Intramolecular and intermolecular cross-links are crucial for
collagen structural stability. Formation of cross-links can be inhibited by two
pharmacologic agents: beta-aminopropionitrile inhibits the enzyme lysyl
oxidase, and D-penicillamine binds to collagen substrate directly to prevent
collagen cross-link formation. Iron is a cofactor for prolyl hydroxylase,

which is important for collagen synthesis. In high enough concentration,


proline analogs prevent collagen formation with minimal effects on
noncollagenase protein synthesis.

77 Which of the following statement(s) is/are true concerning the cell plasma
membrane?
a. The plasma membrane is composed of amphipathic molecules
b. The hydrophobic core of the lipid bilayer of the cell membrane contains
specialized transport proteins which maintain the intracellular ionic milieu
different from the extracellular fluid
c. Plasma membrane proteins extend externally and bear phospholipid
moieties which contribute to the cell coat
d. The membrane proteins of nerve cells are highly voltage-dependent
Answer: a, b, d
The plasma membrane defines the boundary of the cell and serves to contain
and concentrate enzymes and other macromolecule constituents. The plasma
membrane is composed of amphipathic molecules, mainly phospholipids and
proteins that contain distinct regions that are either insoluble in water
(hydrophobic) or soluble in water (hydrophilic). The plasma membrane forms
a continuous barrier between the aqueous extracellular and intracellular
fluids. Transport proteins in the membrane act as regulated channels or
transporters to maintain the intracellular ionic milieu that is clearly different
from the extracellular milieu. In some cells, membrane proteins are
diversified such as in nerve cells where the ion channels are highly voltagedependent, providing the basis for information transmission in the form of
electrical impulses. Most plasma membrane proteins extend externally and
bear carbohydrate moieties primarily as oligosaccharide chains that
contribute to the cell coat or glycocalyx.
78 Which of the following statement(s) is/are true concerning water

movement across cell membranes?


a. Water moves only actively through cell membrane transport proteins
b. For most cells of the body, the transmembrane hydrostatic pressure is 0
c. Water distribution is determined entirely by solute distribution
d. Specialized cells such as the glomerulus of the kidney actively transport
water to maintain hydrostatic pressure
Answer: b, c
The energetics of water transport across cell membranes is simplified by the
fact that water moves only passively due to gradients of hydrostatic pressure
or water concentration. Hydrostatic pressure is an important driving force
only for certain specialized cellsthe capillary endothelium and the
glomerulus of the kidney. For most cells of the body, the transmembrane
hydrostatic pressure is 0 and water moves only in response to water
concentration gradients. Because the concentration of water is determined by
the amount of dissolved solute, the difference in water concentration is
typically expressed as a function of the difference in solute concentration or
osmotic pressure difference. Because there are no specialized, energyconverting transport mechanisms for water, water is distributed at
equilibrium. Water distribution is determined entirely by solute to solute
distribution.

79 The transport of proteins out of the cell is termed exocytosis. Which of the
following statement(s) is/are true concerning this process?
a. Secretory vesicles fuse with the plasma membrane
b. The process can occur in either a constitutive or regulated process
c. A regulated secretion is triggered by a stimulus, most likely a hormone or a
neurotransmitter
d. A decrease in cytoplasmic calcium occurs as part of the secretion process
Answer: a, b, c

Transport vesicles that bud off the Golgi network carry both material to be
secreted from the cell and protein destined to become components of the
plasma membrane. These vesicles can fuse with the plasma membrane in a
process termed exocytosis. Vesicular transport to the cell surface can be
divided into two components, constitutive and regulated secretion. Regulated
secretion occurs in cells secreting digestive enzymes, hormones and other
regulatory molecules, and neurotransmitters. In regulated secretion, the
material to be secreted is sorted in a storage vesicle or granule; fusion with
the plasma membrane in exocytosis then takes place in response to external
stimulation. Regulated secretion is triggered in most cases by a hormone or
neurotransmitter. The ensuing process is termed stimulus-secretion coupling.
In most cases the coupling involves an increase in cytoplasmic concentration
of Ca++, but may also involve generation of diacylglycerol or production of
cyclic AMP which activate kinases or phosphatases.
80 Which of the following statement(s) is/are true concerning the cell
function of phagocytosis?
a. Phagocytosis is a mechanistically distinct process of endocytosis
performed by special cells to take up larger particles such as bacteria or
erythrocytes
b. Lymphocytes are the primary blood cell involved with this process
c. The process involves a coating of the cytoplasmic surface known as
clathrin
d. Phagocytosis is performed only by white blood cells and tissue
macrophages
Answer: a
Phagocytosis is a specialized form of endocytosis by which large particles are
internalized by specialized cells primarily macrophages and neutrophils. To
be phagocytosed, particles must bind to the surface of the phagocytic cell,
usually as the result of specific antibody coating the particle. The phagocytic
cell then extends pseudopods which engulf the particle. This event is

followed by membrane fusion and a pinching off. As opposed to endocytosis,


this process does not involve the membrane protein, clathrin, but rather actin.
A physiologically relevant site of phagocytosis is the thyroid gland, where
thyroid follicular cells phagocytose and digest thyroglobulin from the lumen
of the thyroid follicle, thereby releasing the thyroid hormones, thyroxine
triiodothyronine.
81 A striking feature of living cells is a marked difference between the
composition of the cytosol and the extracellular milieu. Which of the
following statement(s) concerning the mechanisms of maintenance of these
differences is/are true?
a. The cell membrane is able to maintain a 10,000 fold gradient between the
extracellular concentration of ionized calcium and the intracellular
concentration
b. The key to these differences is the fact that the plasma membrane is
normally impermeable to sodium, potassium and calcium
c. The selectivity of biologic membranes is highly consistent and seldom
changes
d. The selectivity of cell membranes relates only to ions and not organic
compounds
Answer: a
The survival of the cell requires that cytosolic composition be maintained
within narrow limits, despite the constant influx of nutrients and the
simultaneous outflow of waste. A familiar example of the distribution of ions
across the cell membrane is that of sodium and potassium. Cells are typically
rich in potassium and contain very little sodium. Despite the fact that they are
constantly bathed by fluid that is precisely the opposite composition. Even
more impressive is the distribution of ionized calcium. The extracellular
concentration of this ion is typically of the order of 103M, whereas that of
cytosol is typically 107M, a 10,000-fold gradient. Such nonequilibrium ion

distributions are even more remarkable in light of the fact that the plasma
membrane is, to varying degrees, leaky to ions such as sodium, potassium
and calcium. The plasma membrane is leaky to a variety of substances, but it
exhibits an astonishing ability to discriminate or select one substance over
another. This selectivity relates to not only ions but also for organic
compounds such as glucose. Finally, the selectivity of biologic membranes
can be altered drastically as a result of regulatory or signaling processes that
occur within the cell.

82 Which of the following statement(s) is/are true concerning DNA?


a. DNA is contained only in the nucleus of the cell
b. DNA strands are encoded by the sequence of four basesadenine,
guanine, cytosine and uridine
c. The basic unit of information of DNA is the intron, a sequence of three
bases
d. There are an infinite number of possible codons
Answer: a
The genetic blueprint of an organism is carried in the nucleus of every cell,
encoded by the sequence of four basesadenine, guanine, cytosine and
thymine, which together make up two long chains bound together by
hydrogen bonds to form a DNA double helix. A gene is a segment of DNA
that is transcribed into a corresponding RNA molecule that either codes for a
protein or forms a structural RNA molecule. Genes are commonly between
10,000 and 100,000 base pairs in length and include, in addition to the coding
sequence, flanking regions and intervening sequences, termed introns. Introns
are removed from the primary RNA transcript by a process called splicing.
The basic unit of information is the codon, a sequence of three bases or
triplet. The four nucleotide bases arranged as triplets lead to 64 possible
codons. Sixty-one of these code for amino acids and three are termination
signals called stop codons.

83 Which of the following statement(s) is/are true concerning cell membrane


receptors?
a. The largest family of cell surface receptors are the G-protein-linked
receptors
b. Activities of the G-protein involve binding and hydrolysis of ATP
c. The G protein receptor generates an intracellular messenger commonly
through the use adenylate cyclase
d. Tyrosine kinase receptors are considered G-protein-linked receptors
Answer: a, c
All water-soluble regulatory molecules bind to the cell surface receptor
proteins. Binding of the appropriate ligand evokes an intracellular signal
which usually regulates enzyme activity, membrane transport, or in some
cases gene expression. Most cell surface receptors belong to one of three
functional classesthese are ion channel receptors, catalytic receptors, and
G-protein-linked receptors. Ion channel receptors are multisubunit assemblies
which, with each subunit, have a multiple membrane spanning segment.
Together these subunits form an ion-selected pore that can be gated by a
change in transmembrane electrical potential or binding of a ligand to one of
the subunits. Catalytic receptors are membrane proteins that possess
enzymatic activity. The best understood receptors of this class are the tyrosine
kinases. The largest family of cell surface receptors are the G-protein-linked
receptors. G-proteins are a family of proteins that bind and hydrolyze GTP.
The final component of single transduction by G-protein-linked cell surface
receptors is the effector that generates the intracellular messenger. The two
best understood effectors are adenylate cyclase, which converts ATP to
cAMP, and the polyphosphoinositide-specific phospholipase C.

84 Which of the following statement(s) is/are true concerning cellular ion


channels?

a. Ion channels are transmembrane proteins that form pores that can conduct
ions across the plasma membrane
b. Ion channels are formed by membrane-spanning peptides that are arranged
so that polar moieties line a central core
c. Ion channel proteins undergo conformational changes between open states
and closed states
d. Ion channels can be blocked
Answer: a, b, c, d
Ion channels are transmembrane proteins that form pores that can conduct
ions across the plasma membrane. Ion channels are formed by membranespanning peptides that are arranged so that polar moieties line a central pore.
These polar groups take the place of the water of hydration, which stabilizes
an ion in an aqueous solution creating, in essence, a water-like environment
into which the ion can partition and move in the presence of the appropriate
driving force. Ion channels are permissive transport elements. Ions flow
through a channel only through the presence of an appropriate driving force.
Ion channels do not conduct all the time, rather the channel protein undergoes
conformational changes between a conducting (open) state and
nonconducting (closed) state. These conformational changes are collectively
referred to as gating. The conduction process can also be blocked by ions or
organic compounds that enter the channel, bind there, and occlude the pore.
85 Which of the following statement(s) is/are true concerning carrier
proteins?
a. Carrier proteins are distinguished by three types of mechanisms: carriertype, channel-type, and conduction-type
b. Conformational changes in the membrane protein occur between the
conducting and the nonconducting states
c. A channel-type carrier protein has two statesclosed and open
d. Carrier-type transport proteins are equally accessible from either side of
the membrane

Answer: b, c
Most transport proteins appear to function as carriers, rather than channels.
Important distinctions can be made between types of carrier proteins on the
basis of transport kinetics. Two primary types can be distinctly identified
based on carrier-type and channel-type mechanisms. The most important
difference between the channel mechanism and the carrier mechanism is the
role in the transport event played by conformational changes in the
membrane protein. The channel is depicted as having two states, closed and
open, so that it operates like a switch. In contrast, carrier transport is
envisioned as requiring a cycle of conformational changes. The transport of
one molecule of substrate requires one complete cycle of the protein. In a
channel mechanism, binding sites within the open pore are equally accessible
from either side of the membrane, whereas in a carrier mechanism, the
binding site is available only one side of the membrane at any instant.
86 Which of the following statement(s) is/are true concerning translation of
the mRNA message to protein synthesis?
a. An adaptor molecule, tRNA, recognizes specific nucleic acid bases and
unites them with specific amino acids
b. Covalent attachment of tRNA to amino acids is energy dependent
c. The formation of a peptide bond between the growing peptide chain and
the free amino acid occurs in the free cytoplasm
d. Complete protein synthesis takes hours
Answer: a, b
The synthesis of protein involves conversion from a four-letter nucleotide
language to one of 20 chemically distinct amino acids. This process is
referred to as translation. There is no mechanism for direct chemical
recognition between specific nucleic acid bases and specific amino acids.
Instead, an adaptor molecule, tRNA, is used. Each tRNA carries only one
amino acid and must be recognized by a distinct enzyme which catalyzes the

covalent attachment of the carboxyl end of the amino acid to the end of the
tRNA in a process using ATP. Protein synthesis occurs by the formation of a
peptide bond between the carboxyl terminal of the growing peptide chain and
the free amino acid of deactivated amino acid tRNA. This event does not
occur in free solution, but within ribosomes. Ribosomes are protein
synthesizing machines that bring all of the necessary components together in
the correct sequence and spacial orientation. Protein synthesis consumes a
great deal of energy because four high-energy phosphate bonds must be split
to make each peptide bond. Complete synthesis of a single protein takes 30
seconds to a few minutes, but multiple ribosomes can initiate translation and
be moving down the mRNA molecules simultaneously, thus increasing the
rate of protein synthesis.
87 Cell regulation can be thought of as the effector side of cell
communication. Most commonly cell regulation occurs by means of
extracellular chemical messengers. Which of the following statement(s) is/are
true concerning these messengers?
a. Paracrine regulation involves a messenger which is produced and acts
systemically
b. The extracellular signal or stimulus is received by a receptor on or in the
target cell
c. Neurocrine regulation depends on a physical connection between the
neuron and the target cell
d. Most hormones, local mediators, and neurotransmitters readily cross the
cell plasma membrane
Answer: b, c
Depending on how the extracellular messenger arrives, cell regulation can be
classified as paracrine, endocrine, or neurocrine. In paracrine regulation, a
chemical messenger or mediator is produced and acts locally. In endocrine
regulation, the extracellular messengers (hormones) are released into the
blood and act on target cells anywhere on the body that has appropriate

receptors. In neurocrine regulation, neurons secrete transmitters into highly


localized regions, the synaptic cleft, so that the regulation depends on a
physical connection between the neuron and the target cell as well as the
presence of a specific receptor. In almost all cases of cell regulation, the
extracellular signal or stimulus is restricted to being an informational
molecule. This information is received by receptor on or in the target cell,
which generally has an affinity for the signal molecule. Most hormones, local
mediators, and neurotransmitters are water-soluble and cannot readily cross
the plasma membrane.
88 Proteins that are destined to be secreted from the cells must pass through a
series of organelles. These organelles include:
a. Endoplasmic reticulum
b. Golgi apparatus
c. Mitochondria
d. Lysosomes
Answer: a, b, d
Proteins targeted for the secretory pathway most commonly begin with
translocation from the cytoplasm across the lipid bilayer into the lumen of the
endoplasmic reticulum. It must then pass through a number of compartments
including the Golgi apparatus where they are further processed and sorted
and end up in a secretory vesicle or lysosome.
89 The best understood intracellular messenger is cyclic AMP (cAMP).
Which of the following statement(s) concerning this intracellular messenger
is/are correct?
a. Intracellular cyclic AMP is constantly degraded by a specific enzyme,
cAMP phosphodiesterase
b. Most of the actions of cAMP are mediated by activation of protein kinase
A
c. Intracellular levels of cAMP are relatively stable and change solely in

response to activation of adenylate cyclase


d. cAMP is the only cyclic nucleotide active as an intracellular messenger
Answer: a, b
The prototypic intracellular messenger is cAMP. To function as a mediator,
the concentration of cAMP must change rapidly. In resting cells, cAMP is
continuously being degraded by a specific enzyme, cAMP phosphodiesterase.
cAMP levels can increase 10-fold or more within seconds of receptor binding
through activation of adenylate cyclase. cAMP acts as an allosteric regulator,
and most, if not all, of its actions are mediated by activation of cAMPdependent protein kinase A. cAMP is not the only cyclic nucleotide active as
an intracellular messenger. Most animal cells also produce cGMP.
Intracellular calcium ions also serve as second messengers in a large number
of cells.
90 The activities of the cytoskeleton is dependent on which of the following
types of filaments?
a. Microtubules
b. Intermediate filaments
c. Actin filaments
d. None of the above
Answer: a, b, c
The cytoskeleton is a collection of filamentous protein structures that allow
cells to assume and maintain a variety of shapes, to produce directed
movement of organelles within the cell, and to affect movement of the entire
cell relative to other cells. These multiple activities depend upon three main
types of filaments: actin filaments, intermediate filaments, and microtubules.
91 Intracellular organelles involved with protein synthesis include:
a. Mitochondria
b. Endoplasmic reticulum

c. Golgi complex
d. Lysosomes
Answer: b, c
Mitochondria are the major source of energy production in eukaryotic cells.
The endoplasmic reticulum is the network of interconnected membranes
forming closed vesicles, tubules, and saccules. The endoplasmic reticulum
has a number of functions and is primarily involved in the synthesis of
proteins and lipids. Adjacent to the rough endoplasmic reticulum and
functionally involved in the sorting and package of secreted protein is the
Golgi complex. Lysosomes are membrane-limited organelles containing acid
hydrolytic enzymes that degrade polymers such as proteins, carbohydrates,
and nucleic acids.

92 An important step in protein synthesis is transcription. Which of the


following statement(s) is/are true concerning this process?
a. The first step in gene transcription involves separating the double helix of
DNA by an enzyme known as DNA polymerase
b. The initial product of DNA transcription is called heterogeneous nuclear
RNA which codes directly for proteins
c. After processing is complete, the mRNA is exported from the nucleus to
the cytoplasm
d. Only one protein can be produced from an initial mRNA strand
Answer: c
Transcription of a gene begins at an initiation site associated with a specific
DNA sequence, termed a promoter region. After binding to DNA, the RNA
polymerase opens up a short region of the double helix to expose the
nucleotides. Once the two strands of DNA are separated, the strand
containing the promoter acts as a template to which ribonucleoside
triphosphates base pair by hydrogen bonds. The initial products of

transcription are known as heterogeneous nuclear RNA because of their large


size variation. These primary transcripts are then processed to form mRNA.
RNA splicing accounts for mature RNA being much shorter than nuclear
RNA. Moreover, alternative splicing can lead to the production of different
mRNA molecules and in some cases different proteins from the same gene.
mRNA is exported from the nucleus only after processing is complete.
93 There are two properties of the cell necessary to maintain nonequilibrium
cellular composition; the first is selectivity and the second is energy
conversion. Which of the following statement(s) is/are true concerning
energy converting transport?
a. The site of energy conversion and transport in the plasma membrane
involves the phospholipid component
b. The Na+-K++-ATPase derives energy from hydrolysis of extracellular ATP
c. In some systems, energy inherent in the transmembrane ion gradient can be
used to drive transport of a second species
d. Examples of species transported via secondary active transport include
hydrogen ions, calcium, amino acids and glucose
Answer: c, d
The selectivity of the plasma membrane, although impressive, cannot account
for the nonequilibrium composition of living cells. A cell can be maintained
in a nonequilibrium state only by continual expenditure of energy. The
maintenance of a steady-state, nonequilibrium cellular composition is
possible because the plasma membrane is the site of energy converters,
membrane proteins that function as biologic transport machines using energy
derived from metabolic processes to perform transport work. The archetype
for the biologic transport machine is the Na+-K+-ATPase, a membrane
protein that hydrolyses cytosolic ATP and couples the resulting free energy to
transport of Na+ and K+. A second equally important type of energyconverting transporter is one in which the energy inherent in a
transmembrane ion gradient, usually that of Na+ can be used to drive the

transport of a second species such as protons, calcium, amino acids, or


glucose.
94 Which of the following statement(s) is/are correct concerning cell
junctions?
a. The major occluding junction is the tight junction or zonula occludens
b. Tight junctions are usually located near the basal pole of the cell
c. Desmosomes are button-like points of attachment which serve to weld
together adjacent cells
d. Gap junctions are a type of cell junction specialized for cell
communication
Answer: a, c, d
Cell junctions are classified as occluding, anchoring, and communicating.
The major occluding junction is the tight junction or zonula occludens which
connects cells in epithelia and thereby allows epithelia to serve as selective
permeability barriers. Tight junctions are normally located near the apical
pool of the cell and form a belt that completely encircles the cell. Anchoring
junctions connect the cytoskeleton of the cell to the extracellular matrix or
neighboring cells. Morphologically these are adherens junctions or
desmosomes. Desmosomes are button-like points of attachment with a
prominent intracellular plaque that serves to weld together adjacent cells by
serving as anchoring sites for intermediate filaments within the cell. The third
functional type of cell junction is a gap junction which is specialized for
communication. This junction mediates both electrical and chemical
coupling.
95 Examples of ion channel blockers include:
a. Tetrodotoxin
b. Amiloride
c. Xylocaine
d. None of the above

Answer: a, b, c
Channel blockade is an important mechanism of action for toxins and some
therapeutic agents. The deadly toxin of the puffer fish, tetrodotoxin, acts by
blocking the Na+ channels that are responsible for the conduction of nerve
impulse. The diuretic, amiloride, acts by blocking the Na+ channels that
inhabit the apical membrane of the epithelial cells of the distal nephron.
Local anesthetics such xylocaine also act by blocking ion channels.
96 Most hormone receptors are localized on the cell membrane and transduce
hormone binding into altered levels of intracellular messengers. A limited
number of intracellular receptors do exist. Which of the following
statement(s) is/are true concerning intracellular receptors?
a. The messengers or hormones must by lipophilic
b. These intracellular receptors generally regulate protein synthesis
c. The intracellular receptors are located entirely in the nucleus of the cell
d. A heat-shock protein serves as an inhibitor protein blocking the DNAbinding domain of the steroid receptor
Answer: a, d
Although most hormone and other messenger receptors are extracellular,
intracellular receptors have been identified. The hormone messengers
involved for these receptors are primarily steroid and thyroid hormones and
are lipophilic. By virtue of their hydrophobic nature, they are able to readily
penetrate the lipid portion of the cell membrane. Receptors for these
hormones exist intracellularly in the cytoplasm or in the nucleus and
generally act as regulators of gene expression. These hydrophobic signaling
molecules exist in plasma bound to protein, so that the concentration of this
class of regulators does not fluctuate rapidly in plasma and their actions are
generally slower in onset and more prolonged than those of water-soluble
class. Some types of steroid receptors, particularly for glucocorticoids, are
located in the cytosol in the inactive state. Once the ligand binds, the receptor

undergoes a conformational change, termed activation. This allows


cytoplasmic receptors to move into the nucleus and bind to DNA. Receptors
already in the nucleus increase their affinity for DNA. In the case of
glucocorticoid receptors and probably others of this class, the inactive
receptor is associated with another protein, the heat-shock protein. They
block the DNA-binding domain of the receptor. Activation involves the
dissociation of the inhibitor protein.
97 Altering the amino acid profile in total parenteral nutrition solutions can
be of benefit in certain conditions. Which of the following conditions are
associated with a benefit by supplementation with the amino acid type listed?
a. Acute renal failure and essential amino acids
b. Hepatic failure and aromatic amino acids
c. Short gut syndrome and glutamine
d. Chronic renal failure and essential amino acids
Answer: a, c
In a number of conditions, altering the amino acid profile of the total
parenteral nutrition solution can be of benefit. TPN with amino acids of high
biologic value may decrease the mortality in patients with acute renal failure.
These solutions, containing high quality amino acids, can improve nitrogen
balance and diminish urea nitrogen. Provision of essential amino acids only
allows the body to maximally utilize nitrogen for the synthesis of nonessential amino acids and thereby helps prevent rapid rises in blood urea
nitrogen. There appears to be no advantages to using essential amino acids if
the patient is already being dialyzed every other day and therefore a balanced
standard amino acid solution is recommended. Because of liver damage and
portasystemic shunting, patients with hepatic failure develop derangements in
circulating levels of amino acids. The plasma aromatic/branch chain amino
acid ratio is increased favoring the transport of aromatic amino acids across
the blood brain barrier. These amino acids are precursors of false transmitters
which contribute to lethargy and encephalopathy. Treatment of individuals

with liver failure with solutions enriched in branch chain amino acids and
deficient in aromatic amino acids results in improved tolerance to
administration of protein and clinical improvement in encephalopathic states.
Glutamine-enriched TPN partially attenuates villous atrophy and may be
useful in treatment of short gut syndrome.
98 Under certain circumstances, the gut may become a source of sepsis and
serve as the motor of systemic inflammatory response syndrome. Microbial
translocation is the process by which microorganisms migrate across the
mucosal barrier to invade the host. Which of the following mechanisms can
promote bacterial translocation?
a. An increased number of gut bacteria
b. Altered intestinal mucosal permeability
c. Decreased host defense mechanisms
d. Lack of enteral feeding
Answer: a, b, c, d
99 Translocation is promoted in three general ways: 1) altered permeability of
the intestinal mucosa as caused by shock, sepsis, distant injury, or cell toxins;
2) decreased host defense (secondary to glucocorticoid administration,
immunosuppression, or protein depletion; and 3) an increased number of
bacteria within the intestine. Because many factors that facilitate bacteria
translocation occur simultaneously in surgical patients, these effects may be
either additive or cumulative. In addition, many patients in Surgical Intensive
Care Units do not generally receive enteral feedings and therefore current
parenteral therapy results in gut atrophy which further promotes
translocation.
Which of the following statement(s) is/are true concerning nutritional support
of the injured patient?
a. The goal of nutritional support is maintenance of body cell mass and
limitation of weight loss to less than 25% of preinjury weight

b. Under-nutrition may compromise the patients available defense


mechanisms
c. Nutritional support is an immediate priority for the trauma patient
d. Fifty percent of non-nitrogen caloric requirements should be provided in
the form of fat
Answer: b
Metabolic response to injury results in increased energy expenditure. If
energy intake is less than expenditure, oxidation of body fat stores and
erosion of lean body mass will occur with resultant loss of weight. When
weight loss exceeds 1015% of body weight, the complications of
malnutrition interact with disease processes, with increased morbidity and
mortality rates. The goal of nutritional support is maintenance of body cell
mass and limitation of weight loss to less than 10% preinjury. The major
impact of nutritional support in the trauma patient is to aid host defense.
Under-nutrition may compromise the available host defense mechanism and
may thus increase the likelihood of invasive sepsis, multiple organ system
failure, and death. Resuscitation, oxygenation and arrest of hemorrhage are
immediate priorities for survival. Nutritional support is an essential part of
the metabolic care of the critically ill patient and should be instituted after
resuscitation before significant weight loss occurs. The nutritional
requirements of a trauma patient can be determined by determining basal
metabolic rate with appropriate increases based on extent of injury and
hospital activity. After initial determination of nitrogen requirements, caloric
requirements should be distributed at a ratio of 70% as glucose and 30% as
fat.
100 Which of the following statement(s) is/are true concerning body fuel
reserves?
a. The largest fuel reserve in the body is skeletal muscle
b. Fat provides about 9 calories/gram
c. Free glucose and glycogen stores are a trivial fuel reserve

d. Body protein is a valuable storage form of energy


Answer: b, c
The body contains fuel reserves which it can mobilize and utilize during
times of starvation or stress. By far the greatest energy component is fat,
which is calorically dense since it provides about 9 calories/gram. Body
protein comprises the next largest mass of utilizable energy, but amino acids
yield only about 4 kcal/gram. Unlike fat reserves, body protein is not a
storage form of energy but rather serves as a structural functional component
of the body; loss of body protein, if severe, is associated with functional
consequences. Glycogen stored in muscle and liver and free glucose have a
trivial caloric value of less than 1000 kcal for a 70 kg male.
101 Which of the following statement(s) is/are true concerning the
indications and administration of nutritional support to cancer patients?
a. Preoperative nutritional support should be provided to all patients with
cancer
b. To be effective, preoperative nutrition must be given for at least two weeks
preoperatively
c. Parenteral nutrition is the preferred route of feeding for all cancer patients
d. Standard total parenteral nutrition solutions maintain integrity of the small
bowel
e. None of the above
Answer: e
The role of nutritional support in the cancer patient remains an important
component of overall therapy. Preoperative nutritional support should be
given only to those patients who do not require an emergency operation and
who have severe weight loss (> 15% of pre-illness body weight) and a serum
albumen < 2.9 mg%. Preoperative nutrition (enteral or parenteral) should not
be given for longer than 7 to 10 days. Enteral nutrition is always the preferred
route of feeding cancer patients if the GI tract is functional. There are several
benefits of using the bowel lumen for nutrient delivery. The trophic effects of

enteral feeding on small bowel mucosa have been well described. The
integrity of the mucosal lining is maintained and it may provide an effective
barrier to intraluminal enteric organisms which might otherwise translocate
into the systemic circulation. Atrophic changes may be seen in the intestinal
epithelium after several days of bowel rest; this atrophy is not reversed by
currently available total parenteral nutrition solutions. 102 Which of the
following hormones can be expected to be released as part of the stress
response? a. Antidiuretic hormone (ADH) b. Aldosterone c. Insulin d.
Epinephrine nswer: a, b, d Several important responses occur in response to
stress. The body immediately attempts to compensate for a reduction in
circulating blood volume in order to maintain adequate organ perfusion.
Afferent nerve signals are also initiated which stimulate the release of both
antidiuretic hormone (ADH) and aldosterone. The pain and fear associated
with the stress response lead to excessive production to catecholamines
which also increase metabolic rate, stimulate lipolysis, hepatic glycolysis,
and gluconeogenesis. Glucagon, which has a potent glycogenolytic and
gluconeogenic effect in the liver, is also released. This hormone has the exact
opposite effect of insulin, which promotes glucose storage and uptake by the
cells. 103 Cytokines which play an important role in the metabolic response
to injury include: a. Tumor necrosis factora (TNF) b. Interleukin-1 (IL-1)
c. Interleukin-6 (IL-6) d. Interferon-g Answer: a, b, c, d TNF or cachetin is
considered the primary mediator of the systemic effects of endotoxin,
producing anorexia, fever, tachypnea, and tachycardia at low doses and
hypotension, organ failure, and death at higher doses. TNF is produced
primarily by macrophages, but lymphocytes, Kupffer cells, and a number of
other cell types have been identified as sources of TNF. IL-1, like TNF, has a variety
of pro-inflammatory activities. IL-6 is now recognized as a primary mediator of
altered hepatic protein synthesis known as acute-phase protein synthetic response.
Glucocorticoid hormones augment the cytokine effects on acute phase protein
synthesis. Interferons are a family of proteins which are readily identified for their
ability to inhibit viral replication in infected sells. IFN-g has the ability to upregulate
the number of TNF receptors on various cell types. 104 A 16-year-old boy suffers a
mid-gut volvulus with massive loss of small intestine. Which of the following
statement(s) is/are true concerning his nutritional requirements and management?
a. If at least 18 inches of residual small intestine survives, the patient may tolerate

some form of enteral nutrition b. A nutritional regimen consisting of supplemental


glutamine, growth hormone, and a modified high carbohydrate, low fat diet may be
beneficial in this patient c. The regimen described above may decrease the cost of
care d. TPN needs will increase after discontinuation of growth hormone Answer: a,
b, c Prior to the availability of TPN, most patients developing short bowel syndrome
from either surgery or catastrophic event died. In selected patients, however, with
residual small intestine (at least 18 inches), post-resectional hyperplasia may
develop with time such that they can tolerate enteral feeds. Recent studies have
demonstrated the requirement for TPN could be decreased or even eliminated in
patients with short-gut syndrome by providing a nutritional regimen consisting of
supplemental glutamine, growth hormone, and a modified high carbohydrate, low
fat diet. There was a marked improvement in absorption of nutrients with this
combin
on of therapy and a decrease in stool output. In addition, TPN requirements were
reduced by 50% as were costs associated with the care of these individuals.
Discontinuation of the growth hormone did not increase TPN needs in patients once
they had undergone successful gut rehabilitation. 105 A number of changes in trace
mineral metabolism are noted during sepsis. Which of the following change(s) may
be observed in a septic or trauma patient? a. Plasma iron levels are noted to
decrease b. Plasma copper levels are noted to decrease c. Plasma serum zinc levels
may decrease d. Administration of iron is appropriate Answer: a, c Changes in the
balance of magnesium, inorganic phosphate, zinc, and potassium generally follow
alterations in nitrogen balance. Although the iron-binding capacity of transferrin is
usually unchanged in early infection, iron disappears from the plasma, especially
during severe pyogenic infection; similar alterations are observed in serum zinc
levels. The administration of iron to the infected host, especially early into the
disease, is contraindicated, however, because increased serum iron concentrations
may impair resistance. Unlike iron and zinc, copper levels generally rise, and the
increased plasma concentrations can be ascribed almost entirely to the levels of the
ceruloplasmin produced by the liver. 106 A 17-year-old patient involved in an
automobile accident is paralyzed with multiple peripheral extremity injuries.
Nutritional support is instituted with a transnasal feeding catheter. Which of the
following statement(s) is/are true concerning the patients management? a. Feeding
into the stomach results in stimulation of the biliary/pancreatic axis which is
probably trophic for small bowel b. Gastric secretions will dilute the feedings
increasing the risk of diarrhea c. The major risk in this patient is tracheobronchial
aspiration d. Placement of the feeding catheter through the pylorus into the first
portion of the duodenum reduces the risk of regurgitation and aspiration Answer: a,
c, d The use of transnasal feeding catheters for intragastric feeding or for duodenal
intubation are popular adjuncts for providing nutritional support by the enteral
route. The stomach is easily accessed by passage of a soft flexible feeding tube.
Intragastric feeding provides several advantages for the patient. The stomach has
the capacity and reservoir for bolus feedings. Feeding into the stomach results in

stimulation of the biliary/pancreatic axis which is probably trophic for the small
bowel. Gastric secretions will have a dilutional effect on the osmolarity of the
feedings, reducing the risk of diarrhea. The major risk of intragastric feeding

is the

regurgitation of gastric contents resulting in aspiration into the


tracheobronchial tree. This risk is highest in patients who have an altered
sensorium or who are paralyzed. The placement of the feeding tube through
the pylorus into the fourth portion of the duodenum reduces the risk of
regurgitation and aspiration of feeding formulas. 107 Although TPN has
major beneficial effects to the patient and specific organ systems, TPN has a
downside which is related to intestinal disuse. Which of the following
statement(s) is/are true concerning the effects of TPN on the GI tract? a.
Patients receiving TPN have an accentuated systemic response to endotoxin
challenge compared to enterally fed volunteers b. TPN can result in
disruption of intestinal microflora c. In experimental models, bacterial
translocation from the gut is increased d. Effects of TPN on the gut may lead
to multiple organ failure nswer: a, b, c, d A number of studies have examined
the effects of TPN on intestinal function and immunity. Although most of
these studies have been done in animal models, TPN has consistently been
shown to have some detrimental effects. In rats, TPN results in significant
disruption of the intestinal microflora and bacterial translocation of the gut to
the mesenteric lymph nodes. In addition, when stresses such as a burn injury,
chemotherapy, or radiation are introduced into these models, animals on TPN
have a much higher mortality. The body of literature suggests that TPN under
certain circumstances may predispose patients to an increase in gut-derived
infectious complications. In a study in human volunteers, individuals
receiving TPN had an accentuated systemic response to endotoxin challenge
compared to enterally fed volunteers. This study is consistent with
impairment of gut barrier function during parenteral feedings which may
promote the release of bacteria and/or cytokines leading to pronounced
systemic responses and possibly multiple organ failure. 108 Total body mass
is composed of an aqueous component and a nonaqueous component. The
nonaqueous component is made up of which of the following? a. Liver b.
Tendons c. Skeletal muscle d. Extracellular fluid e. Adipose tissue Answer: b,

e The nonaqueous portion of total body mass is made up of bones, tendons,


and mineral mass as well as adipose tissue. The aqueous component contains
the body cell mass which is made up of skeletal muscle, intraabdominal and
intrathoracic organs, skin, and circulating blood cells. Also contributing to the
aqueous portion is the interstitial fluid and intravascular volume. 109 Fatty
acids are a major energy source for the body. Which of the following
statement(s) is/are true concerning the use of fatty acids as an energy source?
a. Fatty acids are stored in adipocytes as triglycerides b. Hormone-sensitive
lipase is present only in adipose tissue c. Fatty acids are released into the
circulation traveling freely in plasma d. Approximately 25% of total
nonprotein caloric needs supplied via total parenteral nutrition should be in
the form of fat Answer: a, b, d In most tissues, fatty acids are readily oxidized
for energy. They are especially important energy sources for the heart, liver
and skeletal muscle. In adipose tissue, fatty acids may be re-esterified with
glycerol and stored as triglycerides in adipocytes. Stored fat is mobilized
during starvation and stress. Hormone-sensitive lipase, present only in
adipose tissue, catalyzes the breakdown of stored triglycerides into glycerol
and fatty acids. The fatty acids that are produced are released in the
circulation. The major lipids in plasma do not circulate in a free form, thus
free fatty acids must be bound to albumin. During stress, the activity of
hormone-sensitive lipase is increased which leads to mobilization of fat
stores. However, fat remains an important fuel source for critically ill patients
and as a rule the amount of fat administered to patients receiving total
parenteral nutrition should comprise about 530% of total nonprotein caloric
needs. 110 Which of the following metabolic effects may be observed in
patients with sepsis? a. Increased gluconeogenesis b. Accelerated proteolysis
c. Increased lipolysis d. Impaired gut metabolism of glutamine Answer: a, b,
c, d A number of metabolic responses to sepsis have been defined. Glucose
production is increased in infected patients which appears to be additive to
the augmented gluconeogenesis that occurs following injury. Accelerated
proteolysis, increased nitrogen excretion and prolonged negative nitrogen
balance also occur following infection with a response pattern similar to that

described with injury. Severe infection is often associated with a


hypercatabolic state that initiates marked changes in interorgan glutamine
metabolism. This process results in accelerated muscle proteolysis and net
skeletal muscle glutamine release. The bulk of glutamine is taken up by the
liver at the expense of the gut. It appears that sepsis can impair gut
metabolism of glutamine. Fat is a major fuel oxidized in infected patients,
and increased metabolism of lipids from peripheral fat stores is especially
prominent during a period of inadequate nutritional support. 111 Which of the
following statement(s) is/are true concerning protein/amino acid metabolism
in man? a. The major source of amino acids is breakdown of circulating
proteins b. The recommended daily allowance for protein may triple in
critically ill patients c. Urinary nitrogen losses will approach 0 in the face of
protein starvation d. Negative nitrogen balance refers to a decrease in
nitrogen taken into the body versus the amount of nitrogen lost Answer: b, d
About 15% of the total body weight is made up of proteins, about half of
which are intracellular and half extracellular. In man and other animals,
dietary protein is the source of most amino acids. Intestinal absorption is the
only physiological pathway by which the body obtains exogenous amino
acids. Digestion of ingested protein provides free amino acids that are
absorbed by the small intestine and transported to the liver where they can be
incorporated into new proteins or other biosynthetic products. Excess amino
acids are degraded and their carbon skeleton is oxidized to produce energy or
it is incorporated into glycogen or into free fatty acids. In addition to the
metabolism of dietary amino acids, the existing proteins in the cell are
continuously recycled, such that total protein turnover in the body is about
300 g/day. Vertebrates cannot reutilize nitrogen with 100% efficiency;
therefore, obligatory nitrogen losses occur, mainly in the urine. Urinary
nitrogen losses will diminish when individuals are fed a protein-free diet, but
will never become 0 because of the bodys inability to completely reutilize
nitrogen. In stressed patients, this ability to adapt to starvation is
compromised such that proteolysis of body proteins continues at a substantial
rate. This increases the amount of obligatory nitrogen losses which are

accentuated by the catabolic disease states. This results in a negative nitrogen


balance in which the amount of nitrogen taken in by the patient is exceeded
by the amount of nitrogen lost in the urine, stool, skin, wounds, and fistula
drainage. 112 Which of the following statement(s) concerning intravenous
nutritional support is/are true? a. Concentrations of glucose no higher than
5% should be used to avoid peripheral vein sclerosis b. A major disadvantage
of the peripheral technique is limited caloric delivery c. If total parenteral
nutrition is required, access to the superior vena cava via the external jugular
vein is the most suitable site d. Venous thrombosis is an uncommon
complication for long-term central vein catheterization Answer: b Although
peripheral access can be used for intravenous nutrition, the major
disadvantage of this technique is limited caloric delivery to meet catabolic
demands within tolerated fluid limits. Infusion of glucose (up to 10%), amino
acid solutions, and fat emulsions can be administered peripherally but these
solutions must be nearly isotonic to avoid peripheral vein sclerosis. The
preferred method of access for total parenteral nutrition is into the superior
vena cava by cutaneous cannulation of the subclavian vein. Alternative sites
include the internal and external jugular vein but the catheter exiting from the
neck region makes it more difficult to secure and maintain a sterile dressing.
Complications from long-term central venous catheterization include venous
thrombosis and venous catheter-related infection. Thrombosis of central
vessels is a complication which is often overlooked. The clinical suspicion of
subclavian vein thrombosis is only about 3%, whereas studies that use
phlebography or radionucleotide venography indicate the incidence is as high
as 35%. 113 Sepsis causes a marked metabolic response. Which of the
following statement(s) is/are true concerning the metabolic response to
sepsis? a. Oxygen consumption is increased in the face of infection b. In a
patient with a maximal metabolic rate secondary to trauma, the presence of
infection will increase the rate further c. Metabolic rate increases at a rate of
approximately 10% for each increase of 1C in central temperature d. The
extent of increase in oxygen consumption relates to the severity of the
infection Answer: a, c, d Oxygen consumption is usually elevated in the

infected patient. The extent of this increase is related to the severity of the
infection, with peak elevations reaching 50% to 60% above normal. If the
patients metabolic rate is already elevated to a maximal extent because of
severe injury, no further increase will be observed. In patients with only a
slightly accelerated rate of oxygen consumption, the presence of infection
will cause a rise in metabolic rate added to the preexisting state. A portion of
the increase in metabolism may be ascribed to increase in reaction rate
associated with fever. Calculations suggest that the metabolic rate increases
10% to 13% for each elevation of 1C in central temperature. 114
Interleukin-6 is recognized as the cytokine primarily responsible for the
alteration in hepatic protein synthesis recognized as the acute phase response.
Which of the following statement(s) is/are true concerning acute phase
protein response to surgical stress? a. Glucocorticoid hormones inhibit this
response b. Proteins such as albumin and transferrin which serve in serum
transport are generally increased in this response c. Examples of acute phase
proteins include fibrinogen and C-reactive protein d. In general, the
physiologic role of acute phase proteins are to reduce the systemic effects of
tissue damage Answer: c, d IL-6 is now recognized at the cytokine primarily
responsible for the alteration in hepatic synthesis recognized as the acute
phase response. Glucocorticoid hormones augment this response. The
primary metabolic component of the acute phase response is a qualitative
alteration in hepatic protein synthesis with resulting alteration in plasma
protein composition. Characteristically, proteins which act as serum transport
in binding molecules, (albumin, transferrin) are reduced in quantity and acute
phase proteins (fibrinogen, C-reactive proteins) are increased. Acute phase
proteins are elaborated for the purpose of reducing the systemic effects of
tissue damage. Many act as anti-proteases, opsonins, or coagulation and
wound healing factors that generally inhibit the tissue destruction that is
associated with the local initiation of inflammation. 115 A 59-year-old trauma
patient has suffered multiple septic complications including severe
pneumonia, intraabdominal abscess, and major wound infection. He has now
developed signs of multisystem organ failure. Which of the following

statement(s) is/are true concerning necessary changes to be made in his


nutritional management? a. Carbohydrate load should be reduced in the face
of respiratory failure b. In patients with renal failure, protein intake should be
diminished c. During hemodialysis protein intake should be limited to the
same extent d. In patients with hepatic failure, carbohydrate load should be
increased Answer: a, b The most severe complication of sepsis is multiple
system organ dysfunction syndrome, which may result in death. The
development of organ failure requires changes in the nutritional requirements
and creates special feeding problems. A problem associated with systemic
infection is oxygenation and elimination of carbon dioxide. Most of the
enteral and parenteral formulas used to provide nutritional support for
critically ill patients contain large amounts of carbohydrate, which generate
large amounts of carbon dioxide following oxygenation. Such a large CO2
load may worsen pulmonary function or may delay weaning from the
respirator. If this factor becomes a problem, the carbohydrate load should be
reduced to 50% of metabolic requirements and fat emulsion administered to
provide additional calories. When renal failure becomes progressive, the use
of hemodialysis minimizes the effect of uremia superimposed on the
metabolism of sepsis. Metabolic studies in patients with acute and chronic
renal failure have limited the intake of nonessential amino acids, in an
attempt to lower urea production. Proteins of high biologic value, but in
much smaller quantities than usually given, are administered along with
adequate calories, usually in the form of glucose. When enteral feedings are
not feasible, a central venous infusion of an essential amino acid solution and
hypertonic dextrose provides calories and a small quantity of nitrogen to
reduce protein catabolism while simultaneously controlling the rise in BUN.
During dialysis, protein intake is liberalized, but the BUN should still be
maintained below 100 mg/dl. Hepatic dysfunction is a common manifestation
of septicemia. The carbohydrate load is usually reduced to consist of no more
than 5% of metabolic requirements, and the additional calories should be
provided as fat emulsion. If encephalopathy develops, protein load should
also be reduced0. 116 Which of the following statement(s) is/are true

concerning the role of glutamine in total parenteral nutrition? a. Glutamine is


an essential amino acid b. Glutamine appears to be of primary benefit in
critical illness c. Glutamine is included in most standard TPN solutions d.
Glutamine is the primary energy source for intestinal mucosal cells of the
small bowel and colon Answer: b Glutamine is the most studied gut-specific
nutrient. Glutamine has been classified as a nonessential or nutritionally
dispensable amino acid since glutamine can be synthesized in adequate
quantities from other amino acids and precursors. Glutamine is not included
in most nutritional formulas and has been eliminated from TPN solutions
because of its relative instability and short half life compared to other amino
acids. With few exceptions, glutamine is present in oral enteral diets but only
at relatively low levels characteristic of the concentration in most animal and
plant stores (about 7% of total amino acids). Several recent studies, however,
have demonstrated that glutamine may be an essential amino acid during
critical illness, particularly as it relates to supporting the metabolic
requirements of the intestinal mucosa. These studies demonstrate that dietary
glutamine is not required during states of health but appears to be beneficial
when glutamine depletion is severe and/or when intestinal mucosa is
damaged by insults such as chemotherapy or radiation therapy. The addition
of glutamine to enteral diet reduces the incidence of gut translocation but
these improvements are dependent upon the amount of supplemental
glutamine and the type of insult studied. Glutamine-enriched TPN partially
attenuates villous atrophy that develops during parenteral nutrition. The use
of intravenous glutamine in patients appears to be safe and effective in its
ability to maintain muscle glutamine stores and improve nitrogen balance. In
contrast to glutamine, short chain fatty acids are primary energy source for
colonocytes. 117 Which of the following are determinants of the host
response to surgical stress? a. Gender b. Age c. Nutritional status d. Body
composition Answer: a, b, c, d The pattern of physiologic changes elicited in
response to surgical stress results from the specific interaction of an
individual patient with a stressful stimulus. Several factors specific to the
patient may determine the nature of the host response to stress. Body

composition is a major determinant of the metabolic responses observed


during surgical illness. Post-traumatic nitrogen excretion is directly related to
the size of the body protein mass. A strong relationship between protein
depletion and postoperative complications has been demonstrated in
nonseptic, nonimmunocompromised patients undergoing elective major
gastrointestinal surgery. Protein-depleted patients have significantly lower
preoperative respiratory muscle strength and vital capacity, increased
incidence of postoperative pneumonia, and longer postoperative hospital stay.
Impaired wound healing and respiratory, hepatic, and muscle function in
protein-depleted patients awaiting surgery has also been reported. Many of
the changes in the metabolic responses to surgical illnesses that occur with
aging can be attributed to alterations in body composition and to longstanding patterns of physical activity. Fat mass tends to increase with age and
muscle mass tends to decrease. Loss of strength that accompanies immobility,
starvation and acute surgical illness may have marked functional
consequences. Furthermore, the prevalence of cardiovascular and pulmonary
diseases increase with age. Thus, the delivery of oxygen to tissues may be
impaired in the elderly. Finally, observed differences in metabolic responses
of men and women generally reflect differences in body composition. Lean
body mass is lower in women than in men; and this difference is thought to
account for the net loss of nitrogen after major elective abdominal surgery
generally being lower in women than in men. 118 In contrast to a patient
undergoing an elective operation, which of the following statement(s) is/are
true concerning a patient who has suffered a multiple trauma? a. Basal
metabolic rates are similar b. The patient is highly sensitive to insulin c.
Utilization of the amino acids, glutamine and alanine, is similar to their
composition in skeletal muscle d. Fat and protein stores are rapidly depleted
Answer: b, d The degree of hypermetabolism is generally related to the
severity of injury. Patients with long-bone fractures have a 1525% increase
in metabolic rate, whereas metabolic rates in patients with multiple injuries
increases by 50%. These metabolic rates in trauma patients are contrasted
with those in postoperative patients, who rarely increase their BMR by more

than 1015% following operation. Studies have shown that uninjured


volunteers are able to dispose of exogenous glucose load much more readily
than injured patients. Other studies have demonstrated a failure to suppress
hepatic glucose production in trauma patients during glucose loading or
insulin infusion. Thus, profound insulin resistance occurs in injured patients.
Skeletal muscle is the major source of nitrogen that is lost in the urine
following extensive injury. Although it is recognized that amino acids are
released by muscle in increased quantities following injuries, it has only been
recently appreciated that the composition of amino acid reflux does not
reflect the composition of muscle protein. The release is skewed towards
glutamine and alanine, each of which comprise about one-third of the total
amino acids released by skeletal muscle. To support hypermetabolism, stored
triglyceride is mobilized at an accelerated rate. Although mobilization and
use of free fatty acids are accelerated in injured subjects, if unfed, severely
injured patients rapidly deplete their fat and protein stores. 119 A 47-year-old
patient undergoing a complicated laparotomy for bowel obstruction develops
a postoperative enterocutaneous fistula. Which of the following statement(s)
is/are true concerning parenteral nutritional support in the postoperative
period? a. Oral intake can result in severe dehydration, electrolyte
abnormalities, and perifistula skin injury b. Total parenteral nutrition
increases the spontaneous closure rate of intestinal fistula c. Total parenteral
nutrition decreases mortality rate in patients with intestinal fistulas d. The use
of TPN better prepares the patient for surgery if surgical intervention proves
necessary Answer: a, b, d Patients with gastrointestinal-cutaneous fistulas
represent the classical indication for TPN. In such patients, oral intake of
food almost invariably results in increased fistula output with associated
metabolic disturbances, dehydration, skin breakdown, and death. Several
comprehensive reviews have concluded that TPN clearly impacts on the
treatment course of the disease in patients with GI fistulas. The following
conclusions can be drawn from studies evaluating the use of TPN in patients
with enterocutaneous fistula. First, TPN increases spontaneous closure rate of
enterocutaneous fistulas but does not markedly decrease the mortality rate in

patients with fistulas. Second, if spontaneous closure of the fistula does not
occur, patients are better prepared for operative intervention because of the
nutritional support they have received. Finally, certain fistulas are associated
with a lower rate of spontaneous closure than others and should be treated
more aggressively surgically after a defined period of nutritional support
(unless closure occurs). 120 Appropriate guidelines for the use of TPN in
cancer patients include: a. Long-term TPN in patients with rapid progressive
tumor growth unresponsive to other therapy b. Mildly malnourished patients
undergoing surgery for a curable cancer c. Preoperatively administered TPN
prior to surgery or other therapy in patients with severe malnutrition d.
Patients in whom treatment toxicity precludes the use of enteral nutrition
Answer: c, d As a general rule, the most important factor to consider when
making decisions about the use of TPN in patients with cancer is the response
of the tumor to antineoplastic therapy. Appropriate guidelines would include
the following: Short-term TPN is indicated in severely malnourished patients
or in those in whom gastrointestinal or other toxicities preclude adequate
enteral intake for seven days or a longer period. TPN is not indicated in well
nourished or mildly malnourished patients undergoing therapy or surgery
who would be expected to be able to resume adequate nutrition in
approximately seven days. Long-term TPN is indicated in patients in whom
treatment associated toxicities preclude the use of enteral nutrition and
represent the primary impediment to the restoration of performance status.
These patients should be expected to be responding to anti-tumor therapy.
Long-term TPN is not indicated with rapidly progressive tumor growth which
is unresponsive to such therapy. 121 Which of the following statements(s)
is/are true concerning human energy requirement? a. In normal subjects, less
than 5% of basal energy requirement is spent on cardiac output and the work
of breathing b. Mechanical ventilation can decrease the energy expenditure
for normal respiration c. For a 70 kg male, average resting energy
consumption is almost 1500 kcal/day d. Similar increases in energy
expenditures are associated with elective surgery and trauma or thermal
injury Answer: a, c Basal energy requirements are measured with the subject

at rest when no external work is being done; the energy is used mainly for
transport and synthetic work within cells. A surprisingly small percentage (<
5%) of this energy is spent on cardiac output and the work of breathing in
normal subjects. In contrast, the work of breathing in individuals with chronic
obstructive lung disease or in patients on a ventilator may account for 15
20% of caloric expenditure. The average resting post-absorptive 70 kg male
consumes about 1500 kcal/day. Energy needs increase as severity of illness
increases. The expenditure of kcal is only minimally increased after elective
surgery. The largest increase in energy expenditure occurs in patients with
severe multiple trauma or major thermal injury. The average-sized adult who
sustains a major burn rarely may require more than 3500 kcal/day for
maintenance. 122 Which of the following complications of TPN are
appropriately managed with the listed treatment? a. Air embolismplace
patient in reverse Trendelenburg and the left lateral decubitus position and
aspirate venous air b. Hyperchloremic metabolic acidosisgive sodium and
potassium as acetate salts c. Carbon dioxide retentiondecrease glucose
calories and replace with fat d. Line sepsisintravenous antibiotics Answer:
b, c A number of complications of TPN can occur which can be divided into
three types: mechanical, metabolic, and infectious. 123 A 55-year-old male
undergoes a total abdominal colectomy. Which of the following statement(s)
is/are true concerning the hormonal response to the surgical procedure? a.
Adrenocorticotropic hormone (ACTH) is secreted from the anterior pituitary
gland b. ACTH stimulation results in elevation of serum cortisol levels for up
to a week after the operation c. An increased secretion of aldosterone and
ADH may contribute to postoperative fluid retention d. An increase in serum
insulin and a fall in glucagon accelerate hepatic glucose production and
maintain gluconeogenesis Answer: a, c One of the earliest consequence of a
surgical procedure is the rise in levels of circulating cortisol that occur in
response to a sudden outpouring of ACTH from the anterior pituitary. The
rise in ACTH stimulates the adrenal cortex to elaborate cortisol which
remains elevated for 2448 hours after operation. The neuroendocrine
responses to operation also modify the various mechanisms that regulate salt

and water excretion. Alterations in serum osmolarity and tonicity of body


fluids secondary to anesthesia and operative stress, stimulate the secretion of
aldosterone and ADH. Thus, the ability to excrete a water load after elective
surgical procedures is restricted, and weight gain secondary to salt and water
retention is usual following an operation. Alterations occur in response to the
endocrine pancreas following elective operation. Insulin elaboration is
diminished and glucagon concentrations rise. The rise in glucagon and the
corresponding fall in insulin are important signals to accelerate hepatic
glucose production, and, with other hormones (epinephrine and
glucocorticoids), gluconeogenesis is maintained. 124 A number of
prospective clinical trials have addressed the role of total parenteral nutrition
in the cancer patient. The results have been somewhat conflicting. Which of
the following statement(s) have been proven correct by prospective trials? a.
Preoperative TPN is beneficial in surgical patients with severe preoperative
nutrition b. Postoperative TPN is of value following pancreatic resection c.
Routine use of perioperative (including prior to the procedure) TPN is of
benefit in patients undergoing hepatectomy for hepatoma d. TPN is of no
benefit in patients undergoing bone marrow transplant Answer: a, c
Numerous clinical trials have failed to yield a consensus with regard to the
efficacy of TPN in cancer patients. In 1991, a multicenter VA cooperative
trial demonstrated that preoperative TPN is of benefit in surgical patients
(many of whom had cancer) with severe preoperative malnutrition. Another
study examined the use of routine postoperative TPN following major
pancreatic resection. Patients randomized to receive TPN starting on
postoperative day 1 were noted to have an increased incidence of intraabdominal abscesses as well as a tendency towards increased incidence in
peritonitis and bowel obstruction. These investigators concluded that routine
use of postoperative TPN was not indicated and may, in fact, be harmful
following pancreatic resection. In another study, however, perioperative
(starting 7 days prior to the planned procedure) TPN for patients undergoing
hepatectomy for hepatocellular carcinoma demonstrated that this regimen
statistically reduced infectious complications compared to patients who did

not receive TPN. This was one of the few studies that demonstrated that
routine TPN (without the requirement of severe preoperative malnutrition)
was of benefit. The use of TPN in patients receiving bone marrow
transplantation has also been shown to be a valuable component of overall
care. 125 Which of the following statements concerning perioperative
nutrition is true concerning the above-described patient? a. Since the patients
weight had been stable with no preoperative nutritional deficit, 5% dextrose
intravenous solutions are adequate for the initial postoperative source of
nutrition b. Preoperative immunologic status should be determined including
total peripheral lymphocyte count and delayed hypersensitivity reaction to
determine skin-test response to common antigens c. Routine postoperative
fluid administration with intravenous 5% glucose solutions can provide the
calories to meet basal energy requirements d. A jejunal feeding catheter
should be placed at the time of surgery for postoperative enteral feeding
Answer: a Most patients undergoing elective operations are adequately
nourished. Unless the patient has suffered significant preoperative
malnutrition, characterized by weight loss greater than 1015%, or has major
intraoperative or postoperative complications, solutions containing 5%
dextrose may be administered for five to seven days before initiation of
enteral nutrition, with no detrimental effect on outcome. The usual
postoperative surgical patient is given intravenous glucose at 125 cc/hour
receives about 500 kcal/day, far less than the actual number of kcal needed to
meet energy requirements. The increased cost of feedings and potential
complications associated with intravenous nutrition cannot be justified.
Although the use of jejunal feedings in the postoperative period may be
useful in some patients, especially those undergoing extensive
gastrointestinal surgery, this technique would not appear indicated in the
patient described above. 126 The neurohormonal arm of the stress response is
well defined. Less is known about the inflammatory arm mediated primarily
by cytokines. Which of the following statement(s) is/are true concerning this
arm of the surgical stress response? a. Cytokines primarily work locally via
direct cell-to-cell communication b. Cytokines are never detectable in the

systemic bloodstream c. Cytokines are produced only by immune cells


attracted to the site of injury d. Cytokine release may stimulate the release of
other cytokines leading to an important cascade of events Answer: a, d
Cytokines, which are produced at the site of injury by endothelial cells and
by diverse immune cells throughout the body, also occupy a pivotal position
in the stress response. Cytokines differ from classic endocrine hormones in
that they are produced by a variety of cell types and in that they have the
capacity to exert their tissue effects locally via direct cell-to-cell
communications in a paracrine and/or autocrine fashion. Cytokines can
stimulate the production of other cytokines, leading to important cascades
which both amplify and diversify the effects of the proximal cytokine.
Occasionally, when in excess, cytokines act as hormones and spill over into
the systemic circulation and become detectable in the bloodstream. 127
Which of the following tissues contain significant collagen useful for placing
sutures to allow the prolonged tension necessary to maintain tissue
approximation? a. Dermis b. Intestinal submucosa c. Muscular fascia d.
Blood vessel wall Answer: a, b, c, d It takes at least three weeks for collagen
to undergo sufficient remodeling and cross linking to attain moderate
strength. Since most skin sutures are removed at one to two weeks, the
wound has only a small fraction of its eventual strength and may therefore
disrupt with even modest stress. Therefore, deep sutures are placed in
collagen containing structures to maintain the prolonged tension necessary.
Dermis, intestinal submucosa, muscular fascia, tendon, ligament, Scarpas
fascia, and blood vessel wall represent a partial list of tissues with high
collagen content. 128 Products of platelet degranulation include: a. Tumor
necrosis factor b. Interleukin-1 c. Transforming growth factor b d. Plateletderived growth factor Answer: c, d The initial response to injury and
disruption of a blood vessel is bleeding. The hemostatic response to this is
clot formation to stop hemorrhage. Platelet plug formation initiates the
hemostatic process along with clotting factors activated by collagen and the
basement membrane proteins exposed by the injury. Platelets then
degranulate, releasing the contents of their alpha granules and dense granules,

most notably platelet derived growth factor and transforming growth factor b.
These substances initiate chemotaxis and proliferation of inflammatory cells,
beginning the inflammatory response that will ultimately heal the wound.
Tumor necrosis factor and interleukin-1 also stimulate fibroblast
proliferation, however are produced by macrophages. 129 A patient with
gross fecal contamination and peritonitis from a ruptured sigmoid
diverticulum has his midline wound left open to heal by secondary intention.
Which of the following statement(s) describes this healing process? a.
Wounds healing in this fashion have an altered sequence of healing compared
to a primarily closed wound b. A bed of granulation tissue forms over
exposed subcutaneous tissue c. Epithelialization is enhanced in the face of
bacterial colonization d. The ability of a wound to form granulation tissue is
dependent on the blood supply of the tissue Answer: b, d Open wounds,
whether they be ulcers or open surgical incisions closing by secondary
intention, heal with the same sequence of inflammation, matrix deposition,
epithelialization, and scar maturation as in all wounds. The major difference
is in the healing incisional wound, the healing process progresses in an
orderly temporal sequence. In an open wound, the healing events are spatially
separated. In the healing wound, a bed of granulation tissue forms over the
exposed subcutaneous tissue. Granulation tissue is composed of new
capillaries, proliferating fibroblasts, an immature matrix of collagen,
proteoglycans, substrate adhesion molecules, and acute and chronic
inflammatory cells. Granulation tissue is the cobblestone pink surface of the
healthy new tissue in an open wound. The ability of an open wound to form
granulation tissue is governed by the blood supply to the tissue and the
relative absence of devitalized tissue and bacteria. Epithelialization is
therefore enhanced by limiting bacterial growth which presumably interferes
via bacterial and phagocytic cell products such as proteases, collagenases,
elastases, and other enzymes. 130 Which of the following factors can be
associated with impaired wound healing? a. Chemotherapy b. Chronic steroid
use c. Peripheral vascular disease d. Radiation therapy e. Diabetes mellitus
Answer: a, b, c, d, e Bone marrow suppression, a common consequence of

chemotherapy, is detrimental to wound healing. Quantitative and qualitative


lymphocyte and monocyte deficiency impairs cellular proliferation in the
inflammatory phase of wound healing. Any chemotherapeutic agent that
suppresses the bone marrow will impair healing. Glucocorticoids inhibit
wound healing based on their anti-inflammatory and immunosuppressive
effects. The anti-inflammatory effect of steroids is, in part, the result of
inhibiting arachidonic acid metabolism by impairing macrophage migration,
and by altering neutrophil function. Glucocorticoids also inhibit the synthesis
of procollagen by fibroblasts, thus delaying wound contraction. Radiation
injury leads to arteriolar fibrosis and impaired oxygen delivery. In addition,
there is progressive obliteration of blood vessels in the radiated area over
time. Radiation also causes intranuclear and cytoplasmic damage to
fibroblasts, and this appears to limit their proliferative potential. Diabetes
mellitus is often associated with decreased healing of open wounds and
increased susceptibility of infection. Many factors contribute to poor healing
in diabetic patients and most of them reflect local wound ischemia. However,
healing is not impaired in a normally perfused area in a well-controlled
diabetic. Peripheral arterial occlusive disease secondary to atherosclerosis can
be a primary cause of impaired healing, and may be also a cofactor with other
conditions. 131 Which of the following cells or blood elements play a role in
the initial phases of wound healing? a. Polymorphonuclear leukocytes
(PMNs) b. Platelets c. Monocytes d. Lymphocytes Answer: a, b, c, d Shortly
after the initial injury, the wound is full of debris which is cleared over the
next several days by recruited and activated phagocytic cells. PMNs begin to
arrive immediately, reaching large numbers within 24 hours. The PMNs are
followed by macrophages which appear in wounds in significant numbers
within two to three days. Macrophages are mononuclear phagocytic cells
derived from circulating monocytes or resident tissue macrophages. They
complete the process of removing all material not necessary for the ensuing
steps of wound healing. Lymphocytes also appear in wounds in small
numbers during the inflammatory response. The role of lymphocytes in the
wound healing process remains to be clarified, but they are thought to be

more related to the chronic inflammatory processes than the initial response
to wounding. Platelets are anuclear discoid blood elements derived from bone
marrow megakarocytes which play a role in the initial hemostatic process as
well as releasing chemotactic factors and factors leading to fibroblast
proliferation. 132 Which of the following surgical techniques lead to
improved wound healing? a. Atraumatic handling of tissue b. Approximation
of underlying fatty tissue to obliterate dead space c. Protecting the wound
from water for at least one week d. Meticulous hemostasis Answer: a, d There
are numerous practical implications for the care of wounds and surgical
incisions. Meticulous hemostasis reduces the inflammation of phagocytosis
necessary to clear the wound of blood. Atraumatic handling of tissue
decreases the load of necrotic or nonviable cells at the wound margin. Deep
sutures are best placed only into collagen laden structures that will hold
tension, i.e., fascia and dermis. These tissues have a tensile strength to hold
sutures under tension. Fat does not contain collagen and will not hold tension.
Therefore, fatty tissue should not be sutured as a separate layer. Given that
epithelialization of an incision is normally complete within 2448 hours,
there is no reason to protect the incision from water beyond this time period.
Allowing the patient to wash or shower one or two days after surgery actually
serves useful purpose in debriding the wound. 133 Which of the following
statement(s) is/are true concerning the clinical management of an open
wound? a. A wet-to-dry dressing is the most optimal form of wound
management b. A moist occlusive dressing promotes epithelialization and
reduces pain c. The protein rich plasma exudate covering the open wound
facilitates healing d. Irrigation of the wound disrupts epithelialization
therefore inhibiting the healing process Answer: b Epithelialization is more
rapid under moist conditions than dry conditions. Without dressings, a
superficial wound, or one with minimal devitalized tissue forms a scab or
crust, meaning that the blood and serum will coagulate, dry, and form a
protective moisture barrier over the open wound. If a wound is kept moist
with an occlusive dressing, then epithelial migration is optimized. In addition,
the pain of an open wound is dramatically reduced under an occlusive

dressing. The traditional wet-to-dry dressing if truly left to dry, simply


produces desiccation and necrosis of the surface layer of the wound which
delays epithelialization. Although wet-to-dry dressings can be effective for
debridement of wound exudate, they are generally less desirable than a moist
healing environment combined with effective cleaning of the wound (i.e.
water irrigation). Any open wound will leak plasma. With more
inflammation, the plasma capillary permeability is further increased. This
exudate of serum proteins and inflammatory cells serves as a rich culture
medium. This, in turn, will continue to cycle bacterial proliferation and lead
to further exudate formation. The net result of this cycle is delayed or absent
wound healing. In addition, the edema that results from capillary dysfunction,
increases the distance for diffusion from oxygen and nutrient sources to their
metabolic targets. 134 Which of the following statement(s) is/are correct
concerning the management of an open wound? a. Frequent surgical
debridement is usually necessary b. Water irrigation can effectively debride
most wounds c. Hydrogen peroxide is particularly useful in the management
of open wounds d. A number of the newer dressing products have clearly
been shown to promote wound healing compared to simple moist occlusive
dressing Answer: b Although there are numerous dressing products
commercially available at present, no treatment has been demonstrated to
improve healing beyond that of standard treatment which adheres to basic
principles. In the absence of large amounts of necrotic tissue, wound
debridement does not need to be accomplished surgically. Simple water
irrigation either with whirlpool or by water from a hand held shower spray
can generate enough power to effectively debride most wounds. Frequent
moist dressing changes can accomplish this as well, and in some cases,
occlusive absorptive dressings can generate enough tissue proteases to
effectively degrade proteins which the absorptive dressings remove. Deeper
portions of a wound may accumulate exudate and bacteria. In such cases,
water irrigation may be particularly useful. Commonly used agents such as
hydrogen peroxide actually may be harmful to normal tissue and are weak
oxidants and do a poor job of debriding. Enzymatic debriding agents can be

effective when used properly. Most of the newer dressing products have been
designed to be more absorptive and achieve moist healing without infection
from excess exudate. However, it must be emphasized that as long as moist
healing is achieved, there has been no evidence that one product is better than
another. 135 Which of the following statement(s) is/are true concerning the
proliferative phase of wound healing? a. The macrophage is the predominant
cell type b. The pink or purple-red appearance of a wound is due to ingrowth
and proliferation of endothelial cells c. Collagen, the dominant structural
molecule of the wound matrix, contains two unique amino acids,
hydroxyproline and hydroxylysine d. The predominant collagen type in a scar
is type 3 Answer: b, c The proliferative phase of wound healing begins with
the formation of a provisional matrix of fibrin and fibronectin as part of the
initial clot formation. Initially, the provisional matrix is populated by
macrophages; however, by day three fibroblasts appear in the fibronectinfibrin framework and initiate collagen synthesis. Fibroblasts proliferate in
response to growth factors become the dominant cell type during this phase.
Growth factors produced by macrophages simultaneously induce
angiogenesis which results in the ingrowth and proliferation of endothelial
cells, forming new capillaries. This neovascularity is visible through the
epithelium and gives the wound a pink or purple-red appearance. Collagen is
the dominant structural molecule in the wound matrix and in the final scar.
Collagen is synthesized into an organized cable-like network in a multi-step
process with both intra- and intercellular components. The collagen molecule
has quantities of two unique amino acids, hydroxyproline and hydroxylysine.
The hydroxylization processes which form these amino acids require ascorbic
acid (vitamin C) and is necessary for the subsequent stabilization and cross
linkage of collagen. The principal collagen type scar is type 1, with lesser
amounts of type 3 collagen also present. 136 Which of the following
statement(s) is/are true about the role of macrophages in the wound healing
process? a. Macrophages are the dominant cell type during the inflammatory
phase of wound healing b. Macrophages are not essential for wound healing
c. The macrophage role in wound healing is limited to phagocytosis d.

Macrophages are a source of a number of humoral factors essential for


wound healing Answer: a, d Within three or four days after injury,
macrophages become the dominant cell type in the inflammatory phase of
wound healing. The role of macrophages is not limited only to phagocytosis.
In addition, macrophages are the source of more than 30 different growth
factors and cytokines. These growth factors induce fibroblast proliferation,
endothelial cell proliferation (angiogenesis), extracellular matrix production,
and recruit and activate additional macrophages. The result is the induction of
a wound healing amplification cycle as growth factors recruit macrophages
and elicit additional growth factor release. Experimental studies in which
antibodies, which either destroy PMNs or block certain aspects of their
function, have shown that wounds heal normally, but that healing is
significantly impaired without functional macrophages. These studies
confirm the dominant role of the macrophage and the inflammatory phase of
wound healing. 137 Which of the following statement(s) is/are true
concerning the role of antibiotics in wound care? a. Systemic antibiotics are
indicated for all open wounds b. Bacterial resistance can occur with systemic
but not topical antibiotics c. An indication for systemic antibiotic
administration is a granulation tissue bacterial count in excess of greater than
105 organisms/gram of tissue on quantitative analysis d. Silver sulfadiazine is
useful only for the management of burns Answer: c The role of antibiotics in
wound care is controversial. All open wounds are colonized with bacteria.
Only when surrounding tissue is invaded (cellulitis) are systemic antibiotics
clearly indicated. Antibiotics may also be useful in other situations such as
when granulation tissue has a high bacterial count (> 105 organisms/gram
tissue), or in the case of reduced resistance to bacteria such as in a diabetic
foot ulcer. The routine use of systemic antibiotics for chronic wounds should
be avoided to reduce the development of resistant bacterial strains within the
wound. Topical ointments are frequently used and can be useful. The topical
vehicle may help keep the wound moist and the bacterial count in the wound
may be lowered as the result. However, as with most antibiotics, resistant
organisms quickly emerge. Silver sulfadiazine, frequently used for burn care,

is also useful for chronic wounds. Its broad spectrum of activity, lack of
relevant drug-resistant plasmids in bacteria, and its low cost make it a good
choice.
138 Which of the following statement(s) is/are true concerning wound
contraction?
a. Wound contraction accounts for similar rates of reduction of wound size
regardless of their location
b. The fibroblast, at the cellular level, is the primary force driving wound
contraction
c. Excessive wound contraction, when occurring over a joint, may lead to
disability
d. Actin microfillaments are found in fibroblasts and may play a role in
wound contracture
Answer: b, c, d
Wound contraction is an important event which contrasts healing open
wounds and closed incisions. When open wounds contract, the surrounding
skin is pulled over the open wound to reduce its size. This can occur much
faster than epithelialization. As opposed to other animals, human skin does
not have a significant degree of mobility in most sites and specifically on the
lower leg, the skin is tightly adherent and less elastic. Therefore, although
contraction may account for 90% of reduction of wound size on the
perineum, it accounts for, at most, 3040% of healing of a lower leg ulcer. All
healing wounds generate a strong contractile force. When this force is exerted
across a joint, it may result in scar contracture which may limit the functional
range of motion. At the cellular level, the force which drives wound
contraction comes from fibroblasts. Fibroblasts, like muscle cells, contain
actin microfilaments. When these filaments increase in number, the cells take
a morphologic appearance of myofibroblasts. Myofibroblasts are seen in an
increased number in contracting wounds and are felt to play an active role in
the process of wound contraction.

139 There are a multitude of various dressings available. Which of the


following statement(s) is/are true concerning options for surgical dressings?
a. Hydrocolloids, such as karaya compounds, offer the primary advantage of
increased absorptive ability
b. Films, such as Op-site, provide a water impermeable environment to
achieve a dry wound
c. Impregnates are fine gauze impregnated with a variety of substances such
as antibiotics or moisturizing agents that adhere tightly to the wound and do
not require a secondary dressing
d. Absorptive powders and paste are highly useful in debriding necrotic and
fibrous material from wounds and absorbing wound serum
Answer: a, d
Although the simplest dressing of gauze and tape combined with the use of
antibacterial ointment can achieve moist wound healing in most patients. A
multitude of other products are available. These can be classified into films,
foams, hydrocolloids, hydrogels, and absorptive powders. Films are
semipermeable to water, generally made of polyurethane, and are
nonabsorptive. They are useful to achieve a moist wound healing
environment over a minimally exudative wound such as split thickness skin
graft donor sites. The hydrocolloids deserve special mention because they
have achieved widespread use. These agents contain hydrophilic materials
such as karaya or carboxymethyl cellulose with an adhesive material and are
covered by a semipermeable polyurethane film. The material adheres to the
skin surrounding the wound, is highly absorptive, and achieves a moist
healing environment. Impregnants are generally fine mesh gauze impregnated
with either moisturizing, antibacterial, or bactericidal compounds. They are
generally not adherent and require a secondary dressing. They do promote
reepithelialization and have a antiinfective effect when combined with
antibacterial or bactericidal agents. A variety of absorptive powders and
pastes are available which consist of starch copolymers or colloidal

hydrophilic particles. These agents have high absorbency for tissue wound
fluid and debride necrotic and fibrous material from the wound.
140 Which of the following statement(s) is/are true concerning the
remodeling phase of wound healing?
a. Total collagen content increases steadily through this phase
b. The normal adult ratio of collagen is approximately 4:1 of type I to type III
collagen.
c. Eventually a scar will achieve the strength of unwounded skin
d. The proteoglycans are responsible for the ground substance of the
extracellular matrix
Answer: b, d
The transition from the proliferative phase to the remodeling phase of wound
healing is defined by reaching collagen equilibrium. Collagen accumulation
within the wound becomes maximal by two to three weeks after wounding.
Although supramaximal rates of synthesis and degradation continue
throughout remodeling, there is no further change in total collagen content.
During the initial phase of wound healing, there is a relative abundance of
type III collagen in the wound. With remodeling, the normal adult ratio of 4:1
(type I to type III) collagen is restored. The other important component of the
extracellular matrix is the ground substance or proteoglycans. These
substances are composed of a protein background with long hydrophilic
carbohydrate side chains. The hydrophilic nature of these molecules accounts
for much of the water content of scar.
Scars never achieve the degree of order advanced by collagen in normal skin
or tendons, but they do increase in strength for six months or more,
eventually reaching 70% of the strength of unwounded skin.
141 Which of the following statement(s) is/are true concerning
pharmacologic agents used to accelerate wound healing?
a. A number of these agents are now currently approved for use in this

country
b. PDGF (platelet-derived growth factor) promotes fibroblast proliferation,
chemotaxis, and collagenase synthesis
c. PDGF has been demonstrated in a number of clinical trials to promote
healing in chronic wounds
d. Growth hormone functions by promoting fibroblast proliferation and
collagen synthesis
Answer: b, c
Currently there are no approved clinical agents that accelerate normal
healing. Although a number of clinical trials are in progress, no agents are
currently approved. PDGF (platelet-derived growth factor) accelerates wound
healing by promoting fibroblast proliferation and chemotaxis and collagenase
synthesis. Clinical trials have demonstrated that PDGF has accelerated
healing in patients with chronic wounds such as pressure sores and diabetic
ulcers. Growth hormone has been successfully used in some situations to
reverse the catabolic effect of severe injuries. Wound healing is
fundamentally an anabolic event, and in the setting of a severe burn, growth
hormone administration significantly accelerates donor site healing,
presumably due to its effects in minimizing catabolism.
142 Which of the following statement(s) describe the effects of aging on
wound healing?
a. A finer, more cosmetic scar might be expected
b. In vitro studies demonstrate decreased proliferative potential of fibroblasts
and epithelial cells
c. Skin sutures should be left in for a longer period of time
d. Wound infection occurs more frequently in elderly patients due to
diminished ability to fight infection
Answer: a, b, c
There are important age-dependent aspects of wound healing. The elderly

heal more slowly and with less scarring. There is a gradual attenuation of the
inflammatory response with age, and decreased wound healing is one of the
consequences. In vitro studies have documented an age-dependent decrease
in proliferative potential of fibroblasts and epithelial cells. Clinically this will
account for the formation of finer scars and improved cosmetic appearance in
the elderly. Sutures should be left in place longer to allow for the slow regain
of tensile strength in the aged. This can also be done without concern for
formation of suture marks as slower epithelialization occurs along the
sutures. There is no evidence to suggest that wound infections occur more
commonly in elderly patients.
143 Reconstitution of the epithelial barrier (epithelialization) begins within
hours of the initial injury. Which of the following statement(s) is/are true
concerning the process of epithelialization?
a. Bacteria, protein exudate, and necrotic tissue all will compromise this
process
b. Epithelial cells exhibit contact proliferation
c. Epithelialization occurs only from the margins of the wound
d. Visible scarring can occur only when the injury extends deeper than the
superficial dermis
Answer: a, d
The initial step of epithelialization involves epithelial cells from the basal
layer of the wound edge flattening and migrating across the wound,
completing wound coverage within 2448 hours in a co-opted surgical
wound. Epithelial cells exhibit contact inhibition. That is, they will continue
to migrate across an appropriate bed until a single continuous layer is formed.
Epithelial cell migration occurs by a process in which the epithelial cells send
out pseudopods, attaching to the underlying extracellular matrix by integrin
receptors. Bacteria, large amounts of protein exudate from leaky capillaries,
and necrotic tissue all compromise this process delaying epithelialization. In
the case of open wounds, epithelialization results from migration of epithelial

cells from remaining dermal appendages, sweat glands, and hair follicles, if
the dermis is not completely destroyed. In a full thickness injury, the entire
dermis is destroyed or removed. Epithelialization therefore occurs only at the
margins of a wound, at a dermal rate of 12 mm/day.
Visible scarring occurs only when the injury extends deeper than the
superficial dermis. Superficial abrasions and burns usually heal without scar,
while deeper abrasions and burns may scar significantly. Whenever the
dermis is incised, a scar will form.
144 Scar formation is part of the normal healing process following injury.
Which of the following tissues has the ability to heal without scar formation?
a. Liver
b. Skin
c. Bone
d. Muscle
Answer: c
Every tissue in the body undergoes reparative processes after injury. Bone has
the unique ability to heal without scar and liver has the potential to regenerate
parenchyma, the only organ that has maintained that ability in the adult
human. Although liver does regenerate, it often heals with scar (cirrhosis) as
well. With these exceptions, all other mature human tissues heal with scar.
145 Which of the following factors have been demonstrated to promote
wound healing in normal individuals?
a. Vitamin A supplementation
b. Vitamin C supplementation
c. Vitamin E application to the wound
d. Zinc supplementation
e. None of the above
Answer: e

Several important systemic factors or conditions influence wound healing.


Interestingly, there are no known systemic conditions that lead to enhanced or
more rapid wound healing. Overall nutrition as well as adequate vitamins
play an important role in wound healing. Vitamin A is involved in the
stimulation of fibroplasia, collagen cross-linking, and epithelialization.
Although there is no conclusive evidence in humans, vitamin A may be useful
clinically for steroid-dependent patients who have problematic wounds or
who are undergoing extensive surgical procedures. Vitamin C is a necessary
cofactor in hydroxylization of lysine and proline in collagen synthesis and
cross-linkage. The utility of vitamin C supplementation in patients who
otherwise take in a normal diet has not been established. Vitamin E is applied
to wounds and incisions empirically by many patients. The evidence to
support this practice is entirely anecdotal. In fact, large doses of vitamin E
have been found to inhibit wound healing. Zinc and copper are also important
cofactors for many enzyme systems that are important to wound healing.
Deficiency states are seen with parenteral nutrition but are rare and readily
recognized and treated with supplements. Overall, vitamin and mineral
deficiency states are extremely rare in the absence of parenteral nutrition or
other extreme dietary restrictions. There is no evidence to support the concept
that supranormal provision of these factors enhance wound healing in normal
patients.
146 Which of the following statement(s) is/are true concerning excessive
scarring processes?
a. Keloids occur randomly regardless of gender or race
b. Hypertrophic scars and keloid are histologically different
c. Keloids tend to develop early and hypertrophic scars late after the surgical
injury
d. Simple reexcision and closure of a hypertrophic scar can be useful in
certain situations such as a wound closed by secondary intention
Answer: d

True keloids are uncommon and occur predominantly in dark skinned people
with a genetic predisposition for keloid formation. In most cases, the gene
appears to be transmitted as an autosomal dominant pattern. The primary
difference between a keloid and a hypertrophic scar is that a keloid extends
beyond the boundary of the original tissue injury. It behaves as a tumor and
extends into or invades the normal surrounding tissue creating a scar that is
larger than the original wound. Histologically, keloids and hypertrophic scars
are similar. Both contain an overabundance of collagen. Although the
absolute number of fibroblasts is not increased, the production of collagen
continually out paces the activity of collagenase, resulting in a scar of ever
increasing dimensions. Hypertrophic scars respect the boundaries of the
original injury and do not extend into normal unwounded tissue. There is less
of a genetic predisposition, but hypertrophic scars also occur more frequently
in Orientals and the Black population. They are often seen on the upper torso
and across flexor surfaces. Some improvement in a keloid can be obtained
with excision followed by intra-lesional steroid injection. However, the
resulting scar is unpredictable and potentially worse. Reexcision and closure
should, however, be considered for hypertrophic scars, if the condition of
closure can be improved. This is especially pertinent for wounds that
originally healed by secondary intention or that are complicated by infection.
Keloids typically develop several months after the injury and rarely, if ever,
subside. Hypertrophic scars usually develop within the first month after
wounding and often subside gradually.
147 Which of the following statement(s) is/are true concerning the vascular
response to injury?
a. Vasoconstriction is an early event in the response to injury
b. Vasodilatation is a detrimental response to injury with normal body
processes working to avoid this process
c. Vascular permeability is maintained to prevent further cellular injury
d. Histamine, prostaglandin E2 (PGE2) and prostacyclin (PGI2) are important
mediators of local vasoconstriction

Answer: a
After wounding, there is transient vasoconstriction mediated by
catecholamines, thromboxane, and prostaglandin F2 (PGF2a). This period of
vasoconstriction lasts for only five to ten minutes. Once a clot has been
formed and active bleeding has stopped, vasodilatation occurs in an around the
wound. Vasodilatation increases local blood flow to the wounded area, supplying the
cells and substrate necessary for further wound repair. The vascular endothelial
cells also deform, increasing vascular permeability. The vasodilatation and increased
endothelial permeability is medi

(vascular
endothelial cell growth factor). These vasodilatory substances are released by
injured endothelial cells and mast cells and enhance the egress of cells and
substrate into the wound and tissue.
d by histamine, PGE2, and prostacyclin as well as growth factor VEGF

148 Which of the following statement(s) concerning laboratory studies used in


monitoring a patient with intravenous heparinization is/are correct?
a. The platelet count should be followed because of the risk of heparin-associated
thrombocytopenia
b. The prothrombin time should be observed if prolonged treatment is necessary
c. The activated partial thromboplastin time (aPTT) should be maintained at
approximately 1.5 times normal

d. The serum creatinine should be measured daily to allow adjustments in


dose based on renal function
Answer: a, c
In monitoring the effect of heparin, an activated partial thromboplastin time
(aPTT) of 1.5 control or a thrombin clotting time (TCT) of 2 times control
reflects adequate anticoagulation. The prothrombin time remains normal.
Heparin-associated thrombocytopenia from an immune mechanism is a
potential complication of the use of this anticoagulant. Therefore any patient
undergoing heparin therapy should have a platelet count determined every
other day after the fourth day of therapy or earlier if he or she is known to
have been exposed to heparin in the past. Heparin is not excreted through the
kidneys or the liver but is cleared through the reticuloendothelial system.

Therefore the dose of heparin need not be adjusted in cases of liver or renal
dysfunction.
149 Which of the following statement(s) is/are true concerning heparinassociated thrombocytopenia?
a. Heparin-associated thrombocytopenia occurs only in the face of over
anticoagulation with heparin
b. Severe thrombocytopenia (platelet count less than 100,000) is seen in less
than 10% of patients treated with heparin
c. Heparin-associated thrombocytopenia is due to the aggregation of platelets
and may result in thrombosis or embolic episodes
d. Heparin-associated thrombocytopenia may be seen within hours of
initiation of heparin therapy
Answer: b, c
Heparin-associated thrombocytopenia occurs in 0.6% to 30% of patients who
receive heparin, although severe thrombocytopenia (platelet counts less than
100,000) is seen in fewer than 10% of patients treated with heparin. It is
caused by a plasma factor, most likely a heparin-dependent platelet antibody,
that causes aggregation of platelets when exposed to heparin. Activation of
platelets in this setting results in thrombocytopenia, thrombosis and embolic
episodes, which can lead to death. Both bovine and porcine heparin have
been associated with this syndrome, which usually begins 5 to 15 days after
initiating heparin therapy. Even trivial exposure with heparin such as coating
on pulmonary artery catheters or low rate infusion into arterial catheters may
cause this syndrome.
150 Antithrombin III deficiency is a commonly observed hypercoaguable
state. Which of the following statement(s) is/are true concerning this
condition?
a. A patient with this deficiency usually presents with thrombosis while on
heparin or exhibits an inability to become adequately anticoagulated with

heparin
b. This deficiency may be either congenital or acquired
c. Thrombotic episodes are related to predisposing events such as operations,
childbirth, and infections
d. Treatment involves acutely the administration of fresh frozen plasma
followed by long-term treatment with Coumadin
Answer: a, b, c, d
Antithrombin III deficiency accounts for about 2% of venous thrombotic
event. This deficiency has been described in patients with pulmonary
embolism, mesenteric venous thrombosis, lower extremity venous
thrombosis, arterial thrombosis, and dialysis fistula failure. Antithrombin III
is a serine protease inhibitor of thrombin and factors Xa, IXa and XIa.
Because one of the main actions of heparin is to potentiate the anticoagulant
effects of antithrombin III, a patient with this deficiency usually presents with
thrombosis while on heparin or exhibits the inability to become adequately
anticoagulated with heparin. This deficiency may be either congenital
(1n20005000 births) or acquired. Acquired defects occur with inadequate
production, as in liver disease, malignancy, nephrotic syndrome,
disseminated intervascular coagulation, malnutrition, or increased protein
catabolism. Thrombotic episodes are related to predisposing events such as
operations, childbirth, and infections. Once the diagnosis of antithrombin III
deficiency is established, fresh frozen plasma should be administered
followed by long-term treatment with Coumadin.
151 Mini-dose heparin has been shown to be useful in the prophylaxis of
postoperative venous thrombosis. Mechanism(s) by which low-dose heparin
is/are thought to protect against venous thrombosis include:
a. Enhancement of antithrombin III activity
b. A decrease in thrombin availability
c. Inhibition of platelet aggregation and subsequent platelet release action
d. A mild prolongation of activated partial thromboplastin time

Answer: a, b, c
Low-dose heparin is thought to protect against venous thrombosis through
three different mechanisms. First, antithrombin III activity with its inhibition
of activated Factor X is enhanced by only trace amounts of heparin; second,
there is a decrease in thrombin availability that prevents its activation and
thus its fibrin-stabilizing effect; and third, small doses of heparin may inhibit
the second wave of platelet aggregation and subsequent platelet release
reaction. The standard doses of heparin administered (5000 units bid) does
not affect aPTT.
152 Tests of coagulation are used to monitor anticoagulation treatment and
detect intrinsic abnormalities in coagulation. Which of the following
statement(s) is/are true concerning coagulation tests?
a. Prothrombin time (PT) measures both the intrinsic and extrinsic clotting
pathways and fibrinogen
b. Activated partial thromboplastin time (aPTT) can be used to monitor both
oral anticoagulation with Warfarin and intravenous anticoagulation with
heparin
c. Thrombin clotting time (TCT) is a measurement of the time it takes for
exogenously administered thrombin to turn plasma fibrinogen into fibrin clot
d. Whole blood activated clotting time (ACT) is a measurement of the ability
of whole blood to clot and is used to monitor heparin levels intraoperatively
during cardiovascular and peripheral vascular operations
Answer: a, c, d
Coagulation tests include prothrombin time (PT), which measures the
intrinsic and extrinsic pathways of fibrinogen production and is the most
common method for measuring a level of oral anticoagulant therapy. The
activated partial thromboplastin time (aPTT) identifies the abnormalities of
the contact and intrinsic phases of coagulation. Values of aPTT have variably
been shown to correlate with heparin dosages and serum heparin levels and

are therefore most commonly used in monitoring heparin therapy. It is of no


value in long-term management of patients on oral Warfarin therapy.
Thrombin clotting time (TCT) is the measure of the time it takes for
exogenously administered thrombin to turn plasma fibrinogen into fibrin clot.
It is extremely sensitive to levels of heparin and is an excellent measure of
measuring the level of heparin-induced anticoagulation. The beauty of the
TCT is that it is not specific for any disease condition; thus it may be used to
differentiate factor deficiencies from the presence of heparin, or to separate
lupus anticoagulant from abnormalities in fibrinogen levels. The whole blood
activated clotting time (ACT) is a measurement of the ability of whole blood
to clot, and as such, is an available technique for monitoring heparin levels
intraoperatively. The ACT responds in a linear fashion to increasing heparin
dosage and correlates well with the observed clinical anticoagulation.
Adequate anticoagulation for extracorporeal circulation is defined as an ACT
of 480 seconds or more while for peripheral vascular applications, values of
250 seconds or greater are considered appropriate.
153 Thrombolytic therapy has become a useful adjunct in the management of
peripheral arterial occlusion. In this setting, direct intraarterial administration
rather than intravenous has been advocated to decrease the risk of systemic
bleeding. Which of the following true statement(s) concerning the use of
thrombolytic agents for arterial occlusion is/are true?
a. A standard technique involves infusing high-dose urokinase, 4000 units per
minute for 12 hours, directly into the clot by a catheter embedded in the
thrombus
b. If progress is made, further fibrinolytic therapy is given at 1000 to 2000
units per minute until clot lysis has occurred
c. The usual infusion time by the above-stated technique is usually in excess
of 24 hours
d. Successful clot lysis occurs more frequently in arterial graft occlusions
than native arterial occlusions
e. The use of intraoperative thrombolytic therapy may be indicated for

situations where complete clot evacuation cannot be accomplished surgically


Answer: a, b, e
The most popular method for intraarterial thrombolytic therapy for arterial
occlusion involves passing a guidewire through the thrombus with
arteriographic guidance and then infusing high-dose urokinase, 4000 units
per minute for 12 hours, directly into the clot. If progress is made, further
fibrinolytic therapy is given at 1000 to 2000 units per minute for a 612 hour
period or until clot lysis has occurred. Using this technique, mean infusion
time in a recent study was found to be 18 hours and the incidence of bleeding
complications was significantly lessened. Selective intraarterial infusion of
urokinase was associated with complete clot resolution in 77% of native
arterial occlusions versus only 41% with arterial graft occlusion. After
thrombolytic therapy has reopened an occluded vessel or graft, radiologic or
surgical correction of the lesion responsible for the thrombosis in the first
place must be addressed for any hope of long-term success. The use of
intraoperative thrombolytic therapy is advocated in those situations where
complete clot resolution cannot be accomplished (such as following balloon
embolectomy for acute arterial occlusion) or when distal vasculative is
occluded and precludes appropriate inflow patency.
154 Which of the following statement(s) is/are true concerning hemophilia
A?
a. Hemophilia A is inherited as a sex-linked recessive deficiency of factor
VIII
b. A positive family history for bleeding disorders present in all patients
c. Laboratory tests reveal a prolongation of aPTT, prothrombin time (PT),
thrombin clotting time and platelet aggregation
d. Spontaneous bleeding is unusual with factor VIII levels greater than 10%
of normal
Answer: a, d

Hemophilia A is inherited as a sex-linked recessive deficiency of factor VIII


although 0% of cases are secondary to spontaneous mutation. The incidence
of this abnormality is approximately 1/10,000 births. Laboratory screening
tests usually reveal a prolongation of an aPTT but normal prothrombin time
(PT), thrombin clotting time (TCT) and platelet aggregation testing. The
minimum level of VIII required for hemostasis is 30% for minor bleeding,
whereas spontaneous bleeding is unusual with factor levels greater than 5 to
10% of normal. In severe genetic deficiency states however, factor levels as
low as 1% have been noted and patients are at risk for spontaneous bleeding.
155 Fibrinolytic therapy is based on activation of plasminogen, the inactive
proteolytic enzyme of plasma that binds to fibrin during the formation of
thrombosis. Activation of plasminogen to plasmin results in selective
thrombolysis at the fibrin clot surface. Which of the following statement(s)
is/are true concerning agents used in thrombolytic therapy?
a. Streptokinase is a bacterial protein which is antigenic in humans, resulting
in allergic reactions in up to l5% of cases
b. Tissue plasminogen activator acts directly on plasmin without an
intermediate drugplasmin complex
c. The half-life of urokinase, streptokinase, and TPA all exceed 30 minutes
d. Streptokinase is significantly cheaper than urokinase or TPA
Answer: a, b, d
Streptokinase is a bacterial protein produced by group C b-hemolytic
streptococci. It is therefore antigenic in humans and can be associated with
allergic reaction in between 2 and 18% of cases. In addition an unusual serum
sickness has been reported with streptokinase. Neither urokinase or TPA
which is now manufactured with recombinant DNA technology are either
associated with allergic side effects or antigenicity. Streptokinase acts
through a streptokinase-plasmin complex, whereas urokinase and TPA act
directly on plasmin without intermediate drug plasmin complex. The level of
the lytic state is greatest with streptokinase, intermediate with urokinase, and

least with TPA with the half-lives ranging all less than 1/2 hour in duration.
Although the relative efficacy of the three agents has been compared in a
number of studies, there appears to be no significant benefit of one agent over
the other. Streptokinase however, is markedly less expensive than either
urokinase or TPA.
156 Von Willebrands disease is a common, congenital bleeding disorder.
Which of the following statement(s) is/are true concerning Von Willebrands
disease?
a. As in hemophilia, it is much more common in men
b. A history of spontaneous bleeding is common
c. Screening laboratory tests will include a prolonged aPTT with a normal
prothrombin time
d. Pre-treatment for elective surgery require administration of cryoprecipitate
to achieve levels of 2350% of normal
Answer: c, d
Von Willebrands factor is an adhesive protein that mediates platelet adhesion
to collagen. In addition, it protects and prevents the rapid removal of factor
VIII from blood. The classical deficiency state, Von Willebrands disease, is
caused by reduction of factor VIII activity (although not as great as
Hemophilia A) and the Von Willebrand factor. Clinical manifestations include
epistaxis, gingival bleeding, menorrhagia, rare joint or muscle bleeding, and
subcutaneous bleeding. Spontaneous bleeding is not as common as in classic
Hemophilia A. The syndrome is transmitted as both autosomal dominant
(heterozygous) and autosomal recessive disease (homozygous) traits.
Therefore there is no sex predilection. Screening laboratory tests include a
prolonged aPTT with a normal prothrombin time. In addition, because of the
importance of this factor in platelet adhesion, patients display a prolonged
bleeding time and have decreased level of factor VIII activity, decreased
immunoreactive levels of Von Willebrands antigen, and abnormal platelet
aggregation responses to ristocetin. The most reliable source of Von

Willebrands factor is cryoprecipitate.


157 External pneumatic compression has been advocated for the prevention
of deep venous thrombosis during operative procedures. Which of the
following statement(s) concerning the use of external pneumatic compression
devices is/are true?
a. Intermittent pneumatic compression is as effective as low-dose heparin in
prevention of venous thrombosis
b. These devices function by compressing the lower extremities therefore
augmenting venous return
c. Pneumatic compression devices may also exhibit their antithrombotic
effect through stimulating local and systemic fibrinolysis
d. The length of time that intermittent pneumatic compression should be used
includes through the operation and for at least several days in the
postoperative period
Answer: b, c, d
In many well-controlled studies of venous prophylaxis, intermittent
pneumatic compression has been found to be as effective as low-dose heparin
therapy. In addition to augmentation of venous return with these devices,
local and systemic fibrinolysis appears to be stimulated. Fibrinolytic
activities are usually reduced for a 710 day period after an operation.
Studies have demonstrated that the pneumatic-compression devices may
exhibit their antithrombotic effect through prevention of this fibrinolytic
shutdown even when applied to the upper extremity. The length of time that
intermittent pneumatic compression should be used has not been adequately
determined but most data suggest that devices should be used through the
operation and for at least five days in the face or prolonged immobilization.
158 The standard management oral anticoagulant therapy for chronic
treatment of venous thromboembolism is with the drug warfarin. Which of
the following statement(s) is/are true concerning the administration of

warfarin?
a. An important complication of warfarin therapy is skin necrosis in patients
with protein C deficiency
b. Warfarin interferes with vitamin K dependent clotting factors II, VII, IX, X
c. For effective anticoagulation the prothrombin time (PT) should be kept at 2
control
d. It is recommended that warfarin be continued for at least one year after
initial episode of deep venous thrombosis
Answer: a, b
Warfarin interferes with the vitamin K dependent clotting factors II, VII, IX
and X, protein C, and protein S. An important complication of warfarin is
skin necrosis with patients both with and without protein C deficiency. This
syndrome usually involves full thickness skin slough over fatty areas such as
the breasts and buttocks. Warfarin therapy should be monitored using the one
stage prothrombin time (PT). The PT should be kept at 1.3 to 1.4 control for
effective anticoagulation. At higher levels, there is a five-fold increase in the
frequency of bleeding complications. Two major complications of Warfarin
therapy include recurrent thrombosis and bleeding. It is recommended that
Warfarin be continued four months after an initial episode of deep venous
thrombosis. Between ten weeks and four to six months after deep vein
thrombosis, there is a recurrent thrombosis rate of 8.3 episodes per 1000
patient months. Between four months and three years, recurrences fall to four
episodes per 1000 patient-months. At four months, the risks of bleeding
complications matches and exceeds the benefit from anticoagulant therapy
and thus is the basis for discontinuing warfarin administration at this time.
159 Which of the following statement(s) is/are true concerning the
management of a patient with hemophilia A undergoing an elective surgical
operation?
a. Concentrates of factor VIII should be given several days prior to elective
surgery

b. The half-life of factor VIII concentrates is less than 24 hours


c. A dose of 4050 IU/kg of factor VIII concentrate should be given prior to
the planned surgical procedure
d. Factor VIII concentration administration should be given for the first 24
hours after surgery but may then be stopped if no abnormal bleeding has been
observed
e. A new recombinant preparation of factor VIII offers the advantage of being
virus-free
Answer: b, c, e
Although the half-life of factor VIII is 2.9 days in normal individuals, the
half-life of factor VIII concentrates is 9 to l8 hours. Levels of 80% to 100%
of normal should be obtained for surgical bleeding or life-threatening
hemorrhage. A dose of 40 to 50 IU/kg of factor VIII should be given with half
of this dose then administered every twelve hours. After surgery, transfusion
of factor VIII concentrates should be continued for at least ten days.
Unfortunately, past use of concentrates of factor VIII obtained from donors
has led to a high incidence of HIV infection in the hemophilia population. A
new recombinant preparation of factor VIII offers the advantage of being
virus-free.
160 Transfusions of blood products can be associated with a number of
complications including immediate and delayed hemolytic reactions;
nonhemolytic reactions; infectious disease transmission; and complications of
massive transfusions. Which of the following statements are true concerning
complications of blood transfusions?
a. Immediate hemolytic transfusion reactions are caused by major ABO blood
group incompatibility
b. Nonhemolytic transfusion reactions are usually due to RH incompatibility
and are therefore more common in women of childbearing age
c. The most common complication of massive blood transfusion is dilutional
thrombocytopenia

d. Routine impaired calcium supplementation is necessary during most


massive transfusion episodes
Answer: a, c
Immediate hemolytic reactions are usually caused by blood group ABO
incompatibility although they may be caused by antigens of other blood
group systems on the transfused red blood cells. The clinical manifestations
revolve around the antigen on the red blood cell stroma and the antibody in
the patients serum, and include production of bradykinin, compliment
activation, release of vasoactive agents from platelets, and initiation of
systemic clotting. Chills and fevers, chest pain and lumbar pain, tachycardia
and hypotension in the conscious patient, and often diffuse bleeding in the
anesthetized, unconscious patient constitute this syndrome. Although reaction
occurs immediately, death related to the syndrome is uncommon, unless
associated with a transfusion of more than 100 ml of blood. Death usually
occurs from acute renal failure or hemorrhage due to DIC. Nonhemolytic
reactions occur with the frequency of 1 to 2% of all transfusions and consist
primarily of chills and fevers during the transfusion or in the first 2 to 3 hours
after the transfusion is complete. Mechanism of these reactions includes the
presence of antibodies to white blood cell antigens in the transfused blood,
especially in the multitransfused or multiparous patient. Massive transfusion
complications relate to the rate and volume of blood transfused. The most
common complication is dilutional thrombocytopenia. Factor deficiency of
the labile factors V and VIII rarely is of sufficient magnitude to result in
problems with hemostasis. For hypocalcemia to occur with massive
transfusion, citrated blood must be administered, one unit every five minutes.
Routine empiric calcium supplementation is unnecessary during most
massive transfusion episodes. Conversely, hypothermia is clearly a problem,
especially when associated with massive transfusion during complex
intraoperative procedures such as thoracoabdominal aneurysm resection.
161 A 67-year-old male with advanced cholangiocarcinoma develops gram-

negative sepsis. Excessive bleeding is noted around vascular catheters and


from needle puncture sites. The diagnosis of disseminated intervascular
coagulation (DIC) is considered. Which of the following laboratory test(s)
is/are indicative of DIC?
a. Decreased platelet count
b. Decreased fibrinogen level
c. Normal prothrombin time
d. Elevated fibrin split products
Answer: a, b, d
Disseminated intravascular coagulation (DIC) is the primary form of acute
thrombosis. Causes of this syndrome include abruptio placenta, gram-positive
and gram-negative sepsis, endotoxemia, malignant tumors, pelvic operations,
certain snake bites, hematologic malignancies, and hepatic failure. Blood
coagulation is activated by the release of tissue factor into the circulation,
which activates factor VII of the extrinsic pathway to VIIa, leading to
massive thrombin production and fibrin generation. This in turn activates the
fibrinolytic system, leading to bleeding in the later stages of the syndrome
due to consumption of coagulation factors, depletion of fibrinogen, and
unchecked plasma activities. Laboratory values in DIC usually include a
decline in the platelet count and fibrinogen level, along with an elevation of
fibrin split products.
162 Which of the following substances, not normally present in the
circulation, trigger the initiating events in the hemostatic process?
a. Thrombin
b. Platelet factor 3
c. Tissue factor
d. Collagen
Answer: c, d
The initiating agents for hemostasis involve two substances that are not

normally present in the circulationcollagen and tissue factor. Tissue factor


is released from injured cells, beginning the activation of the extrinsic
pathway of coagulation, while disruption of the protective endothelial barrier
of blood vessels exposes the underlying collagen to the activation of platelets.
In the bloodstream, tissue factor complexes with factor VII which then
activates factor X to factor Xa. At the same time, activated platelets change
from their discoid shape with their procoagulant phospholipid (termed
platelet factor 3) buried on the inner side of the surface membrane to a
spreading shape to allow for the externalization of platelet factor 3 activity.
Activated factor X, activated factor V, ionized calcium and factor II
(prothrombin) then assemble on the platelet phospholipid surface to form the
so-called prothrombinase complex which catalyzes the formation of
thrombin.
163 Bleeding complications are frequently associated with fibrinolytic
therapy. Which of the following statement(s) concerning complications of
fibrinolytic therapy is/are true?
a. Careful monitoring of prothrombin time and aPTT time are necessary to
avoid bleeding complications
b. A level of serum fibrinogen less than 100 mg/dl is associated with an
increased risk of bleeding
c. Recent (less than 10 days) major surgery is a contraindication to systemic
but not regional fibrinolytic therapy
d. A patient with a cerebrovascular event occurring less than two months ago
can be treated with fibrinolytic therapy if head CT scan is normal
Answer: b
Fibrinolytic therapy induces a hemostatic defect through a combination of
factors. Hypofibrinogenemia and fibrin degradation products inhibit fibrin
polymerization and, in combination with a decrease in the clotting factors V
and VIII, prolong the ability of blood to clot. However, coagulation tests in
general do not correlate well with bleeding complications. A level of

fibrinogen less than 100 mg/dl is associated with an increased risk of


bleeding. Absolute contraindications to thrombolytic therapy include active
internal bleeding, recent (less than 2 months) cerebral vascular accident, and
documented left heart thrombosis. Recent (less than 10 days) major surgery,
obstetric delivery, organ biopsy, or major trauma is considered a major
relative contraindication to either regional or systemic thrombolytic therapy.
164 Which of the following statement(s) is/are true concerning the results of
a National Institute of Health Consensus Conference on venous thrombosis
and low-dose heparin prophylaxis?
a. The odds of developing deep venous thrombosis with low-dose heparin
prophylaxis decreases by 67%
b. The risk of pulmonary embolism is decreased by almost 50%
c. There is no increase in mortality from other causes found in patients treated
with low-dose heparin
d. There was no difference in the incidence of bleeding complications
Answer: a, b, c
In a metaanalysis of 70 randomized trials in 16,000 patients comparing lowdose heparin prophylaxis with standard therapy, the odds of developing deep
venous thrombosis with low-dose heparin prophylaxis decreased 67%,
whereas for pulmonary embolism (both fatal and non-fatal), the odds
decreased by 47%. For fatal pulmonary embolism, the odds reduction was
even greater (64%). No increase in mortality from other causes was found in
those patients treated with low-dose heparin. Bleeding complications were
more frequent in the heparin-treated patients, with no difference between
5000 units twice daily and 5000 units three times daily. Similarly, the
effectiveness of prophylaxis was not influenced by either two or three times
daily dosage.
165 Laboratory monitoring of coagulation and anticoagulation includes
testing of platelet function. Which of the following statements is/are true

concerning tests of platelet function?


a. A platelet count of 50,000/L or more usually ensures hemostasis
b. Bleeding time assays assessibility of platelets to perform hemostatic plugs
and is determined from a sample of blood drawn in an EDTA coated test tube
c. Aspirin therapy can be associated with a bleeding time in the range of 815
minutes
d. Tests of platelet aggregation should be part of the standard preoperative
evaluation of patients using aspirin
Answer: a, c
Tests of platelet function include peripheral platelet counts, bleeding times,
and platelet aggregation. Usually, a platelet count of 50,000/mL or more
ensures adequate hemostasis, whereas counts less than 10,000/mL are
dangerous and may lead to spontaneous bleeding. Bleeding time performed
by observing the clotting of blood induced with a small needle stick, assesses
the ability of platelets to perform hemostatic plugs and are usually shorter
than eight minutes. A bleeding time between 8 and 15 minutes most often
reflects a low plasma level of Von Willebrands Factor or the use of
antiplatelet drugs. A bleeding time greater than 15 minutes is clearly
prolonged and indicates severe platelet functional impairment. Platelet
aggregation studies involve the use of a number of different agonists.
Although a relatively straightforward technique, platelet aggregation is not
available in most laboratories, probably because of the observer-dependent
nature of the test.
166 As thrombin generation proceeds, the body has natural anticoagulant
systems opposing further thrombus formation. Natural anticoagulants
include:
a. Tissue plasminogen activator (TPA)
b. Antithrombin III
c. Activated protein C
d. Heparin cofactor II

Answer: b, c, d
Just as thrombin generation is the key to coagulation, antithrombin III is the
most central anticoagulant proteins. This glycoprotein binds to thrombin,
preventing its removal of fibrinoprotein A and B from fibrinogen, prevents
the activation of factor V and VIII and the activation and aggregation of
platelets. The second line of defense is the activated protein C, which
inactivates factors Va and VIIIa. This inactivation reduces the ability of the
prothrombinase complex to accelerate the rate of thrombin formation. A third
natural anticoagulant is heparin cofactor II. Its concentration in plasma is
estimated to be some four-fold lower than antithrombin III, and its action is
primarily implicated in the regulation of thrombin formation in extravascular
tissues. Tissue plasminogen activator (TPA) is a natural catalyst for the
activation of plasminogen to plasmin, the main fibrinolytic enzyme in the
body. Therefore, TPA is part of the fibrinolytic system rather than a natural
anticoagulant.
167 Infectious disease transmission during blood transfusions is of clinical
significance to surgeons and of major importance to patients contemplating
surgery potentially associated with the need for blood administration. Which
of the following statement(s) is/are true concerning the transmission of
infectious disease during blood transfusions?
a. Post-transfusion hepatitis is usually due to hepatitis B
b. Hepatitis and HIV transmission is greatest with the administration of
pooled plasma products such as serum albumin
c. The most important cause of post-transfusion disease in immunosuppressed
patients is CMV infection
d. The risk of HIV transmission in blood transfusions is significantly less
than the risk of hepatitis transmission
Answer: c, d
The most common infectious diseases transmitted during blood transfusions

include viral hepatitis, CMV, and HIV infection. Post-transfusion hepatitis in


90% of cases consists of non-A, non-B hepatitis known as hepatitis C. All
blood products except for immune serum globulin and albumin can carry and
transmit this form of hepatitis. Because heat treatment eliminates the risk of
viral transmission, products from pooled plasma that are heat treated such as
albumin are not at risk for HIV or hepatitis transmission. CMV transmission
exists in three formsprimary, reinfection, and reactivation. Primary
exposure results in an IgM response to the virus. Reactivation is most
commonly related to pregnancy, transplantation, and immunosuppression,
and is the most important cause of post-transfusion disease accompanying
immunosuppression of patients. Although the risk of the public concern for
transmission of HIV disease associated with blood transfusions has
significantly outweighed other infectious disease transmission, the risks of
HIV transmission is markedly less than that of hepatitis.
168 There are a number of hypercoaguable states which can be associated
with arterial or venous thrombosis and embolic phenomenon. These include:
a. Heparin-associated thrombocytopenia
b. Antithrombin III deficiency
c. Von Willebrand disease
d. Vitamin C deficiency
Answer: a, b
A number of hypercoaguable states are present. These include heparinassociated thrombocytopenia in which a heparin-dependent platelet antibody
causes aggregation of platelets when the patient is exposed to heparin.
Activation of platelets in this setting results in thrombocytopenia, thrombosis,
and embolic episodes. Antithrombin III deficiency accounts for about 2% of
venous thrombotic events and has been described in pulmonary embolism,
mesenteric venous thrombosis, lower extremity venous thrombosis, arterial
thrombosis, and dialysis fistula failure. Von Willebrands disease is a
hereditary complex coagulation factor deficiency which is manifested by a

reduction of factor VIII activity, and the Von Willebrand factor which is an
adhesive protein that mediates platelet adhesion to collagen. Severe vitamin
C deficiency results in a disorder in soft tissue increasing vascular
permeability and fragility resulting in the potential for bleeding disorders.
169 Cytokines with clearly defined actions in acute inflammation and early
tissue injury include which of the following?
a. Cysteine-X-Cysteine (C-X-C) chemokines
b. Tumor Necrosis Factor (TNFa)
c. Transforming Growth Factor-b (TGF-b)
d. Interleukin-6 (IL-6)
e. Platelet Derived Growth Factor (PDGF)
Answer: a, b, c, d, e
Polypeptide mediators, such as TNFa and IL-1, are considered early
response cytokines and are actively involved in the initiation of the cascade
of events which precipitate acute inflammation. In addition to being
important triggers for the induction of other cytokines important
inflammatory network, TNFa and IL-1 appear to be key mediators in
promoting the adherence of inflammatory cells to the endothelium. IL-1 is a
complex, multifunctional molecule that shares many overlapping biological
properties with TNFa. In addition, both IL-1 and TNFa potentiate the effects
of one another. The most important function of IL-6 appears to be the
regulation of the hepatic acute phase response. Following injury, a number of
physiologic changes develop within several hours. IL-6 is one of the primary
stimuli for the production of acute phase proteins from the liver. Endotoxin,
IL-1, TNFa and PDGF are capable of causing significant induction of IL-6
synthesis.
Over the last decade, at least 12 different C-X-C chemokines have been
identified. These include IL-8, one of the most potent mediators of
chemotaxis known. TNFa and IL-1 are key molecules for the induction of IL8, which in turn is important for the induction of neutrophil recruitment and

activation.
Similar properties are apparent for other members of this chemokine family.
Platelet activation and degranulation occur during coagulation following
injury, leading to the deposition of a number of cytokines into the provisional
matrix. These cytokines include transforming growth factor-a, (TGFa),
transforming growth factor b (TGF-b), platelet-derived growth factor
(PDGF), and neutrophil activating peptide-2 (NAP-2). These cytokines are
either important growth factors or chemotaxis for leukocytes, endothelial
cells, fibroblasts, and keratinocytes which are key components in the process
of tissue repair. Thus, coagulation and platelet activation provide the initial
foundation for subsequent cellular recruitment.
170 Which of the following statements regarding transforming growth factor
b (TGF-b) are true?
a. TGF-b expression is autoregulated
b. TGF-b enhances collagen synthesis
c. TGF-b inhibits extracellular matrix production
d. TGF-b may inhibit or promote cellular proliferation
Answer: a, b, d
TGF-b appears to be one of the key cytokines in control of tissue repair.
TGF-b is strongly chemotactic for neutrophils, T cells, monocytes, and
fibroblasts. TGF-b activates inflammatory cells to elaborate fibroblast growth
factor, TNFa, IL-1 and increase their synthesis of extracellular matrix
proteins. TGF-b also induces both the infiltrating cells and resident cells to
produce more TGF-b. This auto-induction amplifies its biological effects at
the site of injury and may play an important role in the development of
chronic fibrosis in a variety of pathologic states. TGF-b enhances collagen
synthesis as well. Lastly, TGF-b may function as a mitogen or growth
inhibitor for a wide variety of cell types, including selected cell types of
mesenchymal origin. Whether TGF-b stimulates or inhibits proliferation
depends on the presence of other growth factors, the concentration of TGF-b,

and the cell density. Thus, at low doses, TGF-b stimulates the proliferation of
densely plated human marrow fibroblasts, but is inhibitory at high
concentrations.
171 Leukocyte activation and adhesion to vascular endothelial cells is a
critical step in the inflammatory process. This process is regulated by which
of the following molecules?
a. The selectins
b. The b5 integrins
c. The immunoglobulin supergene family
d. Nitric oxide
e. IL-8
Answer: a, c, d, e
The temporal events that initiate and propagate neutrophil recruitment and
inflammation include endothelial cell activation and expression of
endothelial-derived neutrophil adhesion molecules, neutrophil-endothelial
cell adherence, and neutrophil transendothelial migration via established
neutrophil chemotactic gradients. There are three major families of adhesion
molecules which are expressed on the surface of leukocytes and endothelial
cells and are important for leukocyte-endothelial cell interactions. These
include the immunoglobin supergene family (ICAM-1, VCAM-1, and
PECAM-1), the selectins (E-selectin, P-selectin and L-selectin), and the
integrins. The leukocyte b2 integrin adhesion molecule family consists of
three members with heterodimeric glycoproteins displayed as a variable alpha
and a constant beta chain. Nitric oxide regulates the adhesion process both by
direct influence on leukocyte binding as well as by regulation of regional
blood flow. IL-8 is one of the most potent mediators of chemotaxis in the CX-C chemokine family. It serves an important role in neutrophil recruitment
and activation, and the continued propagation of the inflammatory response.
172 A 65-year old patient has colon carcinoma metastatic to the liver and

lungs. He has had a weight loss of 10 kg. Cytokine-dependent tumor cachexia


is attributable to which of the following?
a. Increased glucose uptake and increased glycogen breakdown occur in this
circumstance.
b. Suppressed activity of lipoprotein lipase results from TNFa
c. TNFa stimulates lipolysis
d. The differentiation process of pre-adipocytes is impaired
e. Partial reversal of differentiated adipocytes to pre-adipocyte morphology
and gene expression occurs
Answer: a, b, c, d, e
Tumor cachexia appears to be mediated by TNFa. Lipopolysaccharide (LPS),
as well as other cytokines, activate a variety of inflammatory cells, most
importantly macrophages, to produce TNFa. Both the chronic administration
of TNFa to rats and implantation of tumors secreting TNFa in mice induce a
syndrome of cachexia. In vitro, higher TNFa concentrations alter glucose
metabolism in cultured myotubules by increasing glucose uptake and
glycogen breakdown. It has also been demonstrated that purified TNFa
suppresses lipoprotein lipase activity and stimulates lipolysis in cultured
adipocytes. Further, TNFa not only inhibits the differentiation process of
preadipocytes, but partially reverses differentiated adipocytes to a
preadipocyte morphology and pattern of gene expression. All of these
metabolic effects at least partially explain the chronic syndromes of anorexia,
weight loss, and cachexia that are associated with both chronic infection and
malignancy.
173 Which of the following statements regarding fibroblasts and their
function in wound healing are true?
a. IL-1 has both inhibitory and promotional effects on fibroblast growth
b. TNFa stimulates fibroblast collagen synthesis
c. IL-1 and TNFa have opposite effects on the healing of bone
d. In human clinical trials, EGF (epithelial growth factor) has been

demonstrated to accelerate epidermal regeneration in cutaneous wounds


Answer: a, d
IL-1 appears to be important in the process of normal wound repair. IL-1 has
been shown to stimulate skin fibroblast and keratinocyte growth, as well as
fibroblast collagen synthesis and keratinocyte chemotaxis. IL-1 also promotes
increased transcription of the matrix degradative enzymes collagenase and
stromelysin. These are important and potent tissue degrading proteinases.
Other studies have demonstrated that IL-1 inhibits fibroblast growth and
matrix synthesis, and stimulates collagenase production. These actions are at
least partly due to the ability of IL-1 to upregulate prostaglandin E2
production which results in the down regulation of matrix synthesis. IL-1 has
both promoting and inhibiting effects on fibroblast collagen synthesis,
therefore, the overall activity in this area is somewhat unclear in comparison
to other well-defined fibroblast growth-promoting cytokines. TNFa inhibits
fibroblast collagen synthesis, however it also has potent mitogenic effects.
The mitogenic response correlates well with an increased stimulation of
tyrosine phosphorylation. Both IL-1 and TNFa have similar effects upon
bone. Both stimulate cartilage resorption, the release of proteoglycans from
cartilage by limited proteolytic degradation, and both inhibit proteoglycan
synthesis. Recent studies have also demonstrated that TNFa inhibits fracture
healing in experimental animals. This is due to the inhibition of cartilage
formation and new bone synthesis, and the inhibition of mesenchymal cell
differentiation into chondroblasts. The family of epithelial growth factor
(EGF)-like molecules induce mitogenesis and play a role in wound healing.
In human clinical trials, EGF has been demonstrated to accelerate epidermal
regeneration in cutaneous wounds. In vitro data show that recombinant EGF
enhances keratinocyte migration. EGF is also a potent chemoattractant for
granulation tissue fibroblasts.
174 Neutrophil chemotaxis is a fundamental aspect of inflammatory injury in
conditions such as the Adult Respiratory Distress Syndrome (ARDS).

Neutrophil chemotaxis is directly attributable to which of the following


molecules?
a. C5a
b. TNFa
c. LPS
d. IL-1
e. ENA-78 (Epithelial Neutrophil Activating Protein)
Answer: a, e
There is a large collection of peptide, polypeptide and lipid mediators which
have chemotactic properties. Although TNF a, IL-1 and LPS were initially
reported to have direct neutrophil chemotactic activity, recent studies have
demonstrated that these molecules are not directly chemotactic for
neutrophils. This finding suggests that cytokine networks may be operative in
vivo and depend on the initial expression of early response cytokines. This
initial interaction is followed by the generation of more distal inflammatory
mediators that directly influence neutrophil chemotaxis and activation. There
is a particularly important group of novel chemotactic cytokines which share
significant homology with the presence of four conserved cysteine amino
acid residues. These cytokines in their monomeric forms are all less than 10
kD, are characteristically basic heparin-binding proteins, have specific
neutrophil chemotactic activity and display four highly conserved cysteine
amino acid residues, with the first two cysteines separated by one nonconserved amino acid residue. Because of their chemotactic properties and
the presence of C-X-C cysteine motif, these have been designated the C-X-C
chemokine family. Twelve different chemokines have been identified in the
last decade. These include IL-8, epithelial neutrophil activating protein
(ENA-78), and others. Among the other endogenous chemoattractants are
several complement-derived peptides. Perhaps, the most potent of these is the
short-lived C5a peptide.
175 Which of the following statements regarding angiogenesis are true?

a. Angiogenesis is a seminal biologic event with clinical relevance limited to


its effect upon tumor growth
b. C-X-C chemokines regulate angiogenesis
c. PF-4 has angiogenic properties
d. Sites of atherosclerosis demonstrate chronic angiogenic activity
Answer: b, d
An important component of tissue repair and wound healing is the process of
angiogenesis. This normal, physiologic process is a local, transient event
which is regulated strictly. A biological imbalance in the production of
angiogenic and angiostatic factors contributes to the pathogenesis of several
angiogenesis-dependent disorders. These include both malignant and
nonmalignant disorders such as rheumatoid arthritis, scleroderma, psoriasis,
atherosclerosis, and idiopathic pulmonary fibrosis. Persistent
neovascularization in these benign disorders is a prerequisite for the
perpetuation of fibroproliferation. IL-8 and potentially other C-X-C
chemokines are involved with the angiogenesis process. IL-8 is a potent
angiogenic factor. In contrast, another member of the C-X-C chemokine
family, PF-4 has angiostatic properties. This suggests that the C-X-C
chemokines may function as either angiostatic or angiogenic factors, and the
biologic balance that is maintained between these factors may govern overall
angiogenic potential in a variety of physiological and pathophysiological
states.
176 Which of the following statements regarding IL-1 are correct?
a. While IL-1 and TNFa share many biologic effects, IL-1 appears to be more
potent
b. IL-1 expression is in part autoregulated
c. IL-1 inhibits prostaglandin production
d. The ability of IL-1 to upregulate endothelial cell-neutrophil adhesion
molecules is relatively limited
Answer: b

IL-1 and TNFa share many biologic properties. In addition, each potentiates
the effects of the other one when given concurrently. Overall, IL-1 alone
probably has weaker effects than TNFa with respect to the induction of
shock; its role is likely to be important with respect to its marked potentiating
abilities as it relates to TNFa. IL-1 expression is regulated by a host of factors
including IL-2, granulocyte macrophage colony stimulating factor (GMCSF), transforming growth factor b (TGF-b), TNFa, all of the interferons,
and IL-1 itself. Other endogenous stimuli for IL-1 production include
antigen-antibody complex, the Fc region of IgG, and C5a; other nonspecific
exogenous stimuli include silica particles and UV irradiation.
One of the key proinflammatory features of IL-1-induced inflammation is the
stimulation of arachadonic acid metabolism. IL-1 stimulates the release of
pituitary stress hormones and increases the synthesis of collagenases,
resulting in the destruction of cartilage, bone and other collagen-rich
structures. IL-1 stimulates prostaglandin production.
One of the most important properties of IL-1 involves its interaction with the
vascular endothelium. This includes the adherence of neutrophils, basophils,
eosinophils, monocytes, and lymphocytes to the vascular endothelium via
interaction between adhesion molecules on leukocytes and adhesion-receptor
complex on the endothelial cells. By inducing the expression of ICAM-1, Eselectin, and VCAM-1 on endothelial cells, IL-1 provides a key step in the
extravasation of leukocytes to sites of local inflammation and injury.
177 Which of the following statements regarding TNFa are true?
a. TNFa has a marked procoagulant effect
b. Passive immunization of patients with neutralizing antibodies to TNFa
improves survival from multi-organ system failure
c. TNFa upregulates E-selectin expression
d. The most potent known stimulus for TNFa production and release is IL-1
Answer: a, c

TNFa has a marked procoagulant effect on endothelial cells, precipitating


intravascular thrombosis. TNFa causes endothelial cells to release
procoagulant activity (tissue factor), platelet activating factor, and von
Willebrand factor, all of which favor thrombosis. TNFa also down regulates
the expression of thrombomodulin, which has the potential to block the
assembly of protein C and protein S complexes, further decreasing the
anticoagulant properties of the endothelial cell surfaces.
Administration of recombinant TNFa to experimental animals produces a
clinical syndrome similar to that seen in septic shock and multi-organ system
failure in humans. Passive immunization of animals with neutralizing
antibodies against TNFa, prior to the infusion of TNFa or endotoxin, has been
shown to prevent the development of this syndrome. No such evidence exists
in human patients.
TNFa upregulates a variety of leukocytic adhesion molecules including
ICAM-1, PECAM-1, VCAM-1, E-selectin and P-selectin. A variety of
exogenous and endogenous factors (including IL-1) are capable of inducing
cells to produce TNFa, however the most potent stimulus for TNFa
production and release is endotoxin.
178 Which of the following belong to the family of C-X-C chemokines?
a. IL-8
b. IL-10
c. Growth Related Oncogene-a
d. Leukotreine B4
e. b Thromboglobulin
Answer: a, c, e
A particularly important group of novel chemotactic cytokines has been
elucidated over the last decade. Twelve are known and are listed below.
C-X-C Chemokines
Connective Tissue Activating Protein III
b-Thromboglobulin

Growth Related Oncogene-a


Growth Related Oncogene-b
Related Oncogene-g
Interleukin-8
Epithelial Neutrophil Activating Protein
Granulocyte Chemotactic Protein-2
Platelet Factor-4
g-Interferon-inducible Protein
Monokine-induced by g-Interferon
Each has unique biologic functions. There appear to be important in vivo
cytokine networks involving these molecules which regulate chemotaxis, and
other fundamental aspects of inflammation.
179 Which of the following statements regarding the complement system are
true?
a. Complement activation yields products which are directly cytotoxic as well
as products which act indirectly via activated leukocytes
b. Complement products referred to as anaphylatoxins include C1, C3a, C4a,
and C5a
c. The principal role of C5a is in bacterial opsonization
d. The alternative and classical pathways converge proximal to generating the
membrane attack complex (C5b-9)
Answer: a, d
The complement system is composed of two different but linked sequences,
the classic and alternative pathways. The pathways involve serum proteins
that act to amplify the inflammatory-immune response as well as to directly
mediate tissue injury. Complement activation by either pathway has been
associated with a cascade of events, some of which are mediated directly at a
physiologic level by complement products and some of which occur
indirectly via activated leukocytes. The direct physiologic effects mediated
by C3a and C5a, and to a lesser extent C4a, include increased vascular

permeability and contraction of smooth muscle. These are key elements of


anaphylaxis. C1 is not an anaphylatoxin as it is the initial complement
component which binds to antigen-antibody complex to initiate classical
pathway activation. C5a acts principally to alter the behavioral characteristics
of leukocytes. Effects include enhanced adherence, enhanced chemotactic
activity, release of proteinases, and production of toxic metabolites of
oxygen. C3, on the other hand, plays a key role in bacterial opsonization,
resulting in enhanced phagocytosis of invading microorganisms. The
alternative and classical complement pathways converge at the C5 level
proximal to generating the membrane attack complex (C5b-9) (Figure 6-3).
180 Which of the following statements regarding neutrophils are true?
a. The neutrophil undergoes final maturation after release into the circulation
b. Patients with chronic granulomatous disease have a defective neutrophil Hoxidase system
c. Neutrophil killing of bacteria is achieved by oxidants, proteinases and
cationic proteins
d. The normal human neutrophil circulates in the blood for 710 days
Answer: b, c
The neutrophil is a migratory phagocytic cell that defends the host against
bacteria and eliminates necrotic tissue. The neutrophil matures in the bone
marrow and is released into the circulation as a fully differentiated cell. It is
loaded with granules containing a variety of proteinases, hydrolases,
antimicrobial agents and cationic proteins. The cell phagocytoses material
and the granules fuse with the phagocytic vacuoles to degrade the foreign
material. When the cells are challenged with a large amount of material, the
granule contents may be released into the extracellular space where damage
to surrounding tissue occurs. The neutrophil normally circulates in the human
bloodstream for 7 to 10 hours. Thereafter, neutrophils are thought to exist for
1 to 2 days in the tissues before being cleared from the system. Granule
constituents are formed during differentiation and replenishment of spent

granules does not occur once the cells are in the circulation. Hence, the
neutrophil is a fully differentiated end-cell poised to respond rapidly to
stimuli, but it is rapidly spent in the process. Neutrophils have a NADPHoxidase enzyme system on the plasma membrane which can be activated to
produce toxic oxygen species including the superoxide anion (02). Patients
with chronic granulomatous disease (CGD) have a defective NADPHoxidase system in their neutrophils, and are thus unable to generate 02.
Although neutrophils from patients with CGD are able to phagocytose
bacteria, they are unable to kill the intracellular microbes and chronic,
unresolved infections result.
181 Which of the following statements regarding the alternative complement
pathway are true?
a. C1, C4 and C2 are involved
b. NH3 apparently activates complement via this pathway
c. Factors B and D are involved
d. Endotoxin activates complement via the alternative pathway
Answer: b, c, d
The alternative pathway differs from the classic pathway in that the first steps
involving C1, C4 and C2 are bypassed. (See Figure 6-3 previously
reproduced.) This pathway can be directly activated by agents other than
antigenantibody complex (e.g., complex polysaccharides like endotoxin and
zymosan). Other serum protein factors (e.g., factors B and D) are involved in
the activation sequence. Ammonia can attack the thiol-ester, producing
amidated C3 and activate the alternative pathway. This leads to membrane
attack complex formation (C5b-9) and activation of a number of phagocytic
cell functions including toxic oxidant production. This phenomenon may
have relevance to several in vivo disease states. In animal models of renal
failure, elevated levels of renal vein NH3 have been correlated with impaired
renal function and the presence of complement components at the sites of
renal injury.

182 Platelet activating factor is:


a. Generated by the action of phospholipase A2 on membrane phospholipids
b. Antiinflammatory in most of its actions
c. Synthesized by endothelial and other cells
d. Exerts a variety of biologic effects which are platelet-independent
Answer: a, c, d
Like the eicosanoids, platelet-activating factor (PAF) is not stored in cells but
is rapidly produced during inflammation. PAF exerts a variety of biologic
effects that are platelet-independent. The synthesis of PAF is initiated by the
activation of phospholipase A2. Activation of phospholipase A2 releases
arachidonic acid in addition to lyso-PAF. Hence, PAF synthesis and
eicosanoid production are coordinately regulated. PAF is synthesized on
activation of a variety of inflammatory cells including platelets, neutrophils,
basophils, mast cells, mononuclear phagocytes, eosinophils and vascular
endothelium. PAF is a stimulatory agonist for many inflammatory cells, as
well as for smooth muscle cells, vascular endothelium and others. PAF
enhances the ability of neutrophils to respond to challenge with Nformylpeptides and LTB4. There is considerable overlap and redundancy in
the effects produced by PAF and eicosanoids.
183 Platelets have a wide array of functions in inflammation. Which of the
following are among these?
a. Synthesis and release of vasoactive eicosanoids
b. Release of chemotactic factors
c. Adherence to and coating of bacterial and tumor cells
d. Increase of vascular permeability
e. Phagocytosis of bacteria
Answer: a, b, c, d
Platelets are anucleated cells derived from megakaryocytes in the bone

marrow. Their central role in hemostasis is well known. Platelets possess a


wide array of functions in inflammation, including the following:
Synthesis and release of vasoactive eicosanoids
Release of chemotactic factors
Interaction with other inflammatory cells
Interaction with endothelial cells
Adherence to and coating of bacterial and tumor cells
Platelets are not capable of phagocytosis.
Few of the factors released or the functions carried by platelets during
inflammation are unique to this cell type. Other inflammatory cells often
have the same or similar capabilities. Indeed, some platelet functions may
reflect vestigial functions inherited from a primitive precursor inflammatory
cell. Platelets serve primarily as an amplifier or modulator of the
inflammatory response.
184 Eicosanoids mediate inflammation in a variety of ways. Of the following
statements, which are true with regard to this?
a. Eicosanoids are stored in cytoplasmic granules for release after receptor
mediated signaling
b. Eicosanoids include prostaglandins, thromboxanes, leukotrienes and
lipoxins
c. Eicosanoids generally have a plasma half-life measured in hours
d. Physiologic responses to eicosanoids include vasodilatation,
vasoconstriction, increased vascular permeability and both chemotaxis and
chemoattractant inhibition
Answer: b, d
The eicosanoids are derived from arachidonic acid (eicosatetraenoic acid) and
consist of prostaglandins, thromboxanes, leukotrienes and lipoxins. The
eicosanoids are not stored in cells but are rapidly synthesized by cells in
response to a variety of stimuli. They have potent effects on vascular and
bronchial smooth muscle including vasodilatation, vasoconstriction,

bronchodilation and bronchoconstriction. In addition, they directly regulate


vascular permeability. LTB4 is a potent, neutrophil chemoattractant whereas
lipoxin A4 inhibits other chemoattractants. It appears that eicosanoids are
important regulators of the endogenous inflammatory response. The rapid
destruction of eicosanoids in the circulation limits their role primarily to that
of mediators of local inflammatory changes. The local effects can be
substantial. In general, the eicosanoids are rapidly metabolized or are so
chemically unstable that they primarily exert their effects near the site of
synthesis. Arachidonic acid does not exist in cells but is esterified to
membrane phospholipids. Thus, the first step in the production of eicosanoids
is phospholipase action, which liberates arachidonic acid. (Figure 6-6)
185 Which of the following statements are true?
a. Eosinophils are the major, if not sole, source of histamine in the blood
b. Basophils are effector cells in allergic reactions by virtue of IgE receptors
c. Mast cells are the major source of tissue histamine except in the stomach
and central nervous system
d. Mononuclear phagocytes release a variety of proinflammatory cytokines
and growth factors
Answer: b, c, d
Eosinophils constitute 1% to 3% of the leukocyte population of the
bloodstream. They also reside in tissues and they exhibit phagocytic
capabilities. They are less effective as bactericidal cells than neutrophils, but
play a major role in defense against parasites. Eosinophils are primary
effectors in allergic reactions by virtue of IgE receptors (which are not found
on neutrophils).
Basophils are fully differentiated cells released into the circulation from bone
marrow. Basophils are the major, if not sole, source of histamine in the blood.
Histamine is a vasoactive amine and the major mediator of the IgE-mediated
immediate hypersensitivity response. Histamine release from basophils is
induced by complement products as well as by IgE receptors.

Mast cells are formed from bone marrow precursors that differentiate and
proliferate in connective tissue. Mast cell granules contain histamine and
proteoglycans. They represent the major source of histamine in most tissues
except the stomach and central nervous system.
The monocytemacrophage system consists of phagocytic cells scattered
throughout the body. During acute inflammation, monocytes respond to
chemoattractants released and are recruited to the site of inflammation.
Mononuclear phagocytes respond to inflammatory stimuli by releasing MCSF, GM-CSF, IL-1, and TNF, in addition to a variety of growth factors.
These factors increase the production of mononuclear phagocytes and several
of these factors enhance the ability of effector cells to respond to chemotactic
stimuli released at the site of injury. Thus, the mononuclear phagocytes are
important in initiating and augmenting the cycle of events that result in
recruitment and activation of inflammatory cells at sites of inflammation.
186 Cellular injury from oxidants may be manifest by which of the
following?
a. Cell membrane lipid peroxidation
b. DNA strand breaks
c. Cytoskeletal disassembly
d. ATP depletion
Answer: a, b, c, d
Free oxygen radicals are chemical species that are intermediates in the
normal process of cellular respiration. Oxidants that are free radicals have
been implicated as initiators of reactions which lead to a variety of cellular
injuries. Oxidants are derived from several sources, notably phagocytes.
Among the effects of oxygen free radicals are membrane lipid peroxidation,
DNA strand breaks, cytoskeletal disassembly and inhibition of glucose
metabolism leading to decreased cellular ATP concentrations. (Figure 6-16)
187 Which of the following acute-phase protein levels are increased in

human plasma following acute inflammation?


a. C-reactive protein
b. Serum amyloid
c. a -Proteinase inhibitor
d. Fibrinogen
e. Albumin
Answer: a, b, c, d
The acute-phase response is a series of homeostatic responses of the
organism to tissue injury in infection and inflammation. After an
inflammatory stimulus occurs, a number of events occur within hours. These
reflect altered set-points for various physiologic parameters including
thermoregulation (fever), nitrogen balance (negative), and levels of various
plasma proteins (increased or decreased). The erythrocyte sedimentation rate,
which increases with inflammatory states, is an example of this phenomenon.
The increased sedimentation rate is due to increased levels of fibrinogen and
some of the other acute-phase reactants in plasma. Some proteins show a
large increase (about 1000-fold), some a 4-to 5-fold increase, and others
about a 50% increase over resting nonstressed levels.
Note that albumin is an acute-phase reactant. Levels of albumin drop after an
inflammatory stimulus, usually 30% to 50% of the level before injury. The
reason for the decrease in production is poorly understood.
188 Which of the following statements regarding endothelial cells in acute
inflammation are true?
a. Endothelial cells are characterized by phenotypic homogeneity
b. Specific patterns of receptor expression regulate leukocyte adherence
c. Endothelial cell nitric oxide generation regulates regional blood flow and
leukocyte adhesion
d. Endothelial cells may be capable of phagocytosis
Answer: b, c, d

Endothelial cells are increasingly recognized to be phenotypically


heterogeneous. Specific receptor molecules are expressed at various sites
where they help to direct lymphocytes and other leukocytes to their
appropriate target organ. In the high endothelial venues, these receptor
molecules are known as vascular addressing. Endothelial cells play a major
role in regulating vascular tone. This is the result of angiotensin-converting
enzyme on the cell surface as well as the production of both endothelia (a
potent vasoconstrictor) and nitric oxide (a potent vasodilator). Both play
important physiologic roles in determining the distribution of blood flow. In
addition, recent evidence suggests that NO may have direct effects upon the
expression of a variety of leukocyte adhesion molecules. Under unusual
circumstances, endothelial cells can exhibit macrophage-like properties in
that they can act as antigen-presenting cells and also phagocytose particles.
They may also be a significant source of oxidants in inflammatory reactions
after ischemic injury. Endothelial cells are not passive participants in
inflammatory processes; rather, they possess the ability to direct and focus
many aspects of an inflammatory event.
189 The first line of host defense is the barrier presented to the external
environment. Which of the following statement(s) is/are true concerning host
barriers?
a. Sebaceous glands secrete chemical compounds that maintain a relatively
high pH, providing effective bacterial stasis
b. Within the respiratory tract, ciliary function serves to extrude microorganis
trapped within the mucus secretion layer
c. The low pH within the stomach markedly decreases bacterial content of the upper
gastrointestinal tract

d. Gut peristalsis serves to prevent microbial adherence and invasion


Answer: b, c, d
The skin, mucus membranes, and epithelial layers of various organs of the
body constitute effective physical barriers against microbial invasion. In
certain portions of the body, these barriers have developed ancillary

adaptations to increase the effectiveness of the barrier functions. Skin


structures such as sebaceous glands secrete chemical compounds that serve to
maintain a relatively low pH, providing effective bacterial stasis. Mucus
secretion by specialized glands within the bronchi and gut provide a mucus
layer that represents a physical and chemical barrier to microbial invasion.
Within the respiratory tract, ciliary function serves to extrude microorganisms
trapped within this mucus layer. In the alimentary track, the very low pH
within the stomach and gut peristalsis both serve to prevent microbial
adherence and invasion.

190 Which of the following statement(s) is/are true concerning the antibody
response to an invading antigen?
a. All antibodies are composed of one type of heavy and one type of light
protein chain
b. The carboxyl terminus of the heavy chain is the antigen binding site
c. Antibody of the immunoglobulin G class is the initial antibody produced in
response to an antigenic stimulus
d. Immunoglobulins A, D, and E play an active role in the circulating
humoral response
Answer: a
Humoral defenses consist of antibody (immunoglobulin; Ig) and complement.
All Ig classes (IgM, IgG, IgA, IgE, IgD) and IgG subclasses are composed of
one type (M, G, A, E, D) of heavy and one type (K and g ) of light protein
chains that consist of several domains both structurally and functionally. Each
Ig molecule contains one or more units that consist of two heavy and two
light chains linked by disulfide bonds. The amino terminus of both heavy and
light chains contain several hypervariable regions that fold in three
dimensions to produce the antigen-binding site. The carboxyl terminus of the
heavy chains contain regions that activate complement and bind Fc receptors,
by which direct adherence to polymorphonuclear leukocytes and

macrophages take place after antigen binding occurs.


Initially, antibody of the IgM class is produced in response to an antigenic
stimulus. A second exposure to the same antigen, or a cross-reactive antigen,
leads to the so-called second set response, in which antibody of the IgG class
with two binding sites is produced more rapidly and in larger quantity
compared to the initial IgM primary response. Immunoglobulin of the IgA
class is secreted by gut-associated lymphoid tissue and is combined with
secretory components of protein to form a dimer termed secretory IgA. This
antibody acts at a variety of epithelial sites to prevent microbial adherence
and invasion. IgD and IgE exist in smaller amounts in the circulation and do
not appear to play a major role as host defense components.

191 Increasing evidence has implicated gram-negative bacterial


lipopolysaccharide (LPS endotoxin) as the portion of the gram-negative
bacterial cell membrane responsible for many, if not all the toxic effects that
occur during gram-negative bacterial sepsis. The following statement(s) is/are
true concerning LPS and the host response.
a. The LPS molecule can in itself cause physiologic responses similar to that
seen during gram-negative bacterial sepsis
b. LPS triggers host macrophages to release a variety of cytokines including
TNF-a, IL-1a, and IL-1b, IL-6, and IFN-a
c. Excessive cytokine production is not associated with detrimental
consequences
d. TNFa and IL-1b appear to be the primary mediators within the host,
exerting deleterious effects on the host when excessive amounts reach the
systemic circulation
Answer: a, b, d
The LPS molecule exerts diverse effects on the mammalian host.
Immunologic responses to LPS include nonspecific polyclonal B-cell
proliferation, macrophage activation and cytokine secretion, tolerance to

subsequent LPS or bacterial challenge, and production of antibody directed


against various portions of the LPS molecule after repeated challenge.
Physiologic responses similar to those seen during gram-negative bacterial
sepsis occur during LPS administration alone and include hypotension,
hypoxemia, acidosis, bacterial translocation across the gut, complement and
coagulation cascade activation, white blood cell and platelet margination, and
death. Indirect effects result from LPS-triggering of host macrophages.
Activated macrophages secrete a wide array of cytokines that include TNF-a,
IL-1a, and IL-1b, IL-6 and interferon-a (IFNa). Excessive secretion of
cytokines produce substantial systemic effects in the mammalian host. TNFa
and IL-1b appear to be the primary mediators within the local host milieu,
exerting deleterious effects on the host only after large amounts are secreted
and reach the systemic circulation.

192 Which of the following statement(s) concerning the gut microflora is/are
correct?
a. Gut microflora evolves constantly throughout development
b. The gut microflora can contribute to the physical and chemical barriers at
the mucus membrane level
c. Most of the microorganisms found in the oropharynx eventually pass into
the intestine
d. In the colon, anaerobic organisms outnumber aerobic organisms in a ratio
in excess of 100:1
Answer: b, d
The composition of the gut microflora is established in neonates after
ingestion of microbes that are acquired during contamination from the birth
canal and during initial feeding, and remain relatively constant thereafter.
Although this flora acts to promote development of the immune system, the
specific interactions that produce this effect have not been fully elucidated.
The microflora also contributes to physical and chemical barriers at the

mucus membrane level, in that many autochthonous microbes possess


adhesion proteins by which they can bind to certain areas of the mucosal cell
or to specific types of bacteria, occupying potential binding sites for
pathogenic organisms and producing a substantial physical mucobacterial
layer. The oropharynx contains a number of aerobic and anaerobic
microorganisms, however, these microbial inhabitants do not usually pass
into the intestine, because the stomach itself represents a significant barrier to
invading microorganisms by virtue of its low pH, which kills most microbes.
The upper small intestine contains few organisms, mainly gram-positive
aerobes and lactobacilli. Conversely, the lower small intestine contains a
large number of aerobes and anaerobic forms, especially in patients in whom
the ileocecal valve allows free backwash of cecal contents into the terminal
ileum. Within the colon, a wide diversity and a large number of facultative
and strict anaerobic isolates are present. Only a small number of aerobes are
present, these microbes being outnumbered 100300 to 1 by anaerobes.

193 The use of antibiotics can be based on either the clinical course of a
patient without the benefit of well-defined microbiologic data (empiric
therapy), or targeted at specific identified pathogens once sensitivity reports
are available (directed therapy). The following statement(s) is/are true
concerning these therapies.
a. The issue of toxic side effects of antibiotics is only important in dealing
with emperic therapy
b. Single agent therapy is generally inferior to specific multi-drug therapy
(aminoglycoside plus an antianaerobic agent) for the treatment of secondary
bacterial peritonitis due to appendicitis, diverticulitis, penetrating
gastrointestinal injury, or anastomotic leak
c. With the empiric use of antibiotics, a diligent search for the septic source
should be undertaken and continued until identified
d. In clinical situations in which polymicrobial infection is identified,
specifically-directed treatment for the predominant organism is satisfactory

Answer: c
The use of empiric therapy without the benefit of well-defined microbiologic
data is appropriate when there is sufficient clinical evidence to support the
diagnosis such that it would be imprudent to withhold antimicrobial therapy.
In this setting, however, a diligent search for the septic focus source should
be undertaken and continued (cultures, radiographic procedures, etc.), and
initial limits should be placed in the course of empiric therapy with continued
reevaluation based on the clinical course of the patient. The choice of
antibiotic agents should be based on the clinical situation and known activity
patterns within the given institution. Single broad-spectrum agents, although
suffering slightly from a lack of individual pathogen specificity, are useful in
this setting in that they provide a broad coverage against several groups of
pathogens and may avoid some of the toxic effects with specific combined
modality regimens. Similarly, for directed therapy, single-agent therapy has
been demonstrated to be equivalent to combined therapy and should be
chosen in an attempt to select agents with appropriate sensitivities which
retain suitable clinical efficacy but exhibit minimal toxicity. After review of
cultural reports, many patients have demonstrated polymicrobial infection.
Because experimental clinical evidence supports the concept of aerobicanaerobic synergy, therapy should be directed against all potential
components of the infection if the body site is such that these microorganisms
may be present.

194 The following statement(s) is/are true concerning newer detection


methods of systemic infection.
a. Enzyme-linked immunosorbent assay (ELISA) is a rapid immunologic
assay used for both antigen and antibody detection
b. Southern, northern, and western immunoblot techniques are used to detect
DNA, RNA, or proteins, respectively
c. Polymerase chain reaction (PCR) is a sensitive assay used to detect small

amounts of microbial DNA, thus detecting infection at its early stages


d. Infectious agents currently detected using advanced molecular techniques
include cytomegalovirus (CMV) and human immunodeficiency virus (HIV)
Answer: a, b, c, d
Although the classic detection of infection based on clinical signs of infection
and bacterial culture remain the most common clinical tools, increasing
reliance has been placed on assays that do not employ cultural data.
Specifically, the antibody and cytokine host responses are being intensely
examined and extremely sensitive amplified assays that rely on antigen,
antibody or microbial DNA detection are employed in the clinical setting.
Enzyme-linked immunosorbent assay (ELISA) is a rapid, antigen-based,
immunologic assay that can be used for both antigen and antibody detection,
for determination of antibody titer, as well as for screening for monoclonal
antibody production. Transblot techniques are being used increasingly in the
clinical setting. These include southern, northern, and western
immunotransblot techniques used to detect DNA, RNA, or proteins,
respectively. The polymerase chain reaction (PCR) is being used in some
centers as a sensitive assay to detect small amounts of microbial DNA. This
technique involves extraction of the DNA from the test sample with in vitro
amplification through repeated nucleic acid denaturing and polymerization so
that the gene copy number increases exponentially. This marked
amplification of the gene copy number results in extremely sensitive tests
which can detect infection at its early stages.
Clinically, these detection methods are being used to detect a wide variety of
infectious agents including CMV and HIV. Furthermore, preliminary
investigations into possible detection of fungal pathogens are underway.

195 Cytokines are low-molecular-weight polypeptides exerting a wide variety


of biologic effects at both local and systemic levels. Which of the following
statement(s) is/are true concerning the production and actions of cytokines?

a. Cytokines are produced solely by macrophages


b. Cytokines act only on other cells within the same local environment
c. Cytokines may have both protective and deleterious effects on the host
d. Each specific cytokine is produced by a single cell type
Answer: c
Macrophages, endothelial cells, lymphocytes, and other cells secrete a large
number of different compounds, termed cytokines, that are most probably
evolved for the purpose of local intercellular and intracellular signaling.
Cytokines frequently are secreted after initial lymphocyte or macrophage
activation, and may act on the secreting cell itself (autocrine activation) or on
other cells within the same local environment (paracrine activation) to cause
increased secretion of the same cytokine or other cytokines, respectively.
Some cytokines are produced by several cell types, and most produce a wide
array of effects. The duality of the effects of the cytokine component of host
defenses, exerting both salutatory and deleterious effects on the host, has
become increasingly evident.

196 The following statement(s) is/are true concerning cellular defense


mechanisms.
a. Macrophages function solely as antigen processing cells in the initial
reaction to exposure to an antigen
b. Macrophages may become activated and secrete cytokines
c. Macrophages serve as phagocytic cells in the tissues but not within the
bloodstream
d. Polymorphonucleocytes (PMNS) are normally present in only small
numbers within the tissue and enter an area of infection through diapedesis
Answer: b, d
A wide variety of cell types serve to provide host defense at several levels.
Macrophages act as the initial antigen processing cell that serves to present

antigen to help T cells, thus initiating the immune response. Macrophages,


however, are pluripotent cells that, in the process of engulfing and processing
antigen, may become activated. Activated macrophages secrete a variety of
cytokines. Macrophages also act as phagocytic cells in the tissues and within
the bloodstream, and because of their resident nature in many tissues, also
represent the first line of host defenses in many areas of the body, even before
activation. PMNS are present within the bloodstream, but only in small
numbers within the tissue, and enter an area of infection through diapedesis
after chemotactic stimuli are excluded by macrophages, bacterial breakdown
products, and complement activation.

197 A diabetic develops a severe perineal infection with skin necrosis,


subcutaneous crepitance, and drainage of a thin, watery, grayish and foulsmelling fluid. Management should consist of:
a. Gram stain of the fluid, which will likely demonstrate multiple bacteria
including predominantly gram-positive rods
b. A CT scan is indicated in a stable patient to define the extent of the disease
c. Broad spectrum antibiotics followed with prompt extensive debridement is
indicated
d. A safe guideline is to resect infected necrotic tissue so that a several
centimeter margin of grossly normal, healthy tissue can be achieved
e. A colostomy is of little benefit in this situation
Answer: a, b, c, d
The presence of severe perineal infection (referred to as Fournier gangrene
when this process involves the perineum and scrotum in males) is associated
with a continued high mortality despite aggressive and appropriate therapy.
The clinical description provided would suggest an underlying soft tissue
necrosis. In a stable patient radiologic studies including a CT scan to define
the extent of the disease and the presence of pelvic infection is indicated.
Gram stain will likely show evidence of polymicrobial organisms but the

presence of Clostridia marked by gram-positive rods would suggest


involvement with this organism. Prompt, aggressive and extensive
debridement to remove all devitalized and affected tissue and the addition of
broad spectrum antibiotics, fluid resuscitation, hemodynamic monitoring, and
nutritional support would appear to afford the patient the best chance of
survival. The clearest guidelines to determine the limits of resection involve
removal of clearly infected, necrotic tissue so that margins several
centimeters into grossly normal, healthy tissue are achieved. Because the
entire perineal region and buttocks are frequently involved in these patients,
performance of a fecal stream diversion by means of a colostomy often
provides improved wound care and patient management, although it is not
invariably a positive outcome.

198 The use of prophylactic antibiotics has become commonplace. Which of


the following statement(s) is/are true concerning the prophylactic use of
antibiotics?
a. The appropriate use of prophylactic antibiotics must include the initiation
of the agent prior to the surgical procedure
b. Continuing the antibiotic into the postoperative period has led to improved
results in antibiotic prophylaxis
c. The prophylactic administration of broad-spectrum agents (third-generation
cephalosporins) has been shown to be particularly advantageous
d. The topical use of antimicrobial agents is of no advantage in the
prophylactic setting
Answer: a
Intravenous administration of an antibiotic is clearly indicated for patients
undergoing clean contaminated operations. These antibiotics should be
administered prior to surgery to obtain adequate tissue levels at the time of
potential contamination. However, there has been no added benefit
demonstrated for the postoperative use of antibiotics with regard to

prophylaxis. The choice of antibiotic is a complex issue which remains


unresolved largely because both superficial and deep wound infections can
occur as a result of either or both skin (superficial wound) flora (e.g.,
Staphylococcus aureus) and body site (deep wound) infection. For this
reason, the administration of agents which possess activity directed against
these expected agents is reasonable. Although administration of a firstgeneration cephalosporin is acceptable, second-generation cephalosporins or
extended-spectrum penicillins with gram-positive and gram-negative activity
and biliary tract excretion may be more suitable for patients undergoing
gastrointestinal or biliary tract procedures. Similarly, the use of agents with
additional anaerobic activity for patients undergoing gastrointestinal
procedures involving the small bowel or colon should be considered. The
administration of broad-spectrum agents such as third-generation
cephalosporins for prophylaxis does not seem to provide additional benefit in
comparison to the above-mentioned type antibiotics and may foster the
development of resistant organisms within a given institution or
superinfection within a given patient. There is evidence that in some cases the
topical use of antimicrobial agents is equivalent to the administration of
intravenous antimicrobial agent antibiotics.

199 If a necrotizing soft tissue infection is considered, therapy mandates:


a. Empiric administration of antibiotics active against gram-positive, gramnegative, and anaerobic bacteria
b. Due to usually resistant species, penicillin is not indicated
c. Immediate operative intervention and aggressive resection of all involved
tissues is mandatory
d. The use of hyperbaric oxygen has been demonstrated to be clearly
advantageous
Answer: a, c
Identification of a necrotizing, soft tissue infection mandates immediate

operative intervention with aggressive resection of all involved tissues and


empiric administration of antibiotics active against gram-positive, gramnegative, and anaerobic bacteria. In most cases, this involves the use of
several antimicrobial antibiotics in combination. Because of concern in all
cases for the presence of Clostridia infection, high doses of aqueous
penicillin G are administered. Gram-positive organisms are treated with
vancomycin or a semisynthetic penicillin and gram-negative organisms are
treated with an aminoglycoside or a monobactam. Anaerobic coverage is
typically achieved by use of metronidazole of clindamycin. The use of
hyperbaric oxygen therapy is controversial and unfortunately due to the rarity
of the disease, prospective randomized data is not available so that the
literature remains without controlled trials demonstrating any additional
benefits derived from hyperbaric oxygen therapy.

200 Wounds are classified according to the likelihood of bacterial


contamination. Which of the following statement(s) is/are true concerning
wound classifications?
a. A clean-contaminated wound would be that associated with an elective
colon resection with adequate mechanical and antibiotic bowel preparation
b. A contaminated wound would include a resection of obstructed bowel with
gross spillage of intestinal contents
c. In a clean wound, no viscus is entered
d. Antibiotic prophylaxis should be administered for all clean-contaminated
and contaminated wounds and selectively in patients involving a clean wound
Answer: a, b, c, d
Wounds are classified under three classes according to the likelihood of
bacterial contamination: 1) clean (no viscus is entered; e.g., herniorrhaphy);
2) clean-contaminated (minimal contamination; e.g., elective colon resection
with adequate mechanical and antibiotic bowel preparation, and 3)
contaminated (heavily contaminated surgery; e.g., resection of unprepared,

obstructed bowel with gross spillage of intestinal contents or stool, drainage


of abscesses, debridement of traumatic neglected wounds). Antibiotic
prophylaxis generally should be administered for class 2 and 3 types of
wounds, but patients undergoing clean surgery do not always require
antimicrobial antibiotic prophylaxis. An exception to this tenet involves cases
in which a prosthetic material may be used (artificial joint, heart valve, tissue
patch).

201 The following statement(s) is/are true concerning HIV infection.


a. Initial screening with ELISA is highly sensitive but can be associated with
a false positive rate of 25%
b. Treatment with azidothymidine (AZD) appears to prolong survival when
administered early in the disease
c. Predisposition to infection in HIV infection is primarily due to reduction in
the number of helper T cells
d. Common infections in patients with AIDS are Pneumocystis, carinii
pneumonia, CMV pneumonitis, Cryptococcus meningitis, and disseminated
infection due to atypical mycobacteria
Answer: b, c, d
Acquired immunodeficiency syndrome (AIDS) is a syndrome caused by the
human retrovirus (HIV-1) that infects T lymphocytes and causes severe
immunosuppression. Individuals who become infected with HIV are prone to
a variety of infections and different types of malignancy. A spectrum exists in
which patients regress from asymptomatic infection, to development of
AIDS-related complex (ARC) of diseases to AIDS itself. Common infections
occurring in patients with AIDS are Pneumocystis carinii pneumonia; CMV
pneumonitis; gastritis, hepatitis and meningitis due to Cryptococcus
neoformans; and pneumonia and disseminated infection due to atypical
mycobacteria. Predisposition to these infections is due, in part, to the
lymphotrophic nature of HIV, which markedly reduces the number of helper

T cells as well as the absolute number of T cells.


HIV detection typically consists of initial ELISA screening, but this test has
about a 13% false-positive rate, thus mandating all positive tests be
confirmed by the western immunoblot analysis.
Treatment of ARC and AIDS consists of aggressive antiinfective therapy
once a specific infection occurs and the use of AZT. AZT has been shown to
prolong survival when administered early in the course of disease and is
considered routine therapy.

202 The following statement(s) is/are true concerning initial microbiologic


diagnostic techniques.
a. Appropriate expeditious transport of specimens to the microbiology
laboratory is essential for obtaining accurate clinical information
b. The use of potassium hydroxide in preparing a specimen slide for light
microscopy will be useful in identification of anaerobic bacteria
c. Antibiotic sensitivity is determined by exposing the specific microorganism
to varying amounts of antibiotic with the concentration of the antibiotic
inhibiting growth referred to as the MIC (minimal inhibitory concentration)
d. Serum levels of antimicrobial agents should achieve in excess of a 4-to 8fold increase over the MIC to be considered clinically efficacious.
Answer: a, c, d
Because most surgical infections are polymicrobial, specimens should be
cultured for aerobic and anaerobic bacteria, as well as fungi. Although
aerobic and aerotolerant microorganisms often do not require special
transport media, a delay in specimen processing may markedly reduce the
yield, and anaerobic transport media have been demonstrated to markedly
increase the cultural yield of this type of organism. The initial piece of
information gained concerning potential infection may come from simple
staining of a specimen. Gram stain, which will identify the staining
characteristics of the organisms, as well as their number should be performed

on all specimens. Potassium hydroxide is useful in that it will lyse bacteria


and other cellular elements within a preparation and allow observation of
yeast or mycelial elements.
Initial culture results may solely indicate that microorganisms are growing
and full characterization may take two to three days. Once a specific
microorganism is identified, a sample is inoculated during the log phase into
broth containing varying amounts of an antibiotic. After an 18-to 24-hour
period, the tube or well that exhibits no visible growth is then noted, and the
reciprocal of this dilution is termed the minimal inhibitory concentration
(MIC). This value may be compared to either measured or known achievable
serum levels for a particular antibiotic. In general, antimicrobial agents that
achieve in excess of a 4-to 8-fold increase over MIC during the peak serum
level have been demonstrated to be clinically efficacious.

203 The complement system consists of a series of serum proteins that exist
in a quiescent or very low-level state of activation in the uninfected host.
Which of the following statement(s) is/are true concerning complement
activation?
a. The alternate (properdin) pathway of complement activation can occur
directly through contact with fungal or bacterial cell wall compounds
b. Complement component fragments may serve to decrease vascular
permeability
c. Excessive complement activation can produce deleterious effects
d. Fragments of certain complement components serve as chemoattractants to
additional cellular components of the host defense mechanism
Answer: a, c, d
Complement activation can occur through either classic or alternate
(properdin) pathways, both of which eventuate in deposition of terminal
complement pathway components on the antigenic cell surface. The classic
pathway of complement activation usually begins with immunoglobulin G-

binding which has also bound the antigen. The alternate pathway activation
occurs in response to activation of direct binding of the antigen or directly
through contact with fungal and bacterial cell wall compounds such as
zymosan and gram-negative bacterial lipopolysaccharide (LPS endotoxin).
Several complement components represent important host defenses acting to
recruit or augment cellular host defenses or to directly inactivate invading
microbes through lytic activity. The production of complement component
fractions C3a and C5a during activation of this cascade serve primarily to
markedly increase vascular permeability, and C5a functions as a PMN and
macrophage chemoattractant. This process leads to the recruitment of
additional humoral and cellular defenses to the specific area of infection.
Excessive complement activation can produce deleterious effects in some
instances. Complement activation causes enhanced PMN adhesion,
margination, and release of lysosomal enzymes that can directly damage
certain target tissues, such as the lung.

204 A 55-year-old renal transplant patient has been hospitalized in a Surgical


Intensive Care Unit, receiving a prolonged course of antibiotics following an
attack of acute cholecystitis. The following statement(s) is/are true
concerning his management.
a. Due to the potential risk of Candida infection, prophylaxis with oral
nystatin should be instituted early in the patients course
b. A Candida urinary tract infection should be treated with systemic
amphotericin B
c. Changes of Candida retinitis are of little significance
d. The presence of a virulent Candida bacteremia should suggest a dosage
reduction in immunosuppressive agents until the infection can be adequately
controlled
Answer: a, d
Infections due to fungal pathogens have become increasingly common during

the past decade, frequently occurring in patients undergoing prolonged


hospitalization in the Surgical Intensive Care Unit and in
immunocompromised individuals. Prophylaxis with oral antifungal agents
(nystatin) is warranted, especially during periods of maximal
immunosuppression in transplant patients, in patients with uncontrolled
diabetes, or during some cases of prolonged antibacterial microbial therapy.
In general, local, apparently noninvasive Candida infections involving the
integument and mucus membranes are treated with oral decontamination and
topical antifungal therapy using topical agents such as nystatin. Candida
urinary tract infections can be treated with either an oral antifungal agent or
with topical amphotericin B as a continuous bladder irrigation. Several
studies have demonstrated that those patients with three positive sites of
Candida infection, or with peritoneal or blood cultures positive for Candida
exhibit higher survival rates when amphotericin B therapy is instituted earlier
in the course of infection. The presence of retinal changes compatible with
Candida retinitis or Candida present within the peritoneal cavity are
considered indications for a limited course of amphotericin B therapy (300%
to 500 mg). Patients receiving exogenous immunosuppressive agents should
undergo a marked dose reduction, and some agents should be discontinued
until evidence of infection is absolutely controlled or is eradicated.

205 The initiation of a humoral immune response involves a complex


interaction of the antigen, cells and intercellular messengers. Which of the
following statement(s) concerning the initiation of the humoral immune
response is/are correct?
a. Helper T lymphocytes stimulate B lymphocytes through secretion of
cytokines such as interleukin 4 and 6
b. A number of cells can aid in presenting the antigen to the helper T cell
including B lymphocytes and macrophages
c. All antigens require coordinated efforts of the various cellular components
of the immune system

d. An antigen must be a living microorganism


Answer: a, b
Stimulation of the immune system occurs after a variety of antigenpresenting cells (B lymphocytes, macrophages, dendritic cells, and
Langerhans cells) act to engulf, process, and present antigen to T
lymphocytes of helper lineage. These T lymphocytes, in turn, act to stimulate
B lymphocytes to become mature plasmacytes (through secretion of
cytokines such as interleukin 4 and 6) dedicated to the production of antibody
directed against the specific antigen. An antigen may be defined as any
substance that stimulates the host immune response; that is, that the host
immune system recognizes is foreign. Thus, an antigen may be an invading
microorganism, an inert particle, or any type of chemical compound that
triggers the host immune system. Although some antigens are able to directly
stimulate B lymphocytes in and of themselves to produce antibody (many
polysaccharides), most antigens require coordinated efforts of the various
cellular components of the immune system.

206 The following statement(s) is/are true concerning viral infections.


a. The most common post-transplantation viral infections are caused by
herpes viruses and include CMV and herpes simplex virus
b. Viral infections occur at equal frequency anytime during the posttransplantation period
c. CMV infection in the post-transplant patient is most likely a pulmonary
process
d. Herpes simplex virus (HSV) infection primarily presents with a
mononucleosis-type syndrome with fever, lethargy, and cough
Answer: a, c,
Solid organ transplant patients are prone to develop viral infection by virtue
of exogenous immunosuppression. The most common post-transplantation

viral infections are those caused by herpes viruses (CMV, herpes simplex
virus [HSV], Epstein-Barr virus [EBV], and Varicella-Zoster virus [VZV]).
All are most common during periods of maximal host immunosuppression
that occur immediately post-transplantation and during periods of allograft
rejection. CMV is a common cause of fever after solid organ transplantation,
and evidence of CMV infection occurs in approximately 30% of patients. The
most common presentation for CMV infection is that of a febrile, leukopenic
patient with a cough, diffuse interstitial infiltrates on chest x-ray, and
hypoxia.
HSV infection causes primarily oral pharyngeal ulcerations in most cases,
although sporadic cases of disseminated disease have been reported. EBV
causes an occasional case of mononucleosis-type syndrome but has also been
clearly indicated in the pathogenesis of post-transplantation lymphomas.
VZV infection can present as disseminated and occasionally life-threatening
infections in the nonimmune transplant patient or as painful herpes zoster in
patients who have previously developed chicken pox.

207 The following statement(s) is/are true concerning necrotizing fascitis.


a. Mortality rates as high as 40% can be expected
b. The infection involves only the superficial fascia, sparing the deep
muscular fascia
c. An impaired immune system is a common factor predisposing to this
condition
d. The infection is usually polymicrobial
e. Necrotizing fascitis is most likely to develop in the face of impaired fascial
blood supply
Answer: a, c, d, e
Necrotizing fascitis is an uncommon infection of the deep and superficial
fascia that is associated with mortality as high as 40% in many series.
Although many underlying disease processes predispose patients to

necrotizing fascitis, three common factors are almost invariably present: 1)


impairment of the immune system; 2) compromise of fascial blood supply,
and 3) the presence of microorganisms that are able to proliferate within this
environment. Infections of this type are usually polymicrobial in nature, with
gram-positive organisms such as staphylococci and streptococci, gramnegative enteric bacteria, and gram-negative anaerobic being frequently
identified. These polymicrobial cultural results are assuredly indicative of the
occurrence of a synergistic process, perhaps in large part accounting for the
severity of these infections. Some microorganisms possess virulence factors
that, in conjunction with an underlying host predisposition, allow this disease
process to occur without dependence on other bacteria. Examples of such
bacteria include Clostridium, Pseudomonas, and Aeromonas. In these
patients, the process is often fulminant and is frequently associated with
cellulitis, myositis, fascitis, and bacteremia with attendant high mortality.

208 New treatment modalities designed to modulate host defense


mechanisms that have been demonstrated conclusively to be of benefit
include:
a. Gut decontamination
b. Anti-LPS antibody
c. Anti-TNF antibody
d. Thymopentin
e. None of the above
Answer: e
Selective gut decontamination involves the use of orally administered
antibiotics that achieve a high intraluminal level directed against gramnegative aerobes and yeast, leaving the host anaerobic intestinal microflora
relatively undisrupted. Although a reduction and alteration of the
microorganisms responsible for infectious episodes have been demonstrated
in certain groups of patients, a clear-cut impact on host mortality has not been

shown. Because LPS may be responsible for toxicity both directly and
through host mediator systems, the availability of agents to bind against this
portion of the gram-negative bacteria to reduce mortality has been intensively
examined. Unfortunately, large multicenter randomized trials provide no
evidence of benefit for this treatment. Similarly, since many of the systemic
manifestations of gram-negative bacteremia are mediated by cytokines, the
effect of an anti-TNF antibody preparation is currently in clinical trial. No
proven benefits have yet been identified. Finally, the use of
immunostimulants to enhance the state of activation of host defenses has
been proposed. Thymopentin is a peptide that contains active thymopoetin, a
thymic molecule that acts to stimulate T-lymphocyte activity. Preliminary
trials indicate that this agent ameliorates host septic response after major
operations and trauma but conclusive evidence that concurrent reduction of
infection-related mortality occurs is not available.

209 Antibacterial agents can be classified with regard to their structure,


mechanism of action, and activity pattern against various types of bacterial
pathogens. Which of the following statement(s) is/are true concerning
antimicrobial classes?
a. Penicillins and cephalosporins share the compound structure of a b-lactam
ring which binds to bacterial division plate proteins
b. Tetracyclines and macrolides such as erythromycin inhibit bacterial
ribosomal activity and therefore protein synthesis
c. Aminoglycosides act in a similar fashion to tetracyclines and therefore are
both bacteriostatic
d. Sulfonamides and trimethoprim act synergistically to inhibit purine
synthesis
Answer: a, b, d
Penicillins, cephalosporins, and monobactams possess a b-lactam ring of
some type and act to bind bacterial division plate proteins, thus inhibiting cell

wall peptidoglycan synthesis and either causing or inducing autolytic


bacteriolysis. Because gram-positive and gram-negative bacteria possess
different types of division plate proteins, many of these agents exhibit
differential activity between these two types of microorganisms.
Tetracyclines, chloramphenicol, and macrolides inhibit bacterial ribosomal
activity, and thus overall protein synthesis by a variety of different
mechanisms. Aminoglycosides act to inhibit protein synthesis and also
presumably act on a different target site, a supposition based on the fact that
aminoglycosides are bacteriolytic and the other agents are bacteriostatic.
Vancomycin inhibits assembly of peptidoglycan polymers, whereas
quinolones bind to DNA helicase proteins and inhibit bacterial DNA
synthesis. Sulfonamides and trimethoprim act in different mechanisms to
inhibit protein synthesis, therefore two agents in combination act
synergistically.

210 The treatment of the following patient should include:


a. Initial empiric therapy directed against both aerobes and anaerobes
b. The addition of anti-fungal therapy in an elderly patient
c. A minimum of two weeks of antibiotic therapy is indicated
d. The addition of appropriate antibiotic therapy has made surgical therapy
unnecessary in such cases
e. Either a single agent or combination therapy is appropriate if the agents
selected possess activity against both aerobic and anaerobic bacteria
Answer: a, e
The primary treatment for a perforated viscus is surgical, however
antimicrobial therapy is an extremely important adjunct. Empiric antibiotic
therapy for secondary bacterial peritonitis and intraabdominal abscess should
be directed against both aerobes and anaerobes. Administration of an agent
directed against only one component of the infection or the other is inferior to
combined therapy. Several studies indicate that the results of using several

agents in combination is equivalent to the use of a single agent therapy as


long as the agents selected possess activity against both components of the
infection. The addition of antientercoccal or antifungal agents as initial
therapy has not been substantiated. The most beneficial duration of antibiotic
therapy must be based on the setting for the specific patient. Minimal
peritoneal contamination with adequate surgical treatment may be treated
with a three-to five-day course of antibiotics, whereas longer periods are
indicated for immunosuppressed patients and with patients with extensive
contamination.

211 The following statement(s) is/are true concerning host defense


mechanisms to intraabdominal infection.
a. Bacterial clearance can occur via translymphatic absorption
b. Phagocytic activity and bacterial killing can occur via resident phagocytic
cells and an influx of PMNs
c. A fibrinogen-rich inflammatory exudate is released into the peritoneal
cavity, trapping large numbers of bacteria and other particulate matter
d. Perforations of a bowel may be walled off but are seldom sealed by the
omentum and other mobile viscera
Answer: a, b, c
The introduction of microorganisms into the normally sterile peritoneal
environment invoke several potent specialized host antimicrobial defense
mechanisms. Bacterial clearance, also termed translymphatic absorption,
occurs through specialized structures found only on the peritoneal
mesothelium on the underside of the diaphragm that act as conduits for both
fluid and particulate matter. Lymphatic channels eventually form which drain
into the venous circulation via the thoracic duct. Bacteria not cleared via
translymphatic absorption are rapidly engulfed by resident and recruited
phagocytic cells including resident macrophages on the peritoneal surface
and omentum and attracted PMNs. The final primitive host defense

mechanism is sequestration by which a fibrinogen-rich exudate containing


plasma oposonins appears during peritoneal infection and fibrin
polymerization occurs. Fibrin has the capacity to trap large numbers of
bacteria and other particulate matter. Acting in conjunction with omentum
and other mobile viscera, perforations are sealed and the contaminated enteric
contents walled off, preventing continued soilage of the peritoneal cavity.

212 A 67-year-old male presents with an intraabdominal abscess secondary to


perforated sigmoid diverticulitis. The following statement(s) is/are true
concerning his intraabdominal abscess.
a. Culture will likely reveal a solitary organism
b. Both aerobic and anaerobic islets are encountered in 50% of specimens
c. The most common aerobic islet will be likely E. coli and other gramnegative enteric bacilli
d. The most common anaerobic islet will be a Bacteroides species
Answer: b, c, d
Typically an intraabdominal infection results in perforation of a hollow viscus
and the ensuing contamination of a normally sterile peritoneal cavity. The
normal bacterial flora found in that particular location of the alimentary tract
thus determines the initial inoculum. In parallel with the overall quantity of
microorganisms, (both aerobes but predominantly anaerobes) perforations of
the lower small bowel and colon produce a high frequency of infections that
contain anaerobic microorganisms. Certain predictable patterns of bacterial
islets are found, but on average four to five islets occur in patients with
established intraabdominal infection, more than half of which are anaerobes.
Both aerobes and anaerobes are encountered in 80% to 90% of specimens.
Commonly encountered aerobes isolated are E. coli and other gram-negative
enteric bacilli such as Enterobacter, Klebsiella. Among the anaerobes,
Bacteroides species (especially B. fragilis, Clostridium), and anaerobic cocci
are most consistently isolated.

213 The following statement(s) is/are true concerning gram-negative bacterial


sepsis.
a. Mortality due to this condition has almost been eliminated due to
therapeutic intervention with antibiotics, aggressive hemodynamic
monitoring and fluid resuscitation
b. Recent series have noted a decrease in the incidence of this condition
c. Predisposing factors include old age, malnutrition, and immunosuppression
d. Pseudomonas bacteremia is the most common cause of gram-negative
sepsis
e. Polymicrobial sepsis is generally considered a more serious problem than
sepsis due to a single organism
Answer: c, e
Gram-negative bacterial sepsis is a serious disease process that produces
substantial morbidity and mortality in both normal and immunocompromised
patients (10% to 20% and 30% lethality, respectively), despite therapeutic
intervention with antimicrobial agents, aggressive hemodynamic monitoring,
fluid resuscitation, and metabolic support. During the past several decades,
nosocomial infections due to gram-negative pathogens have increased in
frequency with resultant increase in the incidence of gram-negative
bacteremia to between 3 and 13 cases per 1000 hospital admissions. Factors
that predispose to these infections include: 1) underlying host disease
processes such as malignancy, diabetes; 2) old age and disability; 3)
malnutrition; 4) previous or concurrent antimicrobial antibiotic therapy; 5)
major operations; 6) respiratory or urinary manipulation or intubation; and 7)
immunosuppression.
Although many different organisms cause this form of sepsis, E. coli
predominates in overall frequency. Also common are isolates of Klebsiella,
Enterobacter and Serratia; Pseudomonas bacteremia is less common. Some
studies, however, have suggested that Pseudomonas sepsis is associated with

the highest lethality. In several series, 10% to 20% of patients have had
polymicrobial series, and most investigators agree that polymicrobial sepsis
is more lethal than infection with a single organism.
214 Which of the following statement(s) is/are true concerning the various
types of shock?
a. Traumatic shock is more commonly associated with subsequent organ
injury and multiorgan failure syndrome than hemorrhagic shock
b. Cardiogenic shock can be of either an intrinsic or compressive nature
c. Hypodynamic septic shock is associated with a decreased mortality risk
when compared with hyperdynamic septic shock
d. Hypoadrenal shock usually responds quickly to resuscitation
e. Neurogenic shock occurs with the absence of sympathetic activity
Answer: a, b, d, e
Classification schemes of shock based on cause have been developed for the
seemingly dissimilar processes leading to circulatory collapse and the shock
state. Hypovolemic shock, the most common, is the result of intravascular
volume depletion through loss of red blood cell mass or plasma volume.
Microvascular hypotension results from a combination of low intravascular
blood volume, diminished cardiac output, and compensatory sympathetic
peripheral vasoconstriction. Shock associated with trauma (traumatic shock)
arises from the consequences of hypovolemia due to hemorrhage in
conjunction with direct soft tissue injury and bone fracture. Hypovolemia
caused by blood loss and fluid extravasation into injured tissues is
compounded by activation of maladaptive inflammatory cascades initiated by
the tissue injury. In contrast to pure hemorragic shock, subsequent organ
injury and multiorgan failure syndrome (MOFS) occurs much more
frequently following traumatic shock due to the over-expression of these
immuno-inflammatory cascades. Cardiogenic shock is the result of failure of
the heart as an effective pump, resulting in inadequate cardiac output, tissue
perfusion and oxygen delivery. Intrinsic causes include myocardial infarction,

cardiomyopathy, valvular heart disease, or rhythm disturbances. Compressive


cardiogenic shock is a discrete entity that results when extrinsic compression
of the heart limits diastolic filling and thus systolic ejection and cardiac
output. Septic shock refers to hypotension and circulatory insufficiency
developing as a consequence of infection and the systemic response to that
infection. In its hyperdynamic form, septic shock is marked by diminished
peripheral vascular resistance and generalized vasodilatation causing relative
hypovolemia. In contrast, hypodynamic septic shock occurs in situations of
inadequate resuscitation or preterminal cardiovascular decompensation, and
is associated with vasoconstriction and a greatly increased mortality risk.
Sympathetic denervation through spinal cord injury, spinal anesthesia, or
severe head injury produces generalized arterial vasodilatation and
venodilation. Shock occurs when the normal blood volume fails to fill the
available intravascular space and severe relative hypovolemia exists. Despite
hypotension, there is a noteworthy absence of sympathetic activity, as occurs
in hypovolemia or cardiogenic shock. Profound shock can occur in surgical
patients following stress due to the loss of the homeostatic corticosteroid
response. Hemodynamic instability may develop after an operative procedure
or coincident with an unrelated illness. The profound circulatory collapse is
often refractory to vigorous resuscitation with fluids and pressor agents. The
response to exogenous corticosteroids is usually dramatic and potentially lifesaving.

215 Which of the following statement(s) is/are true concerning metabolic


derangements in sepsis and the systemic inflammatory response syndrome
which may follow progressive shock?
a. Alterations in glucose metabolism lead to the development of efficient
substrate utilization
b. A progressive rise in serum triglyceride levels result from less efficient
clearance and increased hepatic lipogenesis
c. A net negative nitrogen balance occurs due to the oxidative metabolism of

proteins to meet energy needs


d. The serum aromatic amino acids fall rapidly as they are actively used in
oxidative metabolism
Answer: b, c
A broad spectrum of metabolic abnormalities become apparent in sepsis and
the systemic inflammatory response syndrome following shock. Disruption of
the normal cycles of carbohydrate, lipid, protein, and oxygen metabolism
occur as hypermetabolism develops. Through the Cori cycle, lactate from the
periphery is shuttled back to the liver, where it is used in the production of
glucose. Because pyruvate is converted to alanine in the periphery, flux of
alanine also contributes to hepatic gluconeogenesis. The glycolytic oxidation
of glucose to pyruvate and its subsequent glugoneogenic regeneration from
lactate is an inefficient cycling of substrate. There is no net energy
production, but heat is released in significant quantities. Alterations in lipid
metabolism cause a progressive rise in the serum triglyceride level as a result
of less efficient clearance of exogenous triglycerides coupled with increased
hepatic lipogenesis. Profound alterations in protein and amino acid
metabolism develop with characteristic changes in amino acid levels,
nitrogen balance, and skeletal muscle mass. Initially levels of the branch
chain amino acids are reduced, whereas those of the aromatic amino acids are
elevated. There is an increase in the oxidative metabolism of protein to meet
energy needs and a tremendous mobilization of nitrogen with net negative
nitrogen balance. The branch chain amino acids are preferentially utilized in
the TCA cycle to maintain an activity that otherwise would be lost from the
diminished entry of carbohydrate-and fatty acid-generated acetyl coenzyme
A. This results in reduced serum level of leucine, isoleucine and valine.

216 Which of the following statement(s) is/are true concerning the


microvascular and cellular response to shock?
a. Osmodically induced mobilization of intracellular fluid is the initial

response to restore intravascular volume


b. With larger volume hemorrhagic shock deterioration of normal cellular
transmembrane potential occurs resulting in an increase in extracellular
sodium and water
c. The accumulation of anaerobic metabolites override normal homeostatic
vasomotor tone and contribute to the maladaptive vasodilatation
d. Abnormal intracellular calcium homeostasis may contribute to the cellular
dysfunction of shock
Answer: c, d
Moderate hypovolemia results in a relatively rapid spontaneous restitution of
intravascular volume through expansion of the plasma space. This plasma
expansion by erythrocyte free fluid occurs within one hour as a result of
alterations in pressure and osmolarity and produces an associated
hemodilution. Sympathetic discharge, associated arteriolar constriction, and
induced metabolic changes in osmolarity initiate the compensatory events at
the microcirculatory level. The initial pressure-related phase of restitution of
blood volume in shock is overlapped by a second phase involving
osmotically induced mobilization of intracellular fluid. Osmotic mechanisms
contributing to the restitution of blood volume after moderate hemorrhage are
not adequate in hemorrhage of greater magnitude. In larger hemorrhages
(over 25% blood volume), there is also deterioration of the normal cellular
transmembrane potential, an increase in intracellular sodium and water, and a
concomitant decrease in extracellular fluid volume. Tissue hypoxia results,
anaerobic metabolites accumulate, and the cell cannot maintain the normal
cell membrane potential. Accumulation of hydrogen ion, lactate, and other
products of anaerobic metabolism override homeostatic vasomotor tone and
contribute to a maladaptive vasodilatation, further augmenting hypotension
and hypoperfusion. The uptake of fluid by the failing cell is a major source
of food sequestration following shock. Loss of membrane function is
proportional to both the extent and duration of shock or degrees of sepsis.
The etiology of membrane failure is unclear but appears multifactorial. Loss

of intracellular ATP energy stores during hypoperfusion or direct toxicity


during sepsis may inhibit the membrane sodium-potassium pump. Cellular
dysfunction also appears to be related to abnormal intracellular calcium
homeostasis.

217 Which of the following statement(s) is/are true concerning the pulmonary
response to shock?
a. The acute pulmonary vascular response to shock differs markedly from that
of systemic vasculature
b. The pulmonary edema of ARDS occurs in the face of elevated left heart
pressures
c. The initial physiologic changes of ARDS involve the capillary endothelial
cells and the type I pneumocyte
d. Mechanisms proposed in the pathogenesis of ARDS include injury from
mediators of inflammation elsewhere and from activated cellular elements
e. A decrease in lung compliance may result from the loss of type I
pneumocytes
Answer: c, d, e
Contributing pathophysiologic processes to the pulmonary manifestations of
shock include the pulmonary component of the cardiovascular response,
disruption of the normal lung mechanics, and acute lung injury or ARDS due
to sepsis. Pulmonary function may be further compromised by pathology
intrinsic to the lung itself, including pulmonary contusion, aspiration, airway
obstruction, pneumonia, pneumothorax, hemothorax, and atelectasis. The
acute pulmonary vascular response to shock largely parallels that of the
systemic vasculature. The increase in pulmonary vascular resistance, which
may proportionally exceed that of the systemic circulation, transiently
accompanies the systemic adrenergic response. ARDS is a syndrome of
progressive lung injury that may arise as a direct consequence of shock or
other disease processes. The characteristic findings of ARDS are the presence

of pulmonary edema, hypoxemia, and significantly decreased lung


compliance. The pulmonary edema is noncardiac in origin and occurs in the
face of normal left heart pressures. The hypoxemia results from the
development of intrapulmonary shunting and perfusion of under and
nonventilated alveoli. The decrease in lung compliance results from the loss
of surfactant and lung volume in combination with the presence of interstitial
fluid and alveolar edema. Progressive histologic changes of ARDS become
apparent in pulmonary capillaries, interstitium, and alveoli. Initially,
interstitial edema develops with swelling of the capillary endothelial cells and
the type I pneumocytes. The type I pneumocytes subsequently slough, and
alveolar edema ensues. Functional surfactant is lost with a significant
increase in alveoli opening pressure and decrease in alveolar surface tension.
Mechanisms proposed in the pathogenesis of ARDS include injury from
mediators of inflammation elaborated elsewhere, and from activated cellular
blood elements.

218 Which of the following statement(s) is/are true concerning the diagnosis
and management of hypovolemic shock?
a. A fall in hematocrit or hemoglobin always accompanies hemorrhagic shock
b. The treatment of shock is generic regardless of the etiology
c. Pharmacologic intervention to increase myocardial contractility in
hypovolemic shock is an important part the early management
d. Complications are less frequent after treatment of hemorrhagic shock than
septic or traumatic shock
Answer: d
Hypovolemic shock is readily diagnosed when there is an obvious source of
volume loss and overt signs of hemodynamic instability and increased
adrenergic output are present. After acute hemorrhage, hemoglobin and
hematocrit values do not change until compensatory fluid shifts have
occurred or exogenous fluid is administered. These values decrease once

transcapillary refill, osmotic-induced shifts, or non-RBC volume resuscitation


expands the blood volume. It is imperative that the distinction be made
between hypovolemic and cardiogenic forms of shock, because appropriate
therapy differs dramatically. Restoration of perfusion in hypovolemic shock
requires reexpansion of circulating blood volume in conjunction with
necessary interventions to control ongoing volume loss. Continued
hemodynamic instability after fluid resuscitation implies that shock has not
been reversed or that there is ongoing blood or volume loss. In severe,
prolonged hypovolemia, ventricular contractile function may itself become
depressed and require inotropic support to maintain ventricular performance,
but in general, pharmacologic interventions directed toward increased
contractility in situations of inadequate preload are ineffective, further
complicate metabolic derangements, and are not indicated until adequate
volume replacement has been completed. Complications are less frequent
after treatment of hemorrhagic shock than in situations of septic or traumatic
shock. In the later circumstances, the massive activation of inflammatory
mediator response systems and consequences of their disseminated,
indiscriminate cellular injury can be quite profound.

219 Which of the following statement(s) is/are true concerning the


neuroendocrine responses to shock?
a. Sympathetic nerve endings release epinephrine which is responsible for
greater than 80% of systemic vascular resistance
b. Endogenous epinephrine is the primary contributor to systemic vascular
resistance
c. Increased pancreatic secretion of glucagon contributes to glucose
intolerance associated with injury and sepsis
d. The renin-angiotensin axis further augments the sympathetic-mediated
vasoconstriction
Answer: c, d

The neuroendocrine response to shock attempts to achieve restoration of


effective blood volume, mobilization of metabolic substrates, and
maintenance of central profusion. Both peripheral and central afferent stimuli
to the central nervous system are involved in inducing this response.
Hypotension, associated with a decrease in impulses from the aortic and
carotid baroreceptors, disinhibits the vasomotor center. This disinhibition
results in increased adrenergic output and decreased vagal activity.
Sympathetic nerve endings release norepinephrine, inducing peripheral and
splanchnic vasoconstriction which is responsible for greater than 80% of
systemic vascular resistance and is a major contributor to maintenance of
central organ perfusion and venous return. Plasma levels of both epinephrine
and norepinephrine are elevated with injury, and the degree of the
catecholamine elevation corresponds to the magnitude of injury. In shock the
effects of endogenous epinephrine are largely metabolic. In addition to
initiating autonomic nervous activity, the hypothalamus secretes releasing
hormones, which induce the stress hormone release of the pituitary. As part of
this response, adrenocorticotropic hormone (ACTH) secretion by the anterior
pituitary is increased stimulating cortisol secretion by the adrenal cortex. In
conjunction with elevated plasma levels of cortisol and epinephrine,
increased pancreatic secretion of glucagon accelerates hepatic
gluconeogenesis and further aggravates the glucose intolerance that follows
injury and sepsis. The secretion of renin is increased in responses to
adrenergic discharge and decreased perfusion of the juxtaglomerular
apparatus in the kidney. Renin allows formation of angiotensin I in the liver,
which is then converted to angiotensin II in the lungs. Angiotensin II is an
extremely effective vasoconstrictor that further augments sympatheticmediated vasoconstriction.
220 A 22-year-old man sustains a single stab wound to the left chest and
presents to the emergency room with hypotension. Which of the following
statement(s) is/are true concerning his diagnosis and management?
a. The patient likely is suffering from hypovolemic shock and should respond

quickly to fluid resuscitation


b. Becks triad will likely be an obvious indication of compressive
cardiogenic shock due to pericardial tamponade
c. Echocardiography is the most sensitive noninvasive approach for diagnosis
of pericardial tamponade
d. The placement of bilateral chest tubes will likely resolve the problem
Answer: c
Shock from cardiac compression occurs when external pressure on the heart
impairs ventricular filling. Because ventricular filling is a function of venous
return and myocardial compliance, any process that places pressure on the
heart can cause compressive cardiogenic shock. Included among these are
pericardial tamponade, tension pneumothorax, mediastinal hematoma, and
positive pressure from mechanical ventilation. Any patient with hypotension
after a wound in proximity of the heart should be considered to have
compressive cardiogenic shock until proven otherwise. The classical clinical
findings of pericardial tamponade include Becks triad of hypotension, neck
vein distention and muffled heart sounds. Pulses paradoxus may be noted
(this involves a decrease rather than the normal increase of systolic blood
pressure with inspiration; values 10mmHg are significant). These findings,
however, may be obscured in a noisy emergency room environment by
positive pressure ventilation or by associated injuries. Placement of a CVP
catheter confirms the elevation of right-sided filling pressure. If a pulmonary
artery catheter has been placed, findings consistent with tamponade or other
forms of cardiac compression are a trend toward equalization of chamber
pressures as hypotension progresses. In the patient at risk, echocardiography
is an extremely sensitive and noninvasive approach to demonstrate
pericardial fluid and the need for operation. Pericardial tamponade must be
relieved urgently and cardiac injuries require emergent sternotomy. Chest
tube placement would not be appropriate as the sole treatment in this patient.

221 A 32-year-old man suffers a spinal cord injury with a resultant paraplegia
in a motorcycle accident. He presents to the emergency room with
hypotension. Which of the following statement(s) is/are true concerning his
diagnosis and management?
a. The low blood pressure can be assumed to be due to neurogenic shock
b. The sole cause of hypotension is the loss of sympathetic input to the
venous system
c. Despite significant hypotension, secondary organ injury will be uncommon
d. There is no role for pharmacologic intervention to maintain blood pressure
Answer: c
Neurogenic shock results from interruption of sympathetic vasomotor input
and develops after spinal cord injury, spinal anesthesia, and severe head
injury. Under normal conditions, baseline sympathetic activity establishes a
degree of arteriolar and venous constriction. Ablation of this tone results in
decreased systemic vascular resistance and a dramatic increase in venous
capacity, causing hypotension due to relative hypovolemia. Arteriolar
dilatation not only lowers the systemic vascular resistance but also allows
previously unopened vascular beds to be perfused, greatly expanding venous
capacity. Removal of sympathetic inputs to innervated portions of the venous
system allows further venodilatation. Restoration of an effective, albeit
expanded, intravascular volume may require extremely large volumes of
resuscitation fluid to restore normal cardiac filling pressures. This will restore
cardiac output and reverse hypotension. However, pharmacologic
intervention with vasoactive drugs may be necessary and is preferable to
excessive volume resuscitation. Post-shock sequelae are infrequent. Although
there is significant hypotension with neurogenic shock, there is usually little
if any hypoperfusion. Thus, activation of inflammatory cascade and
subsequent organ injury rarely occur.
A major pitfall in the management of neurogenic shock arises when there is
coexistent hemorrhage or ongoing volume loss that is not appreciated. This is not
an unusual situation because cervical spine trauma causing paraplegia or severe
head injury is frequently associated with multiple injuries. Thus, in trauma the initial
res

se to neurogenic shock is large volume resuscitation regardless of the presu med

etiology. If hemodynamic instability persists after initial trauma resuscitation,


one must assume that the cause is not neurogenic and search for occult blood
loss or cardiogenic causes of shock.
222 Which of the following statement(s) is/are true concerning septic shock?
a. The clinical picture of gram negative septic shock is specifically different than
shock associated with other infectious agents
b. The circulatory derangements of septic shock precede the development of
metabolic abnormalities
c. Splanchnic vascular resistance falls in similar fashion to overall systemic vascular
resistance

d. Despite normal mechanisms of intrinsic expansion of the circulating blood


volume, exogenous volume resuscitation is necessary
Answer: d
The clinical findings in sepsis and septic shock represent the host response to
infection. Gram-positive and gram-negative bacteria, viruses, fungi,
rickettsiae, and protozoa have all been reported to produce a clinical picture
of septic shock, but the overall response is independent of the specific type of
invading organism. Septic shock develops as a consequence of the
combination metabolic and circulatory derangements accompanying the
systemic infection. It appears that the circulatory deficits are preceded by the
metabolic abnormalities induced by infection. In fact, the circulatory changes
in hyperdynamic sepsis appear to be an adaptive response to the underlying
metabolic dysfunction. Cardiac output is high and systemic vascular
resistance low in hyperdynamic septic shock. However, splanchnic
vasoconstriction is pronounced even in the absence of systemic hypotension
and even though systemic vascular resistance is reduced. Expansion of
circulating blood volume can occur through either transcapillary refill or fluid
resuscitation. Due to the ongoing inflammatory mediator-induced increases in
capillary permeability and continued loss of intravascular volume, exogenous
volume resuscitation must be provided to restore venous return and
ventricular filling.

223 Which of the following statement(s) is/are true concerning tumor


necrosis factor (TNF)?
a. TNF is a product of activated macrophages secreted in response to contact
with endotoxin or lipopolysaccharide, antibody complexes, or inflammatory
stimuli
b. The liver and gut appear to be a major source of TNF following
hypoperfusion
c. Circulating levels of TNF correlate well with severity of tissue injury in
shock
d. Recently completed clinical trials of anti-TNF antibody in septic patients
shows a marked improvement in survival
Answer: a, b
Tumor necrosis factor (TNF), a protein product of activated macrophages, is
secreted in response to contact with endotoxin or lipopolysaccharide,
antibody complexes, or other inflammatory stimuli. Elevation of serum levels
of TNF have been reported shortly after experimental trauma and shock,
however, documentation of elevated circulating levels of TNF in human
shock is less clear. Furthermore, circulating levels of TNF cannot be
correlated with severity of tissue injury or shock. This variability is thought
to be due to rapid clearance and uptake by membrane receptors and by
soluble membrane receptors that are released from multiple cells following
stress and injury. Following hypoperfusion the liver and gut appear to be the
major source of TNF that is rapidly cleared but responsible for inducing
hepatocyte changes following shock. The release of breakdown products and
escape of bacterial and endotoxin through the damaged mucosal barrier of the
gut following shock allows or induces activation of tissue-fixed macrophage
(Kupffer cell) of the liver which then produces secondary inflammatory
mediators contributing to the post-resuscitation clinical response and
inflammatory mediator activation seen in the systemic inflammatory response

syndrome.
TNF is central to inflammatory response, particularly in sepsis and following
endotoxemia or bacteremia. TNF also induces secondary inflammatory
responses through direct interaction with specific membrane receptors, TNFr. Treatment with anti-TNF antibody in the experimental setting protects
animals from the deleterious effects of lethal bacteremia and endotoxemia.
However, recently completed clinical trials in septic patients utilizing
infusion of monoclonal antibodies to the TNF molecule have shown no
overall survival benefit.

224 Which of the following statement(s) is/are correct concerning the


immunoinflammatory response to shock?
a. The anaphylactoxins, C3a and C5a, are products of activation of only the
classical pathway of the compliment cascade
b. Eicosanoids, such as prostaglandins are stored in platelets and endothelial
cells and released in response to inflammatory stimuli
c. Thromboxane and PGI2 have similar effects
d. Platelet-activating factor can be released by both circulating and fixed
tissue cells
Answer: d
Inflammatory mediators have recently been recognized as playing a
significant role in the clinical manifestations and progression of shock and
the development of subsequent complications. These mediator systems
function primarily as parcrine and autocrine agents in the local environment
and are not usually detectable systemically. The over-expression and systemic
dissemination of these mediators produces the toxic autodestructive processes
underlying multiorgan failure syndrome with attendant high mortality. The
compliment cascade is activated in shock and tissue injury through both the
classical and alternative pathways. Activation of either pathway results in
generation of the anaphylatoxin, C3a and C5a, soluble products with potent

systemic hemodynamic effects. The eicosanoids, which include the


prostaglandins and leukotrienes are formed acutely from arachidonic acid
released from the membrane phospholipid by phospholipase A2. Eicosanoids
are not stored in any measurable level and are generated as needed from
readily available arachidonic acid in response to various inflammatory
phenomena. Platelets, white cells, and endothelial cells are a rich source of
these compounds. Thromboxane (TXA2) is the major arachidonic acid
metabolite elaborated by platelets. TXA2 induces intense vasoconstriction,
platelet aggregation and degranulation, neutrophil margination in the
microcirculation and bronchial constriction. PGI2, the major arachidonic acid
metabolite formed by endothelial cells, serves a check against actions of
TXA2. PGI2 is a vasodilator and a potent inhibitor of platelet aggregation.
Platelet aggravating factor is a potent phospholipid mediator released by
neutrophils, platelets, macrophages and endothelial cells in response to
ischemia, tissue injury and sepsis. Its effects include decreased cardiac
function, increased pulmonary vascular resistance, bronchoconstriction,
peripheral vasodilatation, and increased vascular permeability.

225 Which of the following physical findings are associated with the various
classes of hemorrhagic shock?
a. Mild shock (< 20% blood volume): Pallor, cool extremities, diminished
capillary refill and diaphoresis b. Moderate shock (20%40% blood volume):
All of the above plus tachycardia and hypotension c. Severe shock (> 40%
blood volume): Systemic hypotension, changes in mental status, tachycardia,
oliguria
d. All of the above
Answer: a, c
PHYSICAL FINDINGS IN HEMORRHAGIC SHOCK*
Moderate
Mild (<20% (20%-40% Severe(>40%

Blood Volume) Blood Volume) Blood Volume)


Pallor Pallor Pallor
Cool extremities Cool extremities Cool extremities
Diminished capillary Diminished capillary Diminished capillary
refill refill refill
Diaphoresis Diaphoresis Diaphoresis
Collapsed Collapsed Collapsed
subcutaneous subcutaneous subcutaneous
veins veins veins
Tachycardia Tachycardia
Oliguria Oliguria
Postural Hypotension
hypotension Mental status
changes
* Alcohol or drug intoxication may alter physical findings.

226 A 68-year-old male who underwent a repair of an abdominal aortic


aneurysm 5 days ago, develops tachycardia, tachypnea, hypotension with
cool, pale, mottled cyanotic extremities. He is agitated and complains of
shortness of breath. Which of the following statement(s) is/ are correct
concerning his diagnosis and management?
a. Myocardial ischemia secondary to preexisting coronary artery disease is
most likely the underlying cause of this problem
b. Invasive hemodynamic monitoring with a Swan-Gantz catheter will
demonstrate a low cardiac output, a high systemic vascular resistance, and
elevated cardiac filling pressures
c. The use of morphine sulphate and nitrates should be part of the initial
management
d. The primary pharmacologic treatment involves the use of moderate doses
of inotropic agents

e. Afterload reduction with nitroprusside is absolutely contraindicated


Answer: a, b, d
Intrinsic cardiogenic shock results from failure of the heart as an effective
pump. Coronary artery disease is the most common cause of myocardial
insufficiency, but contractile dysfunction may also rise as a consequence of
cardiomyopathy, myocarditis, or metabolic abnormalities. Invasive
hemodynamic monitoring often establishes a specific nature of shock and
allows appropriate treatment to be delivered in an effective and expedient
manner. Hemodynamic findings consistent with cardiogenic shock include a
low cardiac output and high systemic vascular resistance, with elevated
cardiac filling pressures. The initial measures in the management of
cardiogenic shock include the administration of supplemental oxygen,
mechanical ventilation (as needed), and appropriate treatment of
dysrhythmias. Hypotension usually precludes the use of morphine sulfate and
nitrates, drugs typically used in simple congestive heart failure to alleviate
cardiac pain and ameliorate pulmonary vascular congestion. The use of betaadrenergic agonists such as dopamine and dobutamine, in moderate doses,
offers positive inotropic support without excessive alpha-adrenergic activity.
Increasing the inotropic state of the heart shifts the entire Starling curve
upward, resulting in increased cardiac output for each level of cardiac filling.
Afterload reduction may prompt increases in cardiac output through
decreases in resistance to flow. The use of nitroprusside or other dilators
requires relative blood pressure stability and close hemodynamic monitoring.
Infusion of afterload-reducing agents can be administered in conjunction with
inotropic support.

227 Which of the following statement(s) is/are true concerning ischemia


reperfusion injury?
a. During ischemia, ATP degradation results in increased plasma and
intracellular levels of hypoxanthine and xanthene

b. Oxygen free radicals such as the superoxide radical are involved in the
expression of the proinflammatory phenotype of endothelial cells,
macrophages and neutrophils
c. The intracellular adhesion molecule-1 (ICAN-1) contributes to injury and
disruption of the endothelial lining, with extensive capillary leak and
resultant interstitial edema
d. Animal models have demonstrated that passive immunization with
antibodies to neutrophil adhesive complex lessen the ischemic/reperfusion
microvascular injury
Answer: a, b, c, d
During the ischemia and hypoperfusion phase, degradation of ATP stores
essential to maintain cell integrity and significant loss of diffusible
intracellular adenine neuclotides occurs. As ATP further degrades there is an
elevation in plasma and intracellular levels of hypoxanthine and xanthene
which upon restoration of perfusion and reoxygenation are catalyzed by
xanthine oxidase resulting in the formation of superoxide radicals. These
radicals plus others such as hydrogen peroxide and hydroxyl radical are
generated and lead to endothelial and parenchymal cell injury through
membrane lipid peroxidation and activation of critical enzymes. These
radicals have also been shown to be involved in the expression of
proinflammatory phenotype endothelial cells and on macraphages and
neutrophils. The proinflammatory phenotype of the endothelium includes
procoagulant activity and the expression of adhesion molecules on the
membrane surface, including the intercellular adhesion molecule-1 (ICAN-1)
and the selectins. The subsequent adhesion of activated neutrophils to the
endothelial leads to an explosive oxidative burst producing additional radicals
and extensive release of proteolytic enzymes leading to injury and disruption
of the endothelial lining, extensive capillary leak, and massive interstitial
edema. Passive immunization of animals with monoclonal antibodies to
either the neutrophil adhesive complex or the endothelial selectins
dramatically lessens ischemia/reperfusion microvascular injury.

228 Which of the following statement(s) is/are true concerning the


physiology of the microvascular system?
a. Filtration of capillary fluid into the interstitial and the subsequent
reabsorption is influenced by Starlings law of ultrafiltration
b. The most important variable controlling blood to a capillary bed is the
length of the vessel
c. Most of the resistance to systemic blood flow occurs at the arteriolar level
d. Adrenergic vasoconstriction can arrest blood flow to an entire capillary bed
Answer: a, c, d
Exchange of material between the vascular space and the cell of various
tissues via the interstitial space is essential for organ viability and occurs at
the capillary level. The filtration of capillary fluid into the interstitium and its
subsequent reabsorption into the post capillary venule is governed by
microvascular permeability in conjunction with the balance between
hydrostatic and oncotic pressures. The relation of these forces to one another
(and their net effects) are illustrated by what is termed Starlings law of
ultrafiltration. In normal circumstances, a net filtration from capillary to
interstitium is effected by a relatively higher capillary hydrostatic pressure,
whereas net reabsorption from the interstitium back into the post capillary
venule occurs as hydrostatic pressure falls and oncotic forces predominate.
Although the mechanisms controlling blood flow to the capillary bed are
complicated and vary among the different tissues, certain concepts are useful.
Poiseuilles law describes the relation between flow of fluid through a tube
and the tube length and radius, the fluid viscosity, and the pressure gradient
between ends of the tube. The radius of the tube (or vessel) is the single most
important variable, because flow is proportional to the radius to the fourth
power. Vasoconstrictive and vasodilatory influences directly impact local
blood flow, as well as flow to other tissues through secondary effects on the
systemic pressure. This secondary effect of peripheral vasoconstriction

maintains the pressure gradient for central perfusion of the heart and brain.
Systemic blood flow meets most of its resistance at the arteriolar level. While
the individual capillary radius is significantly smaller, the vast number of
capillaries offers less total resistance. The vascular smooth muscle in
arterioles has both a-and b- adrenergic receptors. Alpha stimulation affects
vasoconstriction where beta stimulation affects vasodilatation. The efferent
sympathetic fibers innervating the precapillary resistance vessels and the
venous capacitance vessels release norepinephrine on stimulation, which
induces smooth muscle contraction and narrowing of the caliber of the
vessels. These contractions are potent enough that blood flow to entire
capillary beds can be arrested by adrenergic vasoconstriction.

229 Which of the following statement(s) is/are true concerning the effects of
MOFS?
a. Pulmonary dysfunction tends to arise early and may resolve within 7 to 10
days
b. Unless the precipitating insult has prompted oliguric acute tubular
necrosis, renal function tends to be maintained early in the course of MOFS
c. Although hepatic dysfunction is common with MOFS, the GI tract plays
little role in this process
d. Intercurrent nosocomial infection, most commonly pulmonary, is a
common complication providing a second hit to the patient
Answer: a, b, d
Pulmonary dysfunction typically arises early in the development of systemic
inflammation and may represent mild relatively localized acute lung injury or
it may be a prelude to fulminant ARDS. The lung injury, and associated
dysfunction, may resolve over the initial 7 to 10 days or persist, depending on
the ongoing pathologic process. Many times a second hit such as a
nosocomial infection, which is most commonly pulmonary, is a complication
which can frequently worsen the pulmonary condition. Renal function tends

to be maintained early in the course unless the precipitating insult has been
prompted by a sudden oliguric acute tubular necrosis. With persistent
activation and inflammatory mediators, glomerular filtration falls and the
development of oliguric or polyuric renal failure marks the gradual transition
into MOFS. Gastrointestinal abnormalities include ileus, stress ulceration,
diarrhea, and mucosal atrophy. Breakdown of the mucosal barrier allows
translocation of bacteria and endotoxin. Hepatic dysfunction is marked by
progressive rise in serum bilirubin levels after a latent period of several days.

230 Which of the following statement(s) is/are true concerning hypoadrenal


shock?
a. In the United States, idiopathic adrenal atrophy (Addisons disease) is the
most common cause
b. Laboratory abnormalities include hyponatremia, hypochloremia, and
hyperkalemia
c. Fever may be seen with hypoadrenal shock
d. ACTH stimulation test is the diagnostic test of choice to confirm
hypoadrenal shock
Answer: b, c, d
Shock of a dramatic nature, poorly responsive to resuscitation, may develop
as a consequence of adrenal insufficiency. In this country, adrenal
insufficiency most commonly arises as a consequence of the chronic
therapeutic administration of high doses exogenous corticosteroids causing
adrenal suppression. Other causes include idiopathic adrenal atrophy
(Addisons disease), tuberculosis, metastatic disease, bilateral hemorrhage,
and amyloidosis. The stress of illness, operation, or trauma typically requires
that the adrenal glands secrete cortisol in excess of that required in the
nonstressed state (approximately 34 fold). Insufficiency not otherwise
apparent may manifest itself only after major physical stress. Findings
associated with adrenal insufficiency include weakness, fatigue, anorexia,

abdominal pain, nausea, vomiting, and weight loss. Surgical patients with
significant adrenal insufficiency need not present with the above findings.
More typical is the development of refractory shock, frequently with
hyperthermia, in the course of injury or illness. Hypotension may be dramatic
despite massive volume resuscitation and pressor support. Laboratory
findings suggesting hypoadrenalism include hyponatremia, hypochloremia
and hyperkalemia. The diagnosis of adrenal insufficiency may be confirmed
or excluded by means of an ACTH stimulation test. A significant major
cortisol response should be elicited by ACTH administration.

231 Which of the following statement(s) is/are correct concerning the


cardiovascular response to shock?
a. Changes in cardiac contractile function shift the Frank Starling curve up
and down
b. Venoconstriction from skeletal muscle is a significant contributor to the
restoration of blood volume with shock
c. Arterial vasoconstriction affects all vascular beds equally
d. The total circulating blood volume is equally split between the arterial and
venous system
Answer: a
Central in the general cardiovascular response to shock is the action of the
heart itself. The principle determinants of cardiac function in the normal heart
are the volume of blood available for the heart to pump (preload), the systolic
contractile capability, and the diastolic filling of the ventricles. In
hypovolemia, the two dynamic variables of cardiac function, ventricular
filling and myocardial contractility remain paramount and determine the
stroke volume. The product of stroke volume and heart rate in turn
determines the cardiac output. Increases in ventricular end-diastolic volume,
reflecting venous return, cause ventricular distention. Ventricular distention in
turn produces increased volume output with each stroke, the Frank Starling

mechanism. Contractile function may vary independent of volume status.


Changes in the contractile function shift the Starling curve up and down,
producing increases or decreases in stroke volume for any given end-diastolic
volume. A fundamental requirement for cardiovascular function is adequate
cardiac filling, and cardiac output cannot exceed venous return. The venous
system contains nearly two-thirds of the total circulating blood volume,
including 20% to 30% within the splanchnic venous system. Most of this
volume resides in small veins, which comprise the bulk of venous
capacitance. The venous system, especially that of the splanchnic circulation,
becomes important in the physiologic compensation to hypoperfusion
because it serves as a dynamic reservoir for the autoinfusion of blood volume
involving both active and passive mechanisms. The splanchnic circulation
makes major contributions to the maintenance of venous return, therefore, it
is likely that sympathetic venoconstriction is responsible for a portion of the
blood mobilized from the splanchnic venous circulation. Sympathetic
mediated venoconstriction in skin and skeletal muscle is probably not as
significant as a source of blood volume. Selective vasoconstriction occurs in
response to alpha adrenergic receptor stimulation with increased sympathetic
activity in shock. Sympathetic stimulation does not cause significant
vasoconstriction of either cerebral or coronary vessels, with normal blood
flow maintained in these circulations. Blood flow to the skin is sacrificed
early, followed by that to the kidneys and splanchnic viscera.

232 Which of the following statement(s) is/are true concerning


pharmacologic agents used in the treatment of shock?
a. The primary difference between dopamine and dobutamine is the absence
of significant a adrenergic activity
b. The renal and mesenteric vasoconstrictive effects of norepinephrine
complicate and sometimes restrict its use
c. The apparent paradoxical use of vasodilators, such as nitroprusside, in
shock is indicated as a means to augment cardiac function

d. Isoproterenol with its potent b-adrenergic effect, is a particularly useful


agent in the treatment of all forms of shock
Answer: a, b, c
Therapeutic adjustments of intravascular volume (preload) and systemic
vascular resistance (afterload) form the basis of the treatment strategies for all
forms of shock. Optimal volume resuscitation should precede measures to
augment to contractile function of the heart. Inotropic agents are used in
shock when there is inadequate cardiac output despite adequate circulating
blood volume. Dopamine and dobutamine are often times first line agents in
the pharmacologic treatment of shock. Dopamine, at low doses, stimulates
dopaminergic receptors producing renal arteriolar vasodilatation with
associated increases in renal blood flow, urine output, and sodium excretion.
At moderate doses, stimulation of cardiac b-receptors produces increases in
contractility and cardiac output with little effect on heart rate or blood
pressure. At higher doses, peripheral vasoconstriction from increasing a
activity becomes more pronounced, prompting significant increases in
vascular resistance and blood pressure. Dobutamines predominant effect is
an increasing cardiac contractility with lesser increases in heart rate. Some
reduction of peripheral vascular resistance may also occur. When compared
to dopamine, dobutamine produces less peripheral vasoconstriction and less
chronotropic response. Norepinephrine exerts both a and b-adrenergic effects,
with a effects being evident at lower infusion rates and a effects more
prominent at high doses. The major use of norepinephrine in current practice
is in the patient with hypotension that persists despite appropriate volume
resuscitation and the use of inotropic agents. Renal and mesenteric
vasoconstrictive effects of norephinepherine complicate its use, especially
when support is needed for significant periods of time. Isoproterenol is a
potent b-adrenergic agent. With isoproterenol, myocardial oxygen demand is
increased and diastolic coronary feeling is limited by tachycardia or
diminished diastolic pressure. Indications for the use of isoproterenol are
fairly limited, because agents with fewer adverse effects have become

available.
Vasodilators are used to augment cardiac function through optimization of
ventricular filling pressures (preload) and systemic vascular resistance
(afterload) both of which reduce demands on the myocardium. Decreases in
afterload prompt increases in cardiac output and venodilatation contributes to
decreases in pulmonary venous pressure and central venous pressure.
Hypotension, however, may develop therefore patients must have careful
constant monitoring of arterial pressure and repeated hemodynamic
measurements with a pulmonary artery catheter.

233 Which of the following statement(s) is/are true concerning the treatment
of MOFS?
a. Prevention and therapy of MOFS requires control of the infectious or
inflammatory source
b. Restoration of normal clinical parameters such as blood pressure, pulse
rate, and urine output ensures optimal resuscitation in most patients
c. Branch chain amino acids play and important role in the nutritional support
of the patient
d. Because of the nature of gut injury, total parenteral nutrition is preferred
for most patients with MOFS
Answer: a, c
The therapy of MOFS is directed towards interrupting the involving
pathophysiologic process and providing an optimal physiologic environment
for healing and recovery. Fundamental concerns are control of the source of
infection, inflammation or instability; restoration of microcirculatory blood
flow and oxygen transfer, and the institution of optimal supportive care. Both
the prevention and therapy of MOFS, therefore, requires source control and
restoration of adequate profusion. Resuscitation efforts are directed toward
restoration of adequate microcirculatory blood flow in all organ systems.
Restoration of normal clinical parameters such as blood pressure, pulse rate,

urine output, and acid-base balance does not ensure optimal resuscitation.
The physiologic endpoint that most closely corresponds with adequate
microcirculatory flow is the level of cardiac output and the oxygen delivery
at which oxygen consumption and lactate production remain independent of
flow.
The importance of metabolic support in the patient with MOFS cannot be
overemphasized. The malnutrition of MOFS is markedly different than that
of starvation and the nutritional requirements also differ. If optimal quantities
of appropriately formulated amino acid solutions are given, protein synthetic
rates can approach catabolic rates and the goal of nitrogen balance can be
achieved. Formulas rich in branch chain amino acids appear to be more
efficient in promoting nitrogen retention and minimizing urea production.
Whenever feasible, enteral feeding is preferred over TPN because evidence
suggests that bacterial translocation from the gut can be limited through the
use of enteral feeds. Enteral absorption and processing of nutrients appears
superior to TPN and lessens overall complications.

234 Which of the following statement(s) is/are true concerning the


multiorgan failure syndrome (MOFS)?
a. Changes in the splanchnic and pulmonary microcirculation are critical to
the development of MOFS
b. Tissue fixed microphages, including the liver Kupffer cell, have little role
in the development of MOFS
c. MOFS represents systemic consequences of loss of homeostatic control of
local inflammation and microcirculatory hypoperfusion
d. The nature of MOFS is highly dependent upon the etiology of the
underlying problem
Answer: a, c
The nature of multiorgan failure syndrome (MOFS) is that of a diffuse
cellular injury, developing systemically as a consequence of losing

homeostatic control of local inflammation and microcirculatory


hypoperfusion. Endothelial injury, platelet aggregation and activation of
macrophages and neutrophils occur, and the clotting, fibrinolytic, kinin, and
complement cascades are activated, along with the release of potent
inflammatory cytokines. The effects of shock, resuscitation, and reperfusion,
and the subsequent development of MOFS appear to be critically dependent
on changes in the splanchnic and pulmonary microcirculations. These
vascular beds appear to be major sites of activation of subsequent
inflammatory mediator production that underlies the diffuse systemic
inflammatory response. Extensive activation of the liver Kupffer cell and
release of inflammatory mediators coupled with the ongoing release of
activated neutrophils and by-products of activated gut macrophages is
responsible for the injury to the pulmonary microcirculation and secondary
induction of alveolar macrophage and additional inflammatory mediator
systems. Excessive and persistent macrophage activation plays an essential
role in MOFS and is hypothesized to represent the penultimate step in a series
of continuous immuno-inflammatory stimulatory events, including local
hypoxia, exposure to bacteria and toxins, and mediator release from localized
areas of inflammation. When infection is the underlying or major
contributing process, the diffuse inflammatory response develops
independently of the specific type of microorganism. In noninfectious cases,
the response also appears independent of the specific underlying cause.

235 Invasive hemodynamic monitoring using a Swan-Gantz catheter is


essential in the optimal management of patients in shock or those suffering
post-shock sequelae. Which of the following physiologic characteristics are
associated with the various forms of shock?
a. Hypovolemic shock: Decreased pulmonary capillary wedge pressure
(PCWP), decreased cardiac output, increased systemic vascular resistance
(SVR)
b. Cardiogenic shock: Increased PCWP, decreased cardiac output, decreased

SVR
c. Septic shock (hypodynamic): Decreased cardiac output, increased SVR
d. Neurogenic shock: Decreased PCWP, increased cardiac output, decreased
SVR
Answer: a, c
236 Which of the following statement(s) is/are true concerning the
relationship between cardiac function and effective blood volume?
a. A pulmonary capillary wedge pressure of 510 rules out fluid overload as a
cause of pulmonary edema
b. A shift to the right in the Frank-Starling curve is associated with
compromised cardiac function
c. Dilutional anemia may contribute to tachycardia even though blood
volume and filling pressures are normal
d. The sole purpose of a pulmonary artery catheter is to measure pulmonary
artery pressure and cardiac output
Answer: b, c
Although physical findings are often adequate to establish a diagnosis and
institute management of cardiac failure, direct measurement of filling
pressures of the right heart (central venous pressure) or the left heart
(pulmonary artery pressure) may be required. Placement of a pulmonary
artery catheter allows us to measure cardiac output by thermodilution and,
more importantly, to sample mixed venous blood for saturation
measurements which tell us the ratio between systemic oxygen delivery and
oxygen consumption. From all of these measurements we can determine if
cardiac output is normal for the level of filling pressure of the left ventricle,
or if contractility is decreased. In the latter case, cardiac output will be lower
than predicted for a given level of filling pressure. In the Frank-Starling
curve, if the patient is to the right of the normal range, then cardiac function
is compromised either because of valvular disease, extrinsic pressure such as

pericardial tamponade, or more commonly, a decrease in contractility. If


cardiac function and anatomy are normal, then blood volume, filling pressure
and cardiac function are related to the Starling curve. The intake and output
of fluid and salt is autoregulated to maintain the filling pressure of the left
ventricle around 10 mm Hg. Extracellular fluid expansion is usually
associated with normal blood volume. Gross expansion of extracelluar space
results in deleterious effects if tissue edema can and often do exist with
perfectly normal blood volume. In other words, a pulmonary capillary wedge
pressure of 510 does not rule out fluid overload as a cause of pulmonary or
GI dysfunction. In critically ill patients, the fear of hypotension and effect of
perfusion usually results in infusion of intravenous salt and water in
quantities which exceed losses. Consequently, most patients in the Intensive
Care Unit have anemia, dilutional hypoproteinemia, and a compensatory
increase in cardiac output. In response to anemia, these patients are
tachycardic, even though blood volume is normal, filling pressures are
normal, and total body extracellular fluid is excessive.

237 Which of the following statement(s) is/are true concerning methods of


nutritional support?
a. Optimal results for enteral feedings are achieved with approximately half
of calories supplied as carbohydrate and half as fat
b. Diarrhea is the most common complication of enteral feeds and is due to
the high osmolarity of the carbohydrate components
c. The hyperosmolar nature of parenteral fat solutions requires central venous
administration
d. Approximately 2550% of calories should be provided as fat emulsion in
patients receiving total parenteral nutrition
Answer: a, b, d
Most formulas for enteral feeding range from 1.0 to 2 cal/ml and include 3 to

7% protein. Most of the calories are supplied as glucose or sucrose, so that


the solutions have a very high osmolarity. Cramps or diarrhea can result when
these high osmolar solutions are placed into the stomach or intestine.
Diarrhea is the major complication with most tube feeding formulas. Diarrhea
can be minimized by the use of starch or fat as an energy source in tube
feedings. This can be supplied as part of the commercial preparation or added
in the form of medium chain triglycerides or other oils. The best results are
usually achieved by supplying approximately half the calories as
carbohydrate and half as fat. In patients receiving total parenteral nutrition,
energy source is provided as carbohydrate, fat, and amino acid solutions.
Parenteral feeding with carbohydrate is limited by the sclerotic effect of
hyperosmolar solutions on veins. Fat is a more efficient energy source and
can be given through peripheral veins in concentrations of either 10 or 20%.
Most intensivists favor supplementing standard total parenteral nutrition
solution with intravenous fat to provide at least 100 grams of fat emulsion
each week to preclude fatty acid deficiency. Giving up to 25 to 50% of
calories each day as fat emulsion may optimize the delivery of this caloric
delivery.

238 Which of the following statement(s) is/are true concerning the


autoregulation necessary to maintain oxygen consumption and oxygen
delivery?
a. A change in oxygen consumption is followed by a proportionate change in
oxygen delivery
b. A change in oxygen delivery is followed by a change in oxygen
consumption
c. Increases in oxygen delivery are due solely to an increase in cardiac output
d. The normal ratio of oxygen delivery to consumption is 2:1
Answer: a

The relationships between oxygen consumption and oxygen delivery


represent one of the most interesting regulation systems in homeostasis. First
of all, if one of the three components of oxygen delivery is abnormal,
endogenous mechanisms regulate the other two until normal oxygen delivery
has been restored. The various combinations of compensatory mechanisms
supply adequate oxygen for systemic metabolism through a wide range of
variations in oxygen delivery. When there is a change in oxygen
consumption, there is a proportionate change in oxygen delivery, which
occurs almost immediately, mediated completely by a change in cardiac
output. Conversely, a primary change in oxygen delivery is not followed by
any change in oxygen consumption. The normal ratio of oxygen delivery to
consumption is approximately 5:1.

239 Which of the following statement(s) is/are true concerning O2 venous


saturation monitoring?
a. The normal saturation of mixed venous blood is 50%
b. Mixed venous blood obtained for saturation monitoring can be obtained
from any peripheral vein
c. If arterial blood is fully saturated, the saturation of mixed venous blood is
80%
d. In less than fully saturated blood, the difference between arterial and
venous saturation corresponds to oxygen extraction
Answer: c, d
The relationship between oxygen delivery and oxygen consumption is
reflected in the amount of oxygen in venous blood. Under normal
circumstances, oxygen delivery is 1000 cc/min and oxygen consumption is
200 cc/min. The amount of oxygen extracted is 20% of that delivered, and
80% of oxygen is still present in venous blood returning to the heart. Usually
arterial blood is fully saturated, and under normal circumstances, the

saturation of mixed venous blood (SVO2) will be 80%. This measurement


must be made in mixed venous blood since the relative extraction of organs
served by the superior and inferior vena cava and coronary sinus are quite
different. As long as arterial blood is fully saturated, this observation holds
true regardless of the absolute level of oxygen consumption or oxygen
delivery. If the arterial blood is less than fully saturated, the difference
between arterial and venous saturation corresponds to the oxygen extraction,
hence the oxygen delivery/oxygen consumption ratio.

240 Which of the following statement(s) is/are true concerning oxygen


kinetics in a critically ill, febrile patient?
a. Oxygen consumption will likely exceed three times normal
b. The high cardiac output and pulse rate are designed to increase oxygen
delivery
c. The hyperdynamic response may actually increase oxygen delivery to
exceed the increase in oxygen consumption
d. The patient can maintain adequate compensation as long as the oxygen
delivery/oxygen consumption rate is greater than 2:1
Answer: b, d
In critically ill patients oxygen consumption may be elevated or depressed,
but slight to moderate elevations in oxygen consumption is the most common
abnormality in critically ill patients. Oxygen consumption will be elevated in
proportion to the amount of inflammation. A febrile patient with significant
signs of septic toxicity will typically have an oxygen delivery at 1.5 to 2
times normal. It is very unusual for a critically ill patient to experience
oxygen consumption greater than twice normal. This occurs only in situations
of severe muscular exercise such as seizures or tetanus. During
hypermetabolism, a change in oxygen consumption is followed promptly by a
proportionate change in oxygen delivery. Hence, it is normal for a

hypermetabolic patient to have a high cardiac output and pulse rate. Rarely
the hyperdynamic response exceeds the increase in oxygen consumption,
reflected in a ratio higher than 5:1 and venous saturation greater than 80%.
Some patients cannot mount an increased oxygen delivery in response to
increased oxygen consumption because of the combination of hypoxemia,
anemia, and myocardial failure. If this occurs, then the oxygen
delivery/oxygen consumption ratio will be less than 5:1. The patient will
compensate for this by increased oxygen extraction, however, and the patient
will remain stable as long as the ratio is greater than 2:1.

241 Which of the following statement(s) is/are true concerning the treatment
of pulmonary interstitial edema?
a. Diuresis and blood transfusion is a valuable step
b. Salt-poor albumin leaks through the capillaries and worsens the condition
c. Mannitol is contraindicated as a diuretic in this clinical situation
d. Isoproterenol is a poor choice as an ionotropic agent
Answer: a
Treatment of pulmonary edema has two important goals, the first is to
improve oxygenation if it is impaired, and the second is to minimize fibrosis
and bacterial infection, which often accompany pulmonary edema caused by
capillary injury. The treatment of interstitial edema is to maintain the
hydrostatic pressure as low as compatible with adequate cardiac output and to
raise the oncotic pressure selectively in the vascular space. These measures,
combined with fluid restriction and diuresis, will decrease the amount of
pulmonary edema. Since it is desirable to maintain filling pressures of the left
ventricle as low as possible while maintaining a good cardiac output,
inotropic drugs to improve left ventricular contractility are helpful.
Isoproterenol or dopamine should be used, with serial cardiac output and
filling pressure measurements. The first step in decreasing pulmonary edema

is to decrease the pulmonary capillary hydrostatic pressure as low as is


compatible with an adequate cardiac output. This is done by diuresis and
fluid restriction. As the patient falls behind in blood volume, signs of
hypovolemia may appear. Blood volume is then replenished with a fluid that
stays in the vascular space. Packed red cells are ideal for this application.
When the hematocrit is normal, concentrated salt-poor albumin should be
used. This hyperoncotic fluid replenishes the blood volume by attracting
interstitial fluid from throughout the body into the vascular space and
supplements diuresis. This technique is useful even in the septic patient who
may have increased capillary permeability and may loose albumin from the
vascular space at a rapid rate. Even if albumin leaks out, the short term
effects of expanding blood volume and decrease in edema will appear.

242 Which of the following statement(s) is/are true concerning the


pathophysiology of gas exchange?
a. Hypoventilation in relation to perfusion can result in an oxygen saturation
of less than 100%
b. Diffusion block and / mismatch can almost completely be overcome by
breathing 100% oxygen
c. Transpulmonary shunting does not occur under normal circumstances
d. The normal arterial oxygen saturation should be 100%
Answer: a, b
Under normal conditions, red blood cells in the pulmonary capillaries become
fully saturated and oxygen dissolves in plasma resulting in blood PO2 of 100
and O2 saturation of 100%. This equilibration may be disturbed by
hypoventilation in relationship to the perfusion (/ mismatch), diffusion block
caused by interstitial fibrosis, or perfusion of nonventilated alveoli. Diffusion
block and / mismatch can almost be completely overcome by breathing 100%
oxygen, hence hypoxemia during exposure to high alveolar PO2 is caused by

total / mismatch, so-called transpulmonary shunting or venous ad mixture.


Under normal circumstances, about 5% of the blood entering the left atrium
has been shunted away from the pulmonary capillaries, either as the result of
bronchial nutritive blood flow or through thebesian veins opening directly
into the left side of the heart. This phenomenon, combined with a normal
minor / mismatch associated with breathing at rest and positional changes in
pulmonary blood flow, result in the fact that normal arterial PO2 is 90100
mm Hg and the normal O2 saturation is 98%.

243 Which of the following statement(s) is/are true concerning CO2 transfer
in the lung?
a. Carbon dioxide excretion is a direct function of alveolar ventilation
b. Normally end tidal CO2 should be identical to PaCO2
c. The gradient between end tidal and arterial CO2 can be an indirect measure
of nonperfused alveoli
d. Positive pressure ventilation under normal airway pressures creates a
significant end tidal PaCO2 gradient
Answer: a, b, c
The amount of carbon dioxide excretion is directly related to alveolar
ventilation. While oxygenation is a function of matching blood flow to
alveoli, carbon dioxide excretion is a direct function of ventilation or
hyperventilation of alveoli with some blood flow. Normally the end tidal
CO2 represents mixed alveolar gas which is at equilibrium with pulmonary
capillary blood, hence with arterial blood. Therefore, the end tidal CO2 and
the PaCO2 should be identical. End tidal CO2 measurement is a very useful
continuous measurement of PaCO2 which can be used as a monitor when the
lung is normal, as in ventilator weaning. Furthermore, the gradient between
end tidal and arterial CO2, when it is large, acts as an indirect measure of
nonperfused alveoli and/or compression volume. In patients who are

ventilated with positive pressure ventilation, a significant end tidal PaCO2


gradient occurs only when peak airway pressures are very high (over 30 cm
H2O) and the compression volume is a significant component of each
exhaled breath.

244 Which of the following statement(s) is/are true concerning the


assessment of protein reserve?
a. Conventional serum proteins such as albumin and globulin are early
indicators of malnutrition
b. The total lymphocyte count reflects immune status and not nutrition
c. Antigen skin testing reflects patient immunity and not nutrition
d. Measurement of urea excretion in urine can be used as a measurement of
protein breakdown
Answer: d
Since protein is the functional and structural chemical of the body, most
nutritional assessment techniques are estimates of protein reserves. The actual
nitrogen balance can be measured by measuring the amount of nitrogen
excreted. This is most conveniently done by measuring the amount of urea
excreted in the urine, assuming that urea constitutes 85% of the total nitrogen
excretion. Knowing nitrogen excretion, the amount of protein catabolized can
be estimated and compared with the amount of protein ingested by the
patient. Indirect assessments of protein reserves are based on single
measurements of body substances that are dependent on rapid protein
synthesis for maintenance of normal levels. Conventional serum proteins
such as albumin and globulin are not affected by malnutrition until it is very
severe. Proteins such as prealbumen and transferrin, which turn over more
rapidly, are better indicators of protein status. Lymphocytes are rapidly
destroyed and protein is required for the formation of new cells.
Consequently, the absolute lymphocyte count is a useful measure of the status

of protein reserves. The lymphocyte count is considered by some the best


single static measurement characterizing nutritional status. Protein is also
required for synthesizing the cells and mediators involved in skin test
reactivity. Although skin test reactivity is a manifestation of lymphocytemediated immunity, its usefulness in patient assessment is probably that of
assessment of the inflammatory response than lymphocyte activity per se.
Some chronically and acutely malnourished patients convert from reactive to
anergic, and reactivity can be restored by nutritional repletion.

245 Useful steps to optimize systemic oxygen delivery include:


a. Maintaining mean arterial blood pressure between 50 and 90 mm Hg
b. Optimizing PEEP levels by monitoring mixed venous saturation
c. Turning the patient prone
d. Sedation or paralysis
Answer: a, b, c, d
Optimizing systemic oxygen delivery in relationship to oxygen requirement
is the primary goal of management. Improving oxygenation of the blood
itself by improving alveolar inflation is only one of the steps in optimizing
oxygen delivery. Equally or more important are treating anemia and
optimizing cardiac output. Cardiac output should be optimized to maintain
delivery of four to five times consumption. In general, this means avoiding
those factors which decrease cardiac output, rather than actively trying to
increase cardiac output. Blood pressure should be maintained high enough to
provide coronary perfusion (over 50 mm Hg mean pressure) but not so high
as to limit left ventricular function (over 90 mm Hg mean arterial pressure).
Alveolar collapse is treated by cleaning the airways, avoiding 100% oxygen,
and moving fluid from the lung or chest, and finally by the use of positive
end exploratory pressure to hold open those alveoli which have been opened
by other measures. The optimal level of PEEP is that level that maintains

arterial oxygenation but does not decrease venous return or cardiac output.
This optimal level is best determined by monitoring mixed venous saturation.
Another step in optimizing lung function is to take advantage of the
gravitational effects on pulmonary blood flow by turning the patient prone or
to a full lateral position to direct blood flow to areas of optimal alveolar
function. This step will often result in an opening in the closed posterior
alveoli which have been compressed by the weight of the fluid in the lungs.
At the same time that oxygen delivery is optimized, oxygen consumption
should be decreased to normal or even below normal if necessary. Treating
infection, providing adequate sedation, and establishing muscular paralysis
decrease oxygen consumption, and decrease the need for oxygen delivery.

246 Phases of multiorgan failure will include:


a. Generalized increased capillary permeability
b. A hypermetabolic state
c. Organ malfunction
d. All of the above
Answer: d
Clinically the multiple-organ failure patient progresses through well-defined
phases. These phases include: Phase 1a generalized increased capillary
permeability resulting in edema, weight gain, and intravenous volume
replacement, increased protein concentration in urine and lymph. Although
the pulmonary microvasculature has been most thoroughly studied, it is
apparent that the lung is simply the most obvious end organ in a generalized
permeability defect. Phase 2A hypermetabolic state, with increased oxygen
consumption and a compensatory increase in oxygen delivery characterized
by tachycardia and high cardiac output. This condition following systemic
ischemic and reperfusion is similar to hypermetabolism following
endotoxemia, localized sterile inflammation, and infusion of stress hormones,

suggesting a common mechanism. Phase 3Organ malfunction due to


localized edema and cellular injury, particularly in the kidney, liver, brain,
and host defense system. Hemorrhagic shock predisposes to bacterial
translocation and endotoxin absorption from the intestine. Phase 4In the
absence of systemic sepsis, organs may recover to normalcy or may be
irreversibly damaged, leading to a need for chronic support. If the organ
failure phases lead to systemic infection or irreversible tissue damage in the
lung or brain, the death of the entire organ is likely.

247 Which of the following statement(s) is/are true concerning oxygen


consumption (O2)?
a. O2 is normally 100120 cc2/m2/min
b. Resting O2 is controlled by the level of thyroid and catecholamine
hormones
c. Under steady state conditions the amount of oxygen consumed exceeds the
amount of oxygen taken up by the pulmonary capillaries
d. O2 is dependent on the status of pulmonary function
Answer: a, b
Oxygen consumed in the process of metabolism is expressed as the volume
of oxygen per minute (O2). O2 is normally 100120 cc2/m2/min, or 200
cc/min for a typical adult. Resting O2 is a function of metabolizing body cell
mass, with fine tuning control provided by the level of thyroid and
catecholamine hormones. O2 decreases under conditions of hypothermia,
paralysis, and hypothyroidism. O2 increases during exercise or muscular
activity, hyperthermia, profound hypothalamic injury, hyperthyroidism,
catecholamines, and inflammatory mediators, particularly the interleukin
cytokines. Under steady state conditions, the amount of oxygen consumed in
systemic metabolism is exactly equal to the amount of oxygen taken up by
the pulmonary capillaries via the airway. This is true regardless of the status

of pulmonary function or dysfunction, so we measure O2 across the lung and


assume that it is exactly the amount consumed in the systemic metabolism.

248 Which of the following statement(s) is/are true concerning the outcome
in patients with acute renal failure?
a. Mortality for ischemic acute tubular necrosis without other organ failure is
approximately 6%
b. Multiple organ failure complicated with acute renal failure is associated
with mortality ranging from 50% to 90%
c. Recovery of renal function after six weeks is unlikely
d. There is no difference in survival between oliguric and nonoliguric renal
failure
Answer: a, b, c
Survival of patients with acute renal failure is a function of the successful
treatment of the primary disease from which the renal failure was derived.
The mortality for ischemic acute tubular necrosis without organ failure has
been reported at approximately 6%. By contrast, mortality of multiorgan
failure complicated by acute renal failure ranges from 50% to 90%. In
patients who survive the acute phase of illness, recovery of renal function
after acute renal failure is dependent on the type and extent of injuries to the
renal parenchyma. If renal function is not returned after six weeks, recovery
is unlikely. Nonoliguric renal failure is usually limited in its extent and is
almost always reversible.

249 Which of the following statement(s) is/are true concerning oxygen


delivery?
a. The amount of oxygen delivered to peripheral tissues is dependent upon

the oxygen content in arterial blood and cardiac output


b. Oxygen content is commonly measured in arterial blood
c. The normal arterial-venous difference is 4 cc O2/dL
d. Normal systemic oxygen delivery for a typical adult is approximately 1000
cc/min
Answer: a, c, d
The amount of oxygen that is delivered to peripheral tissues is the product of
the oxygen content in arterial blood times the cardiac output. Normally,
oxygen content of arterial blood is approximately
20 cc/dL, and the normal cardiac index is 5 L/min. Therefore, the normal
systemic oxygen delivery is approximately 1000 cc/min. Although oxygen
content is the most important measure of oxygen in the blood, PO2 and
oxyhemoglobin saturation is more commonly measured in the Intensive Care
Unit, hence it is necessary to convert between these measurements. The
normal oxygen content of venous blood is 16 cc/dL. Hence, the normal
arterial-venous difference is 4 cc O2/dL.

250 Which of the following statement(s) is/are true concerning carbon


dioxide kinetics?
a. The amount of carbon dioxide produced is equivalent to the amount of
oxygen consumed
b. Carbon dioxide levels in blood, present mostly as a bicarbonate ion, can
quickly change
c. Normally the amount of carbon dioxide excreted through the lung is
exactly equal to the amount of carbon dioxide produced in peripheral tissues
d. The amount of carbon dioxide excreted is a function of ventilation of
perfused alveoli
Answer: a, c, d

The total amount of carbon dioxide produced by systemic metabolism is


roughly equivalent to the amount of oxygen consumed (100120 cc/m2/min,
200 cc/min in a typical adult). CO2 production is increased or decreased by
each of the factors that causes an increase or decrease in oxygen
consumption. Most of the carbon dioxide in blood is present as bicarbonate
ion which cannot be changed quickly. However, the metabolically produced
CO2 is mostly present as dissolved carbon dioxide, added to the blood in the
peripheral tissues and excreted in the lung. In a steady state, the amount of
carbon dioxide excreted through the lung is exactly equal to the amount of
carbon dioxide produced in peripheral tissues. The amount of carbon dioxide
excreted is a function of ventilation of perfused alveoli (i.e. the alveolar
ventilation/min).

251 Which of the following result in a decrease in functional residual


capacity?
a. Shallow breathing
b. Partial airway occlusion
c. Absorption atelectasis
d. Hemothorax
Answer: a, b, c, d
A decrease in functional residual capacity is caused by incomplete alveolar
inflation related to 1) shallow breathing; 2) partial or complete airway
occlusion, which may be generalized (as in bronchospasm) or localized (as in
gastric aspiration); 3) absorption atelectasis, which occurs when oxygen is
substituted for nitrogen in the inspired gas; or 4) conditions in which air or
fluid is occupying a potential alveolar space in the chest such as
pneumothorax, hemothorax, or pulmonary edema.
252 Which of the following statement(s) meet the criteria for organ failure?

a. Bilirubin greater than 5 mg/dl


b. Creatinine greater than 3 mg/dl
c. Alveoloarterial O2 gradient greater than 300 mm Hg
d. Glasgow Coma score less than 10
Answer: a, b, c, d
Multiple organ failure is defined by dysfunction of two or more of the six
vital organ systems: cardiovascular, respiratory, nervous system, renal, liver,
and host defenses.

253 Which of the following statement(s) is/are true concerning pulmonary


edema?
a. Pulmonary edema effectively narrows bronchi and increases pulmonary
vascular resistance
b. Ventilation and perfusion are decreased equally
c. Positive pressure ventilation improves gas exchange by decreasing lung
edema
d. The condition is frequently caused by decreased plasma protein levels
Answer: a
The causes of pulmonary edema are: 1) increased hydrostatic pressure; 2)
increased capillary permeability and 3) decreased plasma oncotic pressure.
The latter, however, is rarely a problem unless the concentration of plasma
protein is very low. When fluid begins to collect in the lung interstitium, it
migrates to the loose areolar portion of the lung microanatomy that surround
the small bronchioles and pulmonary arteries. The edema in these areas has
the effect of narrowing bronchi and increasing resistance in the pulmonary
vasculature. This will decrease both ventilation and perfusion in the
edematous area, but ventilation is often affected more than blood flow,

resulting in a decreased / ratio, with all of its attendant effects on gas


exchange. Ventilator treatment of pulmonary edema which increases airway
pressure tends to hold the alveoli open, spreading out the space available for
water accumulation and overcomes the effect of small bronchial occlusion.
Positive pressure ventilation does not, therefore, affect the amount of edema
in the lung, only its manifestations.

254 Which of the following statement(s) is/are true concerning various causes
of acute renal failure?
a. Acute tubular necrosis is the most common pathologic finding of acute
renal failure
b. Drug-induced renal failure is compounded in situations of hypovolemia
c. Myoglobin-induced renal failure can be prevented using diuretics and
alkalization of urine
d. The incidence of radiographic contrast dye-induced renal failure occurs
independent of preexisting conditions
Myoglobin is a direct nephrotoxin
Answer: a, b, c
Acute tubular necrosis results from ischemia to the renal parenchyma and is
the most common pathologic finding of acute renal failure. In conditions of
diminishing renal blood flow, perfusion to the kidneys is first maintained by
vasomotor responses which dilate the afferent arteriole and constrict the
efferent arteriole. As continued hypotension occurs, the renin-angiotensin
system is activated and vasoconstriction of the afferent arteriole occurs which
exacerbates corticohypoperfusion. Pigment nephropathy is a common cause
of acute renal failure occurring after trauma, burns, operations, or
hemodynamic catastrophe. With ischemia or blunt injury to large muscles,
myoglobin is released into the circulation. In the kidney, it is filtered from
blood and reabsorbed by the tubule. Although myoglobin is not a direct

nephrotoxin, in the presence of aciduria, myoglobin is converted to


ferrihemate, which is toxic to renal cells. Prevention of myoglobin-induced
renal failure may include the use of diuretics and alkalinization of urine.
Drug-induced acute renal failure is responsible for approximately 5% of all
cases of acute renal failure. Through normal reabsorption and secretion, the
kidney is exposed to high concentrations of drugs and solutes, which may be
toxic. This problem is compounded by hypovolemia, which causes increased
reabsorption of water and solutes and exposes the lumen to even higher
concentrations of toxins. The incidence of radiographic contrast dye induced
nephropathy is approximately 1 to 10% and may be predicted according to a
number of risk factors which include contrast load, age, preexisting renal
insufficiency, and diabetes. The incidence in patients with normal renal
function is significantly lower at 1% to 2%.

255 The patient requires renal replacement therapy. Which of the following
statement(s) is/are true concerning the differences between hemodialysis and
continuous arteriovenous hemodialysis (CAVHD)?
a. Anticoagulation is not required for CAVHD
b. Hemodynamic instability will be a particular problem with both techniques
c. Both techniques will decrease serum urea ni+62trogen levels
d. CAVHD will likely result in better removal of excessive volume
Answer: c, d
256 Which of the following statement(s) is/are true concerning continuous
arteriovenous hemofiltration (CAVH)?
a. The technique runs continuously
b. It is not associated with the hemodynamic instability
c. Systemic heparin anticoagulation is necessary
d. Fluid balance and correcting electrolyte abnormalities takes several days

Answer: a, b
Continuous arteriovenous hemofiltration (CAVH) is an extracorporial
filtration technique that removes extracellular fluid across a synthetic
membrane via hydrostatic pressure gradient created between the indwelling
arterial and venous catheters. Arteriovenous access is accomplished by
percutaneous cannulation of femoral artery and vein with a low incidence of
complications. Although full systemic anticoagulation is not necessary for
CAVH, heparinization of the extracorporial circuit is required. CAVH is run
continuously for as many days as renal replacement is required. Experience
with CAVH has demonstrated very little or no hemodynamic instability with
treatment of critically ill renal failure patients. The stable nature of this
therapy is attributed to a slow and continuous fluid and solute removal and to
the fact that the membrane does not induce compliment activation when in
contact with blood. Fluid balance and serum electrolyte concentrations can be
titrated to any level in a matter of hours by manipulating the composition and
rate of replacement solution. Solute clearance with CAVH is limited by the
ultrafiltration and replacement fluid exchange rate. In patients with high urea
generation rates, solute removal with CAVH may be inadequate and
variations of the technique may be used to enhance clearance.

257 A 64-year-old diabetic patient develops acute renal failure following an


aortic aneurysm repair. Which of the following statement(s) is/are true
concerning his diagnosis and management?
a. Resting energy expenditure will likely be less than would be expected for a
patient with normal renal function
b. Maintenance of positive energy balance reduces protein catabolism and
makes the management of renal failure easier
c. Expected metabolic abnormalities include hyperkalemia, hypercalcemia,
and metabolic alkalosis

d. A nonoliguric renal failure is usually associated with a better outcome


Answer: b, d
In patients with nonoliguric renal failure, treatment may differ little from that
required for identical patients with normal renal function. Management of
fluids, solutes, and nutrition is usually unaffected by nonoliguric renal failure,
although BUN may be elevated. The extent of renal dysfunction is limited
and almost always reversible. The use of renal replacement therapies is rarely
necessary. Acute renal failure can result in severe derangements in electrolyte
and acid-based physiology. Of all electrolyte abnormalities that might occur,
hyperkalemia is the most serious. Other electrolyte abnormalities such as
hyponatremia, hyperphosphatemia, hypocalcemia, and metabolic acidosis are
common and must be monitored carefully.
The metabolic requirements of a patient with acute renal failure are those of a
critically ill hospitalized patient. The actual measurements of resting energy
expenditure has shown that caloric requirements for multiorgan failure
patients with renal failure are often 50% above normal, healthy individuals.
Although acute renal failure may require fluid restriction, providing adequate
nutrition is an important aspect of their treatment. Positive energy balance
may make management of uremia and hyperkalemia less difficult. By
providing adequate calories, endogenous protein catabolism with resultant
generation of urea and release of potassium can be avoided. Maintenance of
positive energy balance with glucose and lipids should reduce protein
catabolism, urea generation, and hyperkalemia.

258 Which of the following statement(s) is/are true concerning various


energy sources?
a. Carbohydrate is the most efficient source of energy
b. Endogenous fat is the major source of energy during starvation
c. The respiratory quotient of carbohydrate is greater than either fat or protein

d. Ketones can be used as a source of energy during starvation


Answer: b, c, d
The major sources of energy are carbohydrates and fats. Carbohydrates are a
major source of energy during normal, non-starving existence. The brain, the red
cells, and some other organs are o

the capacity to use


ketones as an energy source, a process known as starvation adaptation. Fat is
the most efficient source of energy. Fat produces 9 calories of energy per
gram of substrate metabolized while carbohydrate produces only 4 calories.
The respiratory quotient represents the number of molecules of carbon
dioxide for each molecule of oxygen consumed. For carbohydrates it is 1.0,
whereas for fat, this respiratory quotient is 0.7. Endogenous fat is the major
source of energy during starvation. The glycogen storage is basically depleted
after a day of fasting and fat becomes a major energy source with protein
breakdown supplying glucose through the process of gluconeogenesis.
gate glucose users. The brain and red cells can develop

259 Which of the following statement(s) is/are true concerning the treatment of
multisystem organ failure?
a. Forced diuresis with negative fluid balance may improve survival and acute
respiratory failure

b. The titration of ionotropic drugs based on desired blood pressure optimizes


the results.
c. Nutritional support should be withheld for several days until the patients
condition stabilizes
d. Continuous arteriovenous hemofiltration is preferred to intermittent
hemodialysis for most critically ill patients
e. Hepatic failure should be treated specifically with pharmacologic
manipulation
Answer: a, d
The important principles in the management of multiple organ failure are to

avoid further episodes of local or systemic ischemia and to keep the brain
viable by pharmacologic or mechanical support of the failing organs until
organ recovery occurs. Respiratory failure is treated by mechanical assistance
for lung inflation and ventilation and by decreasing lung edema as much as
possible. Airway intubation is usually required. There is now good evidence
that forced diuresis and negative fluid balance is associated with improved
survival and acute respiratory failure. Cardiac failure is treated with inotropic
drugs. Although ionotropic drugs are usually titrated to achieve a desired
arterial blood pressure, it is more sensible to titrate ionotropes to achieve a
normal oxygen delivery/oxygen consumption ratio. Pulmonary artery
pressure and mixed venous saturation monitoring are essential for intelligent
management of the patient with severe respiratory or cardiac failure.
Adequate nutrition is also important for recovery from organ failure. Renal
failure is treated by mechanical substitution of renal function. Although
hemodialysis and peritoneal dialysis can serve this purpose, each has a
significant drawback in the critically ill, multiple organ failure patient.
Continuous arteriovenous hemofiltration (CAVH) and continuous
arteriovenous hemodialysis (CAVHD) are the methods of choice for renal
replacement therapy. Hepatic failure often occurs as part of the multiple
organ failure syndrome but unfortunately there is no specific treatment.

260 Which of the following statement(s) is/are true concerning pulmonary


mechanics?
a. The standard compliance or volume pressure curve is measured during
lung inflation
b. The decreased compliance in acute respiratory failure occurs because the
lung is smaller not stiffer
c. In acute respiratory failure, higher pressures are required to achieve the
same level of inflation
d. Areas of normal lungs are more vulnerable to overdistention which may

lead to progressive lung dysfunction


Answer: b, c, d
The standard compliance or volume pressure curve is drawn by measuring
volume and pressure at stages of lung deflation after total inflation. The
decreased compliance in acute respiratory distress syndrome occurs because
the lung is smaller, not stiffer. In acute respiratory failure, the cause of
decreased compliance is almost always associated with a decrease in
functional residual capacity (FRC). The decreased FRC represents lost alveoli
which are either collapsed or filled with fluid but still perfused with blood.
Because the lung is smaller, the compliance curve has shifted to the right and
much higher pressures are required to achieve the same level of inflation.
Lung damage can be caused by high airway pressure, so that overdistension
is not merely inefficient but actually detrimental. Since the most normal areas
of lung have the best compliance, they are most vulnerable to overdistension,
contributing to the steady progression of lung dysfunction in patients
ventilated at high peak pressure.

261 Which of the following statement(s) is/are true concerning the use of a
ventilator in the treatment of respiratory failure?
a. The assist-control mode is appropriate in the paralyzed patient
b. Peak inspiratory pressure should be optimized at a level in excess of 40 cm
H2O
c. A patient receiving excessive carbohydrate as a nutritional support may
have an elevated minute ventilation and may tire with spontaneous breathing
d. In general, weaning requires an adequate inspiratory force, vital capacity,
and a minute ventilation less than 10 L/min
Answer: c, d
Most intensivists favor setting the ventilator on the assist-control mode at a

low sensitivity. In this fashion, the patient breathes at a rate that regulates the
PaCO2 to normal, but each breath is mechanically assisted, providing
maximal inflation. The volume of each breath is set by limiting the maximal
pressure or maximal volume of each breath. Whichever method is used, the
peak inspiratory pressure should not generally exceed 40 cm H2O. If the
patient is comatose or paralyzed, the assist mode cannot be used and the rate
is set in addition to the volume.
Adequate weaning indices are: inspiratory force greater than 20 cm H2O,
vital capacity twice the tidal volume, adequate gas exchange at assisted
ventilation at FiO2 of 0.3 and 5 cm H2O of PEEP, and minute ventilation less
than 10 L/min. If the patient is hypermetabolic or is receiving excessive
carbohydrate as nutritional support, the minute ventilation will be elevated,
even during assisted mechanical ventilation. If this is the case, the patient will
tire rapidly on spontaneous breathing.

262 Which of the following statement(s) is/are true concerning the estimation
and measurement of energy requirements in the critically ill patient?
a. One can only estimate energy expenditure with actual measurement not
technically possible
b. The amount of oxygen absorbed through the lungs is equal to the amount
of oxygen consumed by metabolic processes
c. Metabolic rate, normalized to body surface area, may underestimate
metabolism in a fat person
d. To convert ccs of oxygen per minute to calories per day, a conversion
factor of 10 kcal of energy per liter of oxygen should be used
Answer: b, c
The actual metabolic rate of any patient can be estimated from the predicted
basal rate according to the clinical situation. The amount of energy is most
conveniently expressed in calories/day. The metabolic rate is normalized to

body surface area; however, the actively metabolizing tissue is the lean body
cell mass. Consequently, reporting per square meter underestimates
metabolism in a fat person and overestimates in a very lean person. Although
most of studies on nutrition in critical illness have been based on estimated
energy expenditure, actual measurement is much more accurate and has
become an important aspect of critical care management. The most
commonly used method of measurement is indirect calorimetry. In this
method, the amount of oxygen absorbed across the lungs into the pulmonary
blood is measured over a given period of time. Assuming the patient is at a
metabolic steady state during this time, the amount of oxygen absorbed
across the lungs is equal to the amount of oxygen consumed in the metabolic
process. The metabolic rate, measured in cubic centimeters of oxygen/minute,
can be converted to calories/hour or /day if the oxygenated substrates are
known. For practical purposes, a conversion factor of 5 kcal of energy/liter of
oxygen consumed is a reasonable approximation.

263 Which of the following statement(s) is/are true concerning the response
to a decrease in functional residual capacity percent (FRC)?
a. Supplying supplemental oxygen will always improve the situation
b. Respiratory alkalosis may occur
c. Decreasing compliance is a common occurrence
d. Respiratory rate and depth of breathing generally decrease
Answer: b, c
Pulmonary arterial spasm in response to local hypoxia autoregulates
pulmonary blood flow and maintains adequate gas exchange during alveolar
collapseup to a point. However, when the loss in ventilation exceeds the
decrease in perfusion, a ventilation-perfusion mismatch occurs, which results
in incomplete oxygenation of blood perfusing that area of the lung. The
resultant hypoxemia stimulates an increased rate and depth of breathing

which may serve to reexpand the persons inflated area of lung. If it does not,
hypoxemia will continue but increased ventilation in other areas of the lung
will result in excess CO2 excretion, hypocapnea and respiratory alkalosis.
The blood gas picture, hypoxemia with respiratory alklalosis, is the most
common abnormality of gas exchange in surgical patients and it is a hallmark
of ventilation-perfusion imbalance. Oxygenation of blood in the poorly
ventilated area of lung can be improved by increasing concentration of
oxygen in the inspired gas. The use of supplemental oxygen, however, treats
the symptom rather than the basic cause and may actually make the problem
worse by adding to absorption atelectasis, depriving the poorly ventilated
area of nitrogen to hold alveoli open. This may result in total alveolar
collapse. In this circumstance, blood perfusing the nonventilated area will
mix with blood from other areas of the lung, resulting in hypoxemia that does
not improve significantly in response of administration of oxygen. Aside
from the effects on gas exchange, loss of alveolar space results in changes in
the volume-pressure relationships in the lung. A decrease in functional
residual capacity always results in a shift in the volume-pressure relationship
toward a condition of decreasing compliance.
264 Which of the following statement(s) is/are correct concerning the body
fluid compartments?
a. Both the extracellular and intracellular components of total body water can
be directly measured
b. The intravascular space accounts for the majority of extracellular fluid
c. All water in the interstitial space is freely exchangeable
d. Transcellular fluid, separated from other compartments by both endothelial
and epithelial barriers, constitute about 4% of total body water
Answer: d
Total body water (TBW) is distributed within the intracellular and
extracellular compartments. Intracellular fluid cannot be measured directly

but is calculated as the difference between TBW and the measured


extracellular water. Extracellular fluid can be measured directly. The
extracellular fluid compartment can be further simplified into the
intravascular and interstitial spaces. Intravascular space, which accounts for
20% of the extracellular fluid, contains the plasma volume which is
approximately 8% of TBW or 5% of body weight. The interstitial space
extends from the blood vessels to the cells themselves and includes the
complex ground substance making up the acellular matrix of tissue. Although
the water within the space is thought to be freely exchangeable, this water
exists in two phases. The free phase contains water that is generally freely
exchangeable and in a constant state of flux. The bound or gel phase is
composed of water that is closely associated with glycosaminoglycans,
mucopolysaccharides, and other matrix components. This water is much less
freely exchangeable. An additional extracellular fluid compartment, the
transcellular compartment, consists of water that is poorly exchangeable
under normal circumstances. This fluid is separated from other compartments
by both endothelial and epithelial barriers and includes cerebrospinal fluid,
synovial fluid, water within cartilage and bone, fluids of the eye, and the
lubricating fluids of the serous membranes. Together, these fluids constitute
about 4% of TBW.

265 Which of the following statements(s) is/are true concerning metabolic


alkalosis?
a. Either increased extracellular bicarbonate concentration or inhibited renal
excretion of bicarbonate can cause metabolic alkalosis
b. In metabolic alkalosis secondary to prolonged gastric outlet obstruction,
the urine pH is usually acidic
c. Hypokalemia can lead to metabolic alkalosis
d. The respiration compensatory mechanisms for metabolic alkalosis are quite
ineffective

Answer: b, c, d
Sustained metabolic alkalosis occurs only if extracellular bicarbonate
concentration is increased and renal excretion of excess bicarbonate is
inhibited. Alone, neither is sufficient to result in metabolic alkalosis.
Extracellular bicarbonate concentration is increased by numerous
mechanisms. Loss of HCl is the leading cause of metabolic alkalosis in
surgical patients. External loss of gastric acid results in net gain in
bicarbonate, which causes metabolic alkalosis. Although the kidney can
excrete excess bicarbonate, this must be accompanied by excretion of
sodium. Renal excretion of sodium is limited in the face of volume depletion,
which also occurs with external losses of gastric secretion. As volume
depletion progresses, sodium is conserved in exchange for hydrogen. Thus, in
metabolic alkalosis secondary to prolonged gastric outlet obstruction, the
urine, although initially alkalotic, becomes paradoxically acidotic in
prolonged or uncorrected cases. Hypokalemia and cellular exchange of
potassium for hydrogen can also lead to metabolic alkalosis. Hypokalemia
results in enhanced proximal tubular bicarbonate reabsorption and distal
tubular acid secretion. The major compensatory mechanism in metabolic
alkalosis is respiratory, since the presence of metabolic alkalosis implies renal
dysfunction in either generating or failing to excrete increased amounts of
bicarbonate. Hypoventilation is limited by the development of hypoxemia,
which stimulates ventilation. Among the four major types of acid-base
disorders, this compensatory mechanism is the least effective.
266 Which of the following statement(s) is/are true concerning respiratory
acidosis?
a. Respiratory acidosis is associated with chronic pulmonary disease far more
commonly than is hypoxemia
b. The initial buffering effect occurs at the cellular level
c. Renal compensation occurs within 24 hours

d. Correction of hypoxemia in patients with chronic lung disease may worsen


respiratory acidosis
Answer: b, d
Respiratory acidosis, the decrease in extracellular pH from a primary increase
in PCO2, is due to inadequate ventilation. Although pulmonary disease
commonly causes hypoxemia, respiratory acidosis is far less common, since
defusion of O2 is more readily impaired than diffusion of CO2. Increased
PCO2 results in increased H2CO3 which disassociates into H+ and HCO3.
Cellular exchange of Na+ and K+ for H+ allows the reaction to continue in
this direction with increased extracellular bicarbonate. This tissue buffering is
accomplished within minutes. Persistently elevated PCO2 also stimulates
increased renal acid excretion. Full renal compensation occurs over 3 to 5
days. The treatment of chronic compensated respiratory acidosis may be
complicated by accompanying hypoxemia. In chronic hypercapnia, the
chemical chemoreceptors may be insensitive and the accompanying
hypoxemia may supply the main respiratory drive through the stimulation of
peripheral chemoreceptors. In such patients, complete correction of
hypoxemia may further depress respiration and worsen the respiratory
acidosis.

267 Which of the following(s) is/are true concerning the control of the
volume of body water?
a. Osmoreceptors and baroreceptors work equally to control fluid balance
during normal conditions
b. The cardiac atrium regulates volume only by means of its sympathetic and
parasympathetic connections
c. The kidney is the primary effector organ in controlling water balance
d. The conversion of angiotensin I to angiotensin II is dependent on the
amount of the enzyme, renin, available

e. Nitric oxide plays a number of important roles in regulation of renal


hemodynamics
Answer: c, d, e
Changes in volume are detected both by osmoreceptors, which detect changes
in plasma osmolality and baroreceptors, which are sensitive to changes in
pressure. The osmoreceptors are responsible for day-to-day fine-tuning of
volume whereas the baroreceptors contribute relatively little to the control of
fluid balance under normal conditions. Changes in effective circulating
volume are sensed by the volume receptors of the intrathoracic capacitance
vessels and atria, the pressure receptors of the aortic arch and carotid arteries,
the intrarenal baroreceptors, and, to a lesser extent, by the hepatic and
cerebrospinal volume receptors. These baroreceptors control volume by
means of sympathetic and parasympathetic connections. The atria also appear
to serve as endocrine organs capable of directing responses to volume
changes with the elaboration of the hormone, atrial natriuretic peptide. The
major hormonal mediator of baroreceptor modulation of volume control is
the renin-angiotensin system. The end result of this complex system of
receptors or messengers is a change in sodium and water balance mediated by
the kidneys. It is through changes in sodium and water reabsorption that
volume and pressure ultimately normalize. Renin is a proteolytic enzyme that
is released in response to changes in arterial pressure, changes in delivery of
sodium to the macula densa of the distal convoluted renal tubule, increases in
beta adrenergic activity and increases in cellular cAMP. Renin cleaves
angiotensin I from circulating angiotensinogen. Angiotensinogen is abundant,
so this reaction is enzyme dependent rather than substrate dependent.
Angiotensin I is further cleaved to angiotensin II which acts with locally and
systemically to increase vascular tone. Angiotensin II affects sodium
reabsorption by decreasing renal plasma flow and the glomerular filtration
coefficient. Finally, angiotensin II increases sodium reabsorption by direct
tubular action as well as by stimulation of aldosterone release from the
adrenal cortex.

The importance of nitric oxide and its many biologic functions has recently
been recognized. Nitric oxide participates in the regulation of renal
hemodynamics and renal handling of water and electrolytes.

268 Which of the following statement(s) is/are true concerning maintenance


intravenous fluid therapy?
a. The total daily water requirement for a 70-kg man is about 2500 mL/day
b. Normal maintenance IV therapy requires administration of sodium,
potassium, calcium, phosphate, and magnesium
c. Fluid volume calculations for elderly patients generally are decreased
compared to their younger counterparts
d. A child requires a lesser amount of maintenance fluid per kilogram than a
larger individual
Answer: a, c
Maintenance fluid replacement is aimed at replacing fluids normally lost
during the course of a day. Calculation of maintenance fluid replacement
does not include replacement of preexisting deficits or ongoing additional
losses. Formulas exist for calculating maintenance fluid requirements which
adjust for differences in body weight and for changes in TBW content. A
smaller (or younger) individual who has a high percentage of TBW in
relation to body weight requires a greater amount of maintenance fluid per
kilogram than a larger individual. The total daily water requirement for a 70kg man is about 2500 mL/day. Because hypervolemia is poorly tolerated in
older individuals and in patients with cardiac disease, the volume calculated
is generally diminished in this age group. Normal maintenance therapy
requires administration of sodium and potassium. Replacement of calcium,
phosphate or magnesium are generally not necessary in patients requiring
short-term therapy. In critically ill patients, however, critical deficits in these
electrolytes may occur and must be replaced.

269 Which of the following statement(s) is/are true concerning abnormalities


in calcium concentration?
a. Parathyroid hormone affects calcium homeostasis only at the exchange of
calcium between bone and extracellular fluid
b. About 45% of total plasma calcium is in the ionized state and is
responsible for most physiologic actions
c. Changes in plasma protein levels or pH can alter the proportion of calcium
in the ionized state
d. Intravenous normal saline administration is the first step in treatment of
hypercalcemia
e. Classic signs of hypocalcemia include hyperactive deep tendon refluxes,
Cvosteks sign and Trousseaus sign
Answer: b, c, d, e
Calcium is a divalent cation found in abundance in the human body. About
99% of total body calcium is located in bone in the form of hydroxyapatite
crystals. Calcium homeostasis depends on the exchange of calcium between
bone and extracellular fluid, renal excretion, and intestinal absorption. These
three processes are controlled to a great extent by parathyroid hormone. In
extracellular fluid, calcium exists in three forms: ionized calcium, nonionized calcium, and protein-bound calcium. Ionized calcium, which
comprises about 45% of total calcium is responsible for most physiologic
actions of calcium in the body, and its level is tightly controlled by a
regulatory mechanisms. Some nonionized calcium is complexed with nonprotein anions, including phosphate and citrate, and does not easily
disassociate. These molecular forms make up only about 15% of total
calcium present in plasma. About 40% of extracellular nonionized calcium is
bound to proteins, with most being bound to albumin. Changes in either
plasma protein levels or pH can alter the proportion of calcium in the ionized

state. The most common cause of hypercalcemia is primary


hyperparathyroidism. Hypercalcemia can also occur secondary to malignant
disease, caused either by a metastasis to bone or by autonomous tumor
secretion of hormone-like substances that alter calcium homeostasis.
Neuromuscular effects may be the earliest manifestations and include muscle
fatigue, weakness, personality disorders, psychosis, confusion, and coma.
Elevation of total serum calcium concentrations to greater than 14mg/dL
requires prompt treatment to prevent any serious and potentially lethal
complications. Immediate measures are directed toward maximizing renal
excretion of calcium. Vigorous hydration with 0.9% saline solution to prompt
diuresis should be the initial step in treatment. The addition of potassium to
the resuscitation fluid as well as the use of furosemide can also be used for
treatment.
Serum calcium levels below 8 mg/dL may be associated with symptoms and
signs that are primary manifestations of neuromuscular abnormalities. These
include muscle cramps, perioral tingling, parastesias, laryngeal stridor, tetany,
seizures and psychotic behavior. Classic signs of hypocalcemia include
hyperactive, deep tendon reflexes, Cvosteks sign, and Trousseaus sign.
Symptomatic hypocalcemia is best treated with intravenous infusion of
calcium in the form of calcium gluconate or calcium chloride.

270 Which of the following statement(s) is/are true concerning total body
water?
a. Total body water in men represents a higher percent body weight than in
women
b. In infants, water comprises up to 80% of body weight
c. Total body water content decreases with increasing age
d. Total body water is equally distributed within the intra-and extracellular
compartments
Answer: a, b, c

The total volume of water within the body is termed total body water. The
relationship between total body water (TBW) and body weight is relatively
consistent for any given individual and depends on the amount of fat within
the body. Because fat contains little water, TBW as a percentage of body
weight decreases with increasing body fat. The estimated TBW in men is
60% of body weight, whereas in women, who typically have more adipose
tissue, the average TBW is 50% of body weight. The percentage of body
weight accounted for by water also varies with age. In infants, water
comprises about 80% of body weight. Throughout adult life, a gradual
decrease occurs in TBW content because of the amount of fat within the body
usually increases with age. In obese patients, estimates of TBW should be
decreased by 10% to 20% whereas in lean patients, estimates should be
increased by about 10%.
TBW is distributed within the intra and extracellular compartments.
Intracellular fluid makes up about 2/3 of the TBW, or 40% of body weight.

271 Which of the following statement(s) is/are true concerning the clinical
presentation and treatment of severe metabolic alkalosis?
a. In most cases clinical signs are obvious
b. Correction of potassium and volume depletion corrects most cases of
metabolic alkalosis
c. Acetazolamide can enhance renal excretion of bicarbonate
d. Acid replacement should be provided at a molar equivalent basis for excess
serum bicarbonate
Answer: b, c
Clinical signs of metabolic alkalosis may not be prominent, since the
condition usually develops relatively slowly. Correction of the underlying
cause is the mainstay of treatment in this disorder. In general, correction of

potassium and volume depletion corrects the metabolic alkalosis. In patients


without intravascular volume deficits, renal excretion of bicarbonate can be
enhanced by administration of the carbonic acid anhydrase inhibitor
acetazolamide. If renal excretion of bicarbonate cannot be increased because
of underlying renal insufficiency or if the metabolic alkalosis is severe, acid
may be administered to directly titrate the excess extracellular bicarbonate.
Acids that can be used include ammonium chloride, arginine hydrochloride,
or dilute hydrochloric acid. Partial correction of alkalosis is the initial goal. A
general guide is that 2.2 mEq/kg decreases serum bicarbonate by about 5
mEq/L.

272 Which of the following statement(s) is/are true concerning abnormalities


in serum sodium?
a. The most common cause of hyponatremia is a deficit in total body sodium
b. Hyponatremia can occur in situations of excessive solute
c. Most surgical patients with hyponatremia are best treated by free water
restriction
d. Central nervous system effects are the predominant symptom of
hypernatremia
e. Hypernatremia should be rapidly corrected with free water administration
Answer: b, c, d
The most common cause of hyponatremia is an excess of free water rather
than a deficit of total body sodium. Hyponatremia is frequently seen in the
postoperative or post-trauma patients because increased ADH secretion acts
on the collecting tubules of the kidney to increase free water reabsorption.
Although hyponatremia most often results from excess free water, it can
occur in the presence of excess solute. In this situation, TBW content is either
normal or diminished but the plasma osmolality is increased. An example of
this hyperosmolar-hyponatremic state is untreated hyperglycemia. Excess

solute may also be due to exogenous administration or ingestion of mannitol,


ethanol, methanol, or ethylene glycol. Most surgical patients with
hyponatremia are euvolemic or hypervolemic. Such patients, if
asymptomatic, are best treated by free water restriction, since free water
overload is the cause of the condition. Hypernatremia is a less common
problem in surgical patients than hyponatremia and is usually the result of
excess free water loss associated with hypovolemia. Hypernatremia may also
be secondary to increased total body content of sodium, which is usually
related to exogenous administration of sodium. The symptoms of
hypernatremia are related to the hyperosmolar state. CNS effects predominate
because of cellular dehydration as water passes into the extracellular space.
Once hypernatremia becomes symptomatic, it is associated with significant
morbidity and mortality. Prompt treatment of hypernatremia is essential.
Rapid correction, however, of hypernatremia is associated with significant
risk of cerebral edema and herniation. Because chronic hypernatremia is
relatively well tolerated, there are few advantages to rapidly correcting the
free water deficit. Moderate degrees of hypernatremia are tolerated well, and
symptoms rarely develop unless serum sodium levels exceed 160 mEq/liter.
The development of symptoms also depends on the rapidity at which
hypernatremia develops.

273 Which of the following statement(s) is/are true concerning abnormalities


in serum potassium?
a. Hyperkalemia can occur in the otherwise normal surgical patient due to
excessive intravenous potassium administration
b. The primary EKG change associated with severe hyperkalemia is peaked
T-waves
c. Temporary treatment of hyperkalemia includes administration of calcium,
sodium bicarbonate, or glucose and insulin
d. Alterations in membrane potentials reflected in cardiac and skeletal muscle

are common results of both hypo-and hyperkalemia


e. A reduction in serum potassium of 1mEq/liter requires replacement of
40mEq of potassium
Answer: c, d
Potassium is the major intracellular cation and is a major determinant of
intracellular osmolality. Because of the large differences between intracellular
and extracellular potassium concentrations, a transmembrane potential is
generated. Alterations in potassium concentration gradient (both hyper- and
hypokalemia) have profound effects on transmembrane potential and
consequently on cellular function. This is especially true for cardiac, skeletal,
and smooth muscle. Extracellular potassium concentration is primarily
determined by renal excretion. About 90% of ingested potassium is secreted
by the urine. Hyperkalemia therefore rarely develops from excessive
potassium intake in the absence of renal insufficiency, since the capacity for
renal potassium excretion is large. In the surgical patient, diminished renal
function is perhaps the most common problem leading to hyperkalemia. Both
chronic and acute renal failure result in the deficit in potassium excretion.
Hyperkalemia can also be associated with cellular disruption, such as with
crush injuries or lysed erythrocytes in large hematomas or after massive
blood transfusion. The clinical manifestations of hyperkalemia are primarily
related to membrane depolarization. The most life-threatening manifestations
are related to the cardiac effects of membrane depolarization. Mild
hyperkalemia results in peaked T-waves on the EKG and may cause
parethesia and weakness. More severe forms of hyperkalemia cause flattened
P-waves, prolongation of the QRS complex, and deep S-waves on EKG.
Ventricular fibrillation and cardiac arrest may follow. Severe hyperkalemia
with EKG abnormalities requires urgent treatment. Rapid infusion of 10% to
20% calcium gluconate may reduce the effects of hyperkalemia on membrane
potentials. Administration of sodium bicarbonate is another temporary
measure. The increase in serum sodium antagonizes the effects of
hyperkalemia on the membrane potential, whereas the increase in

extracellular pH shifts potassium into the cells. Movement of potassium into


the intracellular compartment can also be achieved by giving insulin and
glucose.
Hypokalemia is usually caused by total body potassium depletion secondary
to the decreased potassium intake, increased extra-renal potassium losses, or
increased renal potassium losses. Decreased serum potassium levels may also
be secondary to redistribution of potassium into the intracellular space.
Symptoms of hypokalemia, like those of hyperkalemia, are manifested by
disturbances in membrane potentials. As potassium levels fall below
2.5mEq/L, muscle weakness is common. The primary treatment of
hypokalemia is potassium replacement. The route and rate of potassium
replacement depends on the presence and severity of symptoms. A reduction
in serum potassium of 1mEq/L represents a total body potassium deficiency
of 100 to 200 mEq.

274 Which of the following statement(s) is/are true concerning the


derangement of metabolic acidosis?
a. A major source of acid production of the body is sulfuric acid
b. Excessive loss of bicarbonate can occur with intestinal or pancreatic
fistulas
c. Ketoacidosis can occur in conditions of either hyper-or hypoglycemia
d. Lactic acidosis is present when serum lactate concentration is > 2 mEq/L
e. Lactic acidosis can be associated with ethanol toxicity
Answer: a, b, c, e
Most clinically significant metabolic acidosis is related to the net loss of
bicarbonate, which occurs when consumption due to either loss or titration is
greater than bicarbonate generation. Under normal circumstances of ingestion
of the average amount of protein in an American diet, about 70 mEq acid is
generated daily. The major source of acid production is sulfuric acid from the

metabolism of sulphur-containing amino acids. Increased protein intake and


tissue catabolism resulting in greater metabolism of sulphur containing amino
acids may lead to a generation of increased amounts of sulfuric acid. This
excess acid utilizes excess bicarbonate for neutralization. Diarrhea, intestinal
or pancreatic fistulas, and burns can cause metabolic acidosis secondary to
loss of bicarbonate. The two most common types of organic acidosis are
ketoacidosis and lactic acidosis. The abnormality primarily responsible for
ketoacidosis is deficiency of insulin whether primary, as in diabetic
ketoacidosis, or secondary to hypoglycemia. Under normal conditions a small
amount of ketoacids is produced. During prolonged starvation, production of
ketoacids increase to modest levels, providing an important source of energy
to nonhepatic tissues, particularly the brain. In ketoacidosis, the ketoacid
production is excessive because of insulin deficiency. In diabetic acidosis,
insulin deficiency also contributes to hyperglycemia by decreasing the
metabolism of glucose by extrahepatic tissue and increasing hepatic
production of glucose.
Lactic acidosis can be divided into type A, caused by tissue hypoxia, and type
B, caused by other mechanisms. Hypoxia, the most common cause of lactic
acidosis, impairs the mitochondrial oxidation of NADH to NAD that is
necessary for glycolysis. Normal serum lactate concentration is below 2
mEq/L. Lactate acidosis is secondary to hypoxemia, usually due to an
increased production of lactate as well as decreased use, and serum lactate
concentrations greater than 6 mEq/L. The most common cause of type B
lactate acidosis is ethanol intoxication.

275 Which of the following statement(s) is/are true concerning the osmotic
activity of body fluids?
a. Urea contributes to the osmolality of a solution but not its tonicity
b. The osmolality of the body remains fairly constant at approximately 289
mOsm/kg H2O

c. The two primary regulators of water balance are antidiuretic hormone and
aldosterone
d. Serum sodium is the most valuable laboratory indicator of abnormal total
body water content
Answer: a, b, d
Body fluids are aqueous solutions composed primarily of water and
contained in different compartments of the body. The movement of water
from these compartments depends on a number of physical properties, the
most important of which is osmosis. According to the principles of osmosis,
if two solutions are separated by semipermeable membrane, water moves
across the membrane to equalize the concentration of the osmotically active
particles. The osmotic activity across a semipermeable membrane is
determined by the concentration of solutes on each side of the membrane.
The body is capable of fine regulation of solute and water concentrations, so
that osmolality remains fairly constant at an average of 289 mOsm/kg H2O.
In response to small changes in cell volume, osmoreceptors in the
paraventricular and supraoptic nuclei of the hypothalamus send signals to the
neuronal centers that control the two primary regulators of water balance,
thirst and antidiuretic hormone secretion. Changes in TBW are reflected by
changes in extracellular solute concentration. Because sodium is the primary
extracellular cation and potassium is the predominant intracellular cation, the
serum sodium approximates the sum of the exchangeable total body sodium
and exchangeable total body potassium divided by the TBW. Because total
body solute content remains relatively stable over time, changes in TBW
content result in inversely proportional changes in serum sodium. Thus,
abnormalities in serum sodium are the indication of abnormal TBW content.
In contrast to impermeable solutes that are excluded from the intracellular
space, such as sodium, permeable solutes such as urea can freely cross the
cell membranes. Although urea contributes to the osmolality of a solution, it
has no effect on tonicity because it distributes equally across membranes, and
as such does not contribute to the osmols that affects cell volume.

276 Which of the following statement(s) is/are true concerning the


compensatory mechanisms and treatment of metabolic acidosis?
a. Maximal renal compensation for metabolic acidosis occurs before full
respiratory compensation can occur
b. All patients with lactic acidosis should receive prompt treatment with
bicarbonate
c. Potassium replacement is essential even in the face of normal or high
serum potassium when treating diabetic ketoacidosis
d. Sodium bicarbonate administration should begin simultaneous with
volume resuscitation in patients with hypoxia secondary to shock
Answer: c
The kidney is extremely sensitive to changes in serum bicarbonate
concentration and responds by increasing net acid excretion primarily by
increasing ammonia excretion. Maximal renal compensation requires 2 to 4
days. Delay in achieving maximal renal response to an increased acid load
causes blood pH to decline, which stimulates hyperventilation. Although
effective in promptly raising blood pH, ventilatory compensation is only
partial, and full respiration compensation requires 12 to 24 hours. The major
principal of treatment for mild to moderate acute metabolic acidosis is
correction of the underlying cause. In surgical and trauma patients, metabolic
acidosis is often the result of hypoxia secondary to inadequate tissue
perfusion and subsequent lactic acidosis. Volume and/or blood resuscitation
alone may be enough to correct the acidosis. Attempts to correct acidosis with
exogenous bicarbonate before correction of inadequate tissue perfusion are
usually unsuccessful. The use of bicarbonate for the treatment of lactic
acidosis is controversial at best. In several studies the use of bicarbonate in
patients with lactic acidosis does not improve clinical parameters or outcome.
The correction of both acidosis and hypoglycemia of diabetic ketoacidosis is

best achieved by the administration of insulin. Volume resuscitation is also


required. Potassium replacement is essential, even in the face of normal or
high serum potassium, and as hypokalemia develops as acidosis in
hyperglycemia are corrected.

277 Which of the following statement(s) is/are true concern renal tubular
acidosis?
a. Renal tubular acidosis is primarily caused by reduction in ammonia
excretion
b. The renal tubular defect in renal tubular acidosis can either be at the distal
or proximal renal tubule
c. In distal renal tubular acidosis associated with hyperkalemia, the defect
involves increased tubular permeability with backleak of secreted sodium and
potassium into the tubular cell
d. Uremic acidosis occurs independently of protein intake
Answer: a, b
The impaired ability of the kidney to excrete acid and hence generate
bicarbonate may be secondary to a decrease in the number of functioning
nephrons and is termed uremic acidosis or renal tubular acidosis. Renal
tubular acidosis, which can occur both in acute and chronic renal failure, is
primarily caused by reduction in ammonia excretion secondary to a reduction
in the number of functioning proximal tubular cells. In addition, decreased
proximal tubular bicarbonate reabsorption contributes to the development of
acidosis. Although the onset of uremic acidosis is related to declining renal
function, its appearance may be influenced by diet-dependent protein and
organic anion ingestion. Renal tubular acidosis may be classified as distal or
proximal, depending on the primary site of the renal tubular defect leading to
acidosis. In renal tubular acidosis with hyperkalemia, the mechanism is
decreased luminal negativity secondary to impaired sodium reabsorption. In

distal renal tubular acidosis with hypokalemia, mechanisms including


increased tubular permeability with backleak of secreted H+ into the tubular
cell and reduced H+ pump activity are proposed mechanisms.

278 Which of the following statement(s) is/are true concerning the


postoperative fluid management in a surgical patient?
a. Standard formulas are available that essentially can direct the therapy for
all patients
b. Isotonic solutions containing potassium should be used throughout the
entire postoperative period
c. Urine output should be maintained at a level greater than 0.5 ml/kg/h
d. A urine specific gravity of greater than 1.012 may indicate that the patient
is dehydrated
Answer: c, d
Fluid therapy during the postoperative period should be tailored to each
patient and depends on the adequacy of patients volume status at the
completion of the operative procedure, as well as ongoing fluid losses.
Maintenance fluid should be supplemented by replacement of the additional
fluids needed to replace the ongoing third space loss as well as losses from
various tubes and drains. In general, isotonic solution should be used for
volume resuscitation during the early postoperative period. It is best not to
give potassium supplements during this period unless they are specifically
required as indicated by serum electrolyte measurements. Monitoring fluid
status during the postoperative period is best accomplished by careful
monitoring of vital signs, urine output, and central venous pressure, if
necessary. Urine output is maintained at a level greater than 0.5 mL/kg/h. A
urine specific gravity of greater than 1.010 to 1.012 indicates that urine is
being concentrated and the patient may not be receiving adequate hydration.

279 Which of the following statement(s) is/are true concerning parenternal


electrolyte solutions?
a. Lactated Ringers solution contains physiologic concentrations of all
important electrolytes
b. Glucose is added to hypotonic saline solutions to increase their tonicity
c. About 1/2 of all exogenously administered albumin ends up in the
extravascular space
d. Normal saline solution provides excessive sodium and chloride which may
lead to body sodium overload
Answer: b, c, d
A number of electrolytes solutions are available for parenteral administration.
Lactated Ringers solution is a physiologic solution containing many of the
electrolytes found in plasma. The disadvantage of this solution is the
relatively low sodium content (130 mEq/L) as compared to plasma.
Hyponatremia can occur with extended use of lactated Ringers solution.
Isotonic saline (0.9% or normal saline) contains 154 mEq of both sodium and
chloride. The excess of both sodium and chloride can lead to electrolyte and
acid-base disturbances. Infusion of large volumes of 0.9% saline can lead to
total body sodium overload and hyperchloremia. The less-concentrated saline
solutions are hypo-osmotic and have excess free water. In addition, 0.2%
saline solution is hypotonic with respect to plasma and can result in red blood
cell lysis if rapidly infused. For this reason, 5% dextrose is added to these
solutions to increase the tonicity. Plasma expanders are commonly used in
surgical patients. Plasma protein solutions such as 5% and 25% albumin act
initially by increasing plasma oncotic pressures. Abnormalities in
microvascular permeability such as those found in the pulmonary circulation
in adult respiratory distress syndrome, in regional circulatory bed burns or
infections, and in the systemic circulation in sepsis, may result in
extravasation of these proteins into the interstitial space. About half of all

exogenously administered albumin eventually ends up in the extravascular


space. The half life of exogenously administered albumin is about 11 days.

280 An 11-year-old boy has experienced severe diarrhea for 10 days. He


presents with decreased skin tungor, sunken eyes, orthostatic hypotension,
and tachycardia. Which of the following statement(s) may be true concerning
his diagnosis and treatment?
a. His hematocrit will likely be elevated
b. His BUN may be elevated out of proportion to serum creatinine
c. His serum sodium will be elevated
d. Fluid resuscitation should begin with D5/.2 normal saline because of the
expected high serum sodium associated with excessive fluid loss
Answer: a, b
Chronic volume deficits may be manifested by decreased skin turgor, weight
loss, sunken eyes, hypothermia, oliguria, orthostatic hypotension and
tachycardia. Serum BUN and creatinine may be elevated, with a high
BUN/creatinine ratio. The hematocrit may be elevated as well. Plasma
sodium is not an indicator of intravascular volume, and if the loses have been
isotonic, plasma sodium concentration remains normal. Fluid resuscitation
for hypovolemia is initiated with an isotonic solution such as lactated
Ringers solution. Urine flow in critically ill patients is monitored with an
indwelling Foley catheter, with the goal of a urine output 0.5mL/kg/h
desirable.

281 Clinical manifestations of acute metabolic acidosis include:


a. Decreased cardiac contractility
b. Decreased catecholamine secretion

c. Peripheral arteriolar dilitation


d. Shift of the oxygen-hemoglobin disassociation curve to the left
Answer: a, b, c
The major cardiovascular effects of acute metabolic acidosis are peripheral
arteriolar dilitation, decreased cardiac contractility, and central venous
constriction. These may lead to cardiovascular collapse and pulmonary
edema. Catecholamine secretion is stimulated by metabolic acidosis and in
mild cases, heart rate may be increased. In addition to these cardiovascular
effects, metabolic acidosis may also affect oxygen delivery by shifting the
oxygen-hemoglobin disassociation curve to the right.

282 Which of the following statement(s) is/are true concerning respiratory


alkalosis?
a. Exposure to high altitudes can result in respiratory alkalosis
b. Renal compensation for respiratory alkalosis is obtained by increasing
excretion of bicarbonate
c. Symptoms of respiratory alkalosis may mimic those of hypocalcemia
d. The treatment of acute respiratory alkalosis may involve a brown paper
bag
Answer: a, c, d
A primary decrease in PCO2 resulting in an increase extracellular pH is
referred to as respiratory alkalosis. Hyperventilation and the ensuing fall in
PCO2 may be secondary to hypoxia, reflux simulation from decreased
pulmonary compliance, drugs, mechanical ventilation, and other causes. The
two most common causes of hypoxia resulting in respiratory alkalosis are
pulmonary disease and exposure to high altitudes. Renal compensation for
respiratory alkalosis is not achieved by increasing excretion of bicarbonate
but by decreasing net acid excretion, primarily through the reduction in

ammonia excretion and increases in organic anion excretion. Chronic


respiratory alkalosis is generally asymptomatic. Acute respiratory alkalosis
may cause sensations of breathlessness, dizziness, and nervousness and can
result in circumoral and extremity parathesias, altered levels of
consciousness, and tetany. These signs are related to decreased cerebral blood
flow secondary to decreased PCO2 and decreased ionized calcium
concentration secondary to increased blood pH. In acute symptomatic
respiratory alkalosis rebreathing, by breathing in and out of a paper bag, can
temporarily relieve the symptoms.

283 Which of the following statement(s) is/are true concerning alterations in


serum magnesium?
a. Renal failure is the primary cause of hypermagnesemia
b. Hypomagnesemia may occur during prolonged periods of intravenous fluid
replacement
c. Symptoms of hypomagnesemia may mimic symptoms of hypocalcemia
d. Intravenous administration of magnesium sulfate is usually the most
efficient method of correction of magnesium deficiency
Answer: a, b, c, d
Renal failure is the primary cause of hypermagnesemia. Because of the
kidneys ability to excrete large magnesium loads, hypermagnesemia rarely
occurs if renal function remains normal. Because the kidneys are able to
conserve magnesium well in states of magnesium depletion,
hypomagnesemia rarely occurs from poor intake alone. The combination of
low intake and increased gastrointestinal loss may lead to hypomagnesemia.
Prolonged periods of intravenous fluid replacement without magnesium
replacement and the chronic use of loop diuretics or other medications such
as cyclosporine or aminoglycosides can also result in hypomagnesemia.
Deficiencies of magnesium may present signs and symptoms similar to

hypocalcemia. Hypomagnesemia may be treated by the oral administration of


magnesium however large doses frequently leads to diarrhea. Correction of
major deficits is therefore best managed by intravenous administration of
magnesium sulfate at a dose of 50 to 100 mEq/d.
284 Muscle relaxants are a class of anesthetic agents used to prevent
movement and facilitate surgical exposure. Which of the following
statement(s) is/are true concerning the use of muscle relaxants in surgical
procedures.
a. Succinylcholine produces rapid obvious muscle fasciculations
b. Pancuronium can be reversed by increasing the acetylcholine concentration
using an anticholinesterase inhibitor (neostigmine)
c. Prolonged periods of muscle relaxation in patients requiring prolonged
ventilation should be used in conjunction with analgesics and amnesic agents
d. The best clinical test for complete reversal of neuromuscular blockade is
the ability of the patient to produce a large negative inspiratory force
Answer: a, b, c
Neuromuscular blocking agents can be classified as depolarizing or
nondepolarizing inhibitors of the neurotransmitter, acetylcholine at the
neuromuscular junction. The only noncompetitive inhibitor employed
clinically is succinylcholine. This drug rapidly binds to the neuromuscular
junction and produces depolarization, clinically obvious as fine muscle
fasciculations occurring approximately 60 seconds after injection. All other
clinically useful muscle relaxants are termed competitive inhibitors and do
cause depolarization when they attach to the neuromuscular junction. Since
these agents compete with acetylcholine, the block produced is in direct
proportion to the concentration of the agent relative to the concentration of
acetylcholine. If the concentration ratio is low enough, competitive relaxants
can be reversed if the concentration of acetylcholine is artificially elevated.
Increase of acetylcholine concentration can be achieved by giving a drug

which blocks metabolism of anticholinesterase (neostigmine).


Nondepolarizing relaxants are frequently used in critically ill patients who
are difficult to manage otherwise during prolonged periods of mechanical
ventilation. It is imperative that these drugs be given in conjunction with
analgesics and amnesic agents, since neuromuscular blocking agents have no
analgesic or amnestic properties and only prevent motion of voluntary
muscles. Patients may therefore be totally aware and in pain and unable to
communicate. All muscles of the body do not have equal sensitivity in muscle
relaxants. The diaphragm is both resistant to neuromuscular blockade while
the neck and pharyngeal muscles that support the airway are most sensitive. It
is possible for an intubated patient to spontaneously ventilate and even to
produce a large negative inspiratory force and yet have complete airway
obstruction when extubated due to effects of residual muscle relaxants on
upper airway muscles. The definitive clinical test for complete reversal of
neuromuscular blockade is the ability of the patient to sustain a head lift from
the bed for five seconds.

285 Local anesthetics are essential agents used in current surgical practice.
Which of the following statement(s) is/are true concerning the use of local
anesthetic agents.
a. Complications due to excessive plasma concentration can result only from
inadvertent intravascular injection of the agent
b. Bupivacaine is noted for a slow onset but long duration
c. The addition of epinephrine to a local anesthetic agent will both lower the
toxicity and increase the duration of local anesthesia
d. Hypotension observed when a local anesthetic is administered in the form
of a spinal epidural block, is the result of myocardial depression
Answer: b, c
Local anesthetics constitute a class of drugs which produce temporary

blockage of nerve conduction by binding to neuronal sodium channels.


Adverse consequences associated with the use of local anesthetics fall into
three categories: acute central nervous system toxicity due to excessive
plasma concentration, hemodynamic and respiratory consequences due to
excessive conduction block of the sympathetic or motor nerves, and allergic
reactions. Whenever a local anesthetic has been injected, there may be
inadvertent intravascular injection or an overdose of the drug due to rapid
uptake from the tissues. All may produce seizures. Complications can be
minimized by aspirating prior to injection to avoid intravascular injection and
limiting the doses to the safe range. When local anesthetics are administered
for a spinal or epidural block, there will be a progressive blockade of the
sympathetic nervous system which will produce systemic vasodilatation. If
the block travels along the thoracolumbar region, a sympathetic blockade will
result in profound systemic vasodilatation and bradycardia with resultant
hypotension.
Local anesthetics are divided into two groups: esters and amides. Most
commonly used agents, the amides, include lidocaine and bupivacaine.
Lidocaine is noted for a fast onset of action but a short duration whereas
bupivacaine has a slower onset with the duration lasting for four to 12 hours.
The addition of epinephrine (100 g) will lower the toxicity and increase the
duration of the local anesthetic.

286 A 65-year-old gentleman with a history of coronary artery disease and a


recent myocardial infarction requires an elective colon resection for a
nonobstructing neoplasm. Which of the following statement(s) is/are true
concerning the risks of general anesthetic in this patient?
a. The age of the previous infarct has no effect on the perioperative
reinfarction risk
b. The incidence of reinfarction appears to stabilize after six months
c. Invasive hemodynamic monitoring has no effect on perioperative

reinfarction rates
d. Reinfarction has minimal effect on mortality
e. Perioperative infarction most frequently occurs after the first 72 hours from
surgery
Answer: b, d
The history of myocardial infarction is an important risk factor for general
anesthesia. Large retrospective studies have found that the incidence of
reinfarction is related to the time elapsed since the previous myocardial
infarction. The incidence of reinfarction appears to stabilize at approximately
1% after six months, with the highest rate of reinfarction occurring in the first
three months after the infarct. Mortality from reinfarction, for patients
undergoing non-cardiac surgery, has been reported to be between 2050%
and usually occurs within the first 48 hours after surgery. Invasive
hemodynamic monitoring with pulmonary artery catheters and aggressive
pharmacologic intervention has been demonstrated to reduce reinfarction
rates.

287 General anesthesia is not without risks. Which of the following


statement(s) is/are true concerning the risk associated with general
anesthesia.
a. Current estimates for mortality due to anesthesia alone are 1:10,000
b. Human error accounts for between 50 and 75% of anesthetic-related deaths
c. Most anesthetic-related deaths are associated with overdose of analgesic
agents
d. The most common problems associated with adverse anesthetic outcomes
are related to the airway
Answer: b, d
Anesthetic agents effectively obtund or completely block nearly all

physiologic protective mechanisms, therefore, there is an associated risk even


without a surgical procedure. Fortunately, with the advent of newer agents
and monitoring techniques, it is estimated the mortality due to anesthesia
alone has decreased from approximately 1:10,000 in the 1950s to as low as
1:100,000 or less for healthy patients today. It has been estimated that
between 5075% of anesthetic-related deaths are due to human error and are
preventable. The most common problems associated with adverse outcomes
are related to the airway: inadequate ventilation, unrecognized esophageal
intubation, unrecognized extubation, and unrecognized disconnection from
the ventilator.

288 Over the last decade, the routine use of both invasive and noninvasive
monitoring devices has been instituted for the administration of most
anesthetics. The following statement(s) is/are true concerning monitoring of
the surgical patient.
a. A pulse oximeter reading will reflect changes in PaO2 only below 80 mm
Hg
b. Monitoring of end tidal CO2 will reflect changes in ventilation but not
cardiac output
c. Intermittent, noninvasive systemic blood pressure monitoring using an
oscillometric blood pressure cuff has essentially replaced clinical
measurement by auscultation
d. Pulmonary arterial catheter monitoring is generally reserved for critically
ill patients with significant left ventricular dysfunction
Answer: a, c, d
Pulse oximetry continuously, noninvasively and inexpensively provides
arterial hemoglobin saturation and peripheral pulse determination. It must be
remembered, however, that a pulse oximeter measures oxygen saturation and
not arterial oxygen tension (PaO2). The PaO2 must drop below 80 mm Hg

before any significant change in oxygen saturation will occur. End tidal CO2
monitoring reflects metabolism (the production of CO2), circulation (blood
flow to the lungs), and ventilation (respiratory rate in an intact ventilatory
circuit). It can be used as a surveillance monitor for both the respiratory
circuit and the cardiovascular system. Any acute decrease in cardiac output
will decrease output to the lung and increase alveolar dead space, causing an
acute drop in end tidal CO2.
Hemodynamic stability can be monitored in a variety of methods, the most
basic of which is systemic arterial blood pressure measure. Intermittent,
noninvasive measure of systemic blood pressure with an oscillometric blood
pressure cuff has become the standard in the operating room with an accuracy
equal to that of clinical measurement by auscultation. When tighter control is
required in patients with significant hypertension, serious heart disease, or in
patients who may suffer acute blood loss, invasive arterial monitoring is
employed. In patients with left ventricular dysfunction who are undergoing
extended surgical procedures with significant fluid shifts and potential blood
loss, central venous pressure monitoring is frequently used, with pulmonary
arterial catheter monitoring reserved for more critically ill patients and for
those with significant left ventricular dysfunction.
289 Correct statement(s) concerning complications occurring in the postanesthetic care unit include which of the following?
a. The use of nitrous oxide has been well documented to increase the
incidence of postoperative nausea
b. Perioperative myocardial ischemia is usually easily diagnosed in the early
postoperative period
c. Hypothermia results in a deleterious effect on drug metabolism therefore
delaying recovery from anesthesia
d. The serotonin antagonist, odansetron, holds promise as the superior
antiemetic agent in the perioperative period
Answer: c, d

Twenty-four percent of patients experience a post-anesthetic care unit


complication. Nausea, vomiting and airway support comprise 70% of these
complications. The need to maintain airway support is by far the most
common respiratory complication. Hypothermia has a deleterious effect on
altering drug metabolism and delaying recovery. Nausea and vomiting are
rarely unifactorial and cause considerable discomfort to the patient. There is
little evidence to favor one anesthetic or anesthetic technique over another.
Nitrous oxide does not appear to increase incidence of nausea in well
documented studies. The new serotonin antagonist, odansetron, has been
shown in several studies to be superior to other agents as a perioperative
antiemetic agent.
Perioperative myocardial ischemia is an extremely important complication
but difficult to recognize. Diagnosis is complicated by the fact that only 10
30% of patients suffering documented myocardial infarction will have pain
and that postoperative EKG changes are often nonspecific. One must
therefore look for secondary indications of on-going ischemia such as
hypotension, arrhythmias, elevated filling pressures, or postoperative
oliguria.

290 Patient-controlled analgesia ( PCA) is a commonly used technique for


postoperative analgesia. The following statement(s) is/are true for the use of
PCA.
a. Satisfactory pain relief is provided by the administration of higher narcotic
doses
b. The technique is not applicable in the semiconscious or uncooperative
patient
c. PCA is as safe as conventional intramuscular administration of pain
medication
d. Excessive administration of narcotic medication can be limited by a

lockout duration which controls administration of the narcotic


Answer: b, c, d
The technique of patient-controlled analgesia is based on investigations that
small intravenous bolus doses of narcotic on demand can provide patients
with improved pain relief at the same or less total narcotic dose. The system
requires some degree of sophistication and a conscious patient who has been
instructed in the technique. Numerous studies have demonstrated that PCA is
as safe as conventional IM medication. The patient can be restricted from
receiving excessive agents via setting a lockout interval duration of several
minutes during which time a dose of narcotic cannot be successfully
administered. In addition, limits to the total hourly dose can be set.
291 Narcotics are commonly used in the administration of general anesthesia.
Which of the following statement(s) is/are true concerning this class of
agents.
a. Narcotics have both profound analgesic and amnestic properties
b. Narcotics can cause hypotension by direct myocardial depressive effects
c. Naloxone should be used routinely for the reversal of narcotic analgesia
d. Acutely injured hypovolemic patients are at significant risk for decreased
blood pressure with the use of narcotic analgesics
e. Propofol is a new intravenous short-acting narcotic used frequently in the
outpatient setting
Answer: d
Narcotics and synthetic analogues belong in the class of drugs called opioids.
Narcotics produce profound analgesia and respiratory depression. They have
no amnesic properties, no myocardial depressive effects, and no muscle
relaxant properties. Narcotics may produce significant hemodynamic effects
indirectly through the release of histamine and/or blunting of the patients
sympathetic vascular tone due to analgesic properties. Acutely injured

patients may be hypovolemic and in pain, with high sympathetic tone and
peripheral resistance. Therefore, such patients can experience a dramatic drop
in systemic blood pressure with minimal doses of opioids. All opioids can be
reversed with naloxone. Naloxone reversal, however, can be dangerous
because the agent acutely reverses not only the analgesic effects of the opioid
but also analgesics effects of native opioids. Naloxone treatment has been
associated with acute pulmonary edema and myocardial ischemia and should
not be used electively to reverse the effects of narcotic. Propofol is a lipidsoluble substitute isopropyl phenol non-narcotic agent that produces rapid
induction of anesthesia followed by awakening in four to eight minutes.

292 Anesthetic techniques used in the management of patients with


significant pulmonary disease include:
a. Intubation at a deep level of anesthesia
b. Choice of an anesthetic agent which produces bronchodilatation
c. The use of epidural analgesia for postoperative pain control
d. Perioperative use of intermittent positive pressure breathing
Answer: a, b, c
Patients with significant pulmonary diseases require special anesthetic techniques.
Obstructive pulmonary disease can either be chronic (COPD) or acute (asthma). In
either case, the reversible component of obstruction should be reversed prior to
elective surgery. In patients with reactive airway disease, the endotracheal tube
may induce severe bronchospasm. Even in patients who are well treated
preoperatively, reactive bronchospasm may complicate anesthetic induction and
emergence from anesthesia. The principal method used to prevent or diminish this
foreign body induced bronchospasm is intubation of the patient at a deep level of
anesthesia when reflexes are blunted. The classic way of managing a patient with
severe asthma is to induce with an agent that produ
bronchodilatation and to ventilate the patient with an inhalation agent until deeply
anesthetized prior to laryngoscopy and intubation. The patient should be extubated
while spontaneously ventilating, but with the inhalation agent still in effect, bringing

the patient to consciousness while ventilating by mask.


Because of the potential adverse effects of systemic narcotics on respiratory drive,
the use of epidural narcotics and local anesthetics for postoperative pain control has
become very popular. These techniques allow the patient to be extubated earlier,
and patients with intrathoracic and upper abdominal surgery, help restore
pulmonary function toward preoperative values. Preoperative use of intermittent
positive pressure breathing has not been demonstrated to decrease the incidence of
postoperative pulmonary complications.

293 Which of the factors listed below will adversely affect the risk of perioperative
cardiac complications and reinfarction in the patient described above?

a. Greater than five premature ventricular beats per minute on EKG rhythm strip
b. The anesthetic technique used

c. Withdrawal of medical therapy with beta blockers and topical nitrates


d. Length of surgical procedure less than three hours
e. Known three vessel coronary artery disease
Answer: a, c, e
The incidence of reinfarction is increased in patients undergoing intrathoracic
or intra-abdominal procedures lasting longer than three hours. The site of
surgery or anesthetic technique have not been shown to change the incidence
of reinfarction if the procedure is less than three hours in duration. Patients
with known three-vessel or left main coronary artery disease are at increased
risk, while those who have undergone prior coronary artery bypass grafting
are of substantially decreased risk of reinfarction. Prophylactic therapy with
beta blockers, calcium channel agents, and nitrates has not been proven
beneficial; however, withdrawal of these agents has been associated with
perioperative ischemia, myocardial infarction, and death. CHF is the single

most important factor predicting postoperative cardiac morbidity. Rhythm


disturbances, particularly frequent premature ventricular beats, more than
five beats/minute, are also independently associated with an increased risk of
perioperative cardiac complications.

Você também pode gostar